You are on page 1of 127

Radiant Notes—FCPS PEARLS Golden File 4 By: Dr.

Rafi Ullah

Radiant Notes—Fcps Pearls


Golden file 4 including March & May papers 2017
Below is the contents, preface and sample chapters of the coming book FCPS PEARLS 4th Edition.

Important Note
This book is the most updated book—with the most corrected answers with refernces and explanation
where necessary
I have given references with all the controversial bcqs so don’t follow Facebook posts answers blindly.
If U found any mistake, let me know, to continue this chain of helping others
May Allah succeed u all
Least u can do is pray for me, if u like it. Have did a lot of effort in finding references and correct
answers
Share it if you like it. JazakAllah.
Book releasing date= 3rd February 2018
Join our Facebook group

Group Members
 Dr. Tanveer (Author Of Tanveer Notes)
 Dr. Rabia Ali (Author Of Rabia Ali)
 Dr. Rafi Ullah (Author Of Fcps Pearls & Fast Aid)
Our other important post
o Golden file 3 including September Oct and November papers 2017
 https://web.facebook.com/groups/854578911349847/938752202932517/
o Fast aid sample chapters including endocrinology, general anatomy, Histology
 https://web.facebook.com/groups/854578911349847/937882296352841/
o Golden File zero
This book will help the students in many ways including
It includes notes covering all subjects and each and every chapter, all these notes are made
from the topics from which BCQ’s comes repeatedly.These noteswill solve more than 6000 old
and new BCQ’S. It will cover 2015 papers BCQ’s as well.
It also contains CPSP demo questions, with CPSP key that are repeatedly asked in exams
It contains a special golden file containing golden file zero, one and two where all the
controversial BCQ’s and most repeated MCQ’s are given with explanation and references
It contains Golden file zero, 1, 2, 3, 4 & 5 with the most authentic key
Golden file 5 contains the recent September and November papers 2017
Radiant Notes—FCPS PEARLS Golden File 4 By: Dr. Rafi Ullah
Radiant Notes—FCPS PEARLS Golden File 4 By: Dr. Rafi Ullah
Radiant Notes—FCPS PEARLS Golden File 4 By: Dr. Rafi Ullah

Golden file 4

Includes March and


May 2017 papers

“If You Are Working On Something That You Really Care About, You Don’t Have To Be
Pushed. The Vision Pulls You.”- Steve Jobs
Radiant Notes—FCPS PEARLS Golden File 4 By: Dr. Rafi Ullah
12. Structure present in renal column.
Medicine and Allied 1st March a.
b.
Inter lobar artery
Intra lobar artery.
2017 Morning c. Micro calyx
1. Thymus blood supply d. Medullary rays/collecting ducts
a. Costocervical trunk Ans: A
b. Thyrocervical trunk 13. Pneumothorax scenario?
c. Subclavian artery Ans: Ipsilateral lung collapse inward and chest spring
d. Inf thyroid artery outward
Ans: D 14. Toradol acts on which receptor?
2. Colorectal Ca APC gene mutations biomarker a. Mu
Ans: CEA b. Kappa
3. Isthmus crosses what Ans: A
a. 2 to 4 tracheal ring 15. Clotrimazole where it acts?
b. Hyoid a. Ribosomes.
Ans: A b. Phospholipase
4. Which drug acts on NA+/K+ ATPase and its Ans: B
inhibitors? 16. Hypercalcemia caused by
Ans: Digoxin a. Loop diuretics
5. Tongue ant 2/3rd lymphatic drainage b. Thiazide diuretics
Ans: Submandibular Ans: B
6. Basophil comes from 17. Hypercalciuria cause
a. Lysosomes a. Furosemide
b. RER b. Thiazide
c. Mitochondria Ans: A (if asked management than Thiazide)
Ans: B 18. The Maximum conductivity of AV node is.
7. Something like different In etiology Of cell line a. 290 impulse/min.
a. Osteoblast b. 200 impulse/min.
b. Osteocyte c. 350 impulse/min.
c. Osteoclast d. 230 Impulse/min
d. Chondroblasts e. 400Impulse/min.
e. Chondrocyte Ans: D
Ans: C 19. Adrenal Medullary substance comprising of how
8. Lower esophageal constrictor will open much of Adrenal Gland.
a. Before peristaltic wave a. 80%.
b. After peristaltic wave b. 50%.
Ans: B c. 28%.
d. 90%.
9. Achalasia, decrease lower esophageal sphincter e. 10%.
tone due to defect in Ans: C
a. Myenteric
b. Meissner 20. New Cortical cells in Adrenal Gland is
c. Vagal continuously formed by.
Ans: A a. Zona Reticularis.
b. Zona Fasciculata.
10. Psuedostratified which is true? c. Zona Glomerulosa
a. All cells lie on basement membrane. d. Renal Capsule
b. Nuclei are at different levels. e. Perinephric Cells
c. Attached to basal lamina /basement Ans: C
membrane
Ans: A 21. Which of the following does not inhibit the
Endothelin 1 secretion.
11. 6 week child normal at birth now presented with a. Anxiety.
jaundice and bilirubin stained diaper cause b. Shear Stress.
a. Gilbert syndrome c. Fear.
b. Crigler najjar d. Awake state.
c. Hemolytic anemia e. Sleep
d. Biliary atresia Ans: B
Ans: D
22. Most Common mendelian disorders
Radiant Notes—FCPS PEARLS Golden File 4 By: Dr. Rafi Ullah
a. Autosomal Dominant 34. Pupillary light reflex nerves
b. Autosomal Recessive a. 2-3
c. Fragile X syndrome b. 3-4.
d. Prader Willi. c. 6
Ans: B Ans: A
23. Peripheral nerves surrounded by 35. Which of the following is most likely to lead to a
a. Epineurium metabolic alkalosis?
b. Perineum a. Ectopic ACTH syndrome
Ans: A b. Cushing’s Disease
24. 02-Hb Curve shift to Right c. Addison’s disease
a. Acidosis d. Hyperthyroidism
b. Dec temp e. Hypothyroidism
Ans: A Ans: B
25. Pregnant lady, pallor spoon Shape nails 36. An Arterial Blood sample revealed following
a. Low MCV, MCHC. And low MCH values in an inpatient settings at an ICU.ph 7.3,
b. Low MCV, MCH and high MCHC PO2 60 mmHg, PCO2 57mmHg and HCO3 30
Ans: A mmol/l. What could be the cause of his
26. Pregnant lady with raised hepatic enzymes. presentation?
Remote area visit a. Lobar Pneumonia.
a. Hep A. b. Post Lobectomy
b. Hep E c. Neuromuscular weakness.
Ans: B d. Acute severe Asthma
e. Left ventricular Failure
27. Cheilosis, seborrhic dermatitis and gingivitis due Ans: D
to Def of
a. Riboflavin 37. Bacterial Vaginosis DOC
b. Vit C Ans: Metronidazole
Ans: B 38. Raised ALT, AST of CLD, Mallory bodies in
28. Old lady difficulties in swallowing, bruises skin, microscope due to...
poor lady can’t type on keyboard? Ans: Alcoholic Hepatitis
a. Crest syndrome 39. A patient has lost 2 liters water by sweating and
b. Scleroderma replaced by 2 liters pure water. What will happen
Ans: A a. Decrease intracellular volume
29. Causes anabolic hepatic protein synthesis and b. Decreased intracellular osmolarity
muscle catabolic protein. c. Increased extracellular osmolarity
a. Insulin. d. Increased ECF volume
b. Cortisol. e. Decreased ECF osmolarity
c. Thyroid. Ans: D (in sweating more water is lost than solutes so
d. GH inc ECF osmolarity as a result water will move from ICF
Ans: B TO ECF- than after drinking pure water also –inc ECF
volume)
30. Crescents are protective for Glomerulus. They
are produced by 40. Correct sequence of events?
a. Mesangium a. Damaged valve, thrombus, perforation,
b. Epithelial cells emboli
Ans: A b. Damaged valve , perforation , thrombus ,
bacteremia
31. In hyperthyroidism c. Damaged valve , bacteremia , thrombus ,
Ans: Dec TSH perforation
32. Cough. Sore throat, low grade fever. Enlarge d. Bacteremia , thrombus , perforation , emboli
cervical lymph node X-ray chest show hilar Ans: A (Correct sequence is DTBP)
lymphadenopathy 41. Stagnant hypoxia is caused due to:
a. Tb. a. COPD
b. Sarcoidosis b. Polycythemia
Ans: A c. Severe anemia
33. Accident but talks a lot and irrelevant Ans: B
a. Wernicke 42. Brodie's abscess
b. Broca’s a. Pyogenic osteomyelitis
Ans: A b. Pyogenic arthritis
Radiant Notes—FCPS PEARLS Golden File 4 By: Dr. Rafi Ullah
c. Tuberculous osteomyelitis 52. Death is caused by bilateral damage to which
d. Tuberculous arthritis nerve
Ans: A a. Vagus
43. Regarding Acetaminophen most likely is: b. Hypoglossal
a. Antiflammatory properties c. Trigeminal
b. is uricosuric d. Facial
c. resembles methadone or something like that Ans: A
d. its therapeutic window is less than aspirin 53. First line of defense against microbial pathogens
Ans: D a. Skin
44. Transport tubular maximum for proteins: b. Complement
a. 10 per minute or something like that c. Opsonin
b. 20 d. Immunoglobulins
c. 30 Ans: A
d. 40 54. In normal person decreased acid secretion
e. 50 causes
Ans: C a. Abnormal protein digestion
45. True hermaphrodite: b. Increase gastric secretion
a. XXY c. Inhibition of acid secretion
b. XY Ans: A
c. XYY 55. Regarding SLE most appropriate is
d. XO a. Anticentromere antibodies acting against
Ans: A antibodies
46. Diabaetes with complication b. B and T-cells acting against their immune or
a. Dry gangrene something
b. Gas gangrene Ans: B
c. Wet gangrene 56. You have to give your house officer some blood
Ans: C to check arterial blood gases
47. Policeman tested for drugs and opium and a. Plasma in citrated tube
cannabinoids were found in urine. The first b. Whole blood in a heparinized syringe
psychological response would be Ans: B
a. Delusion of grandeur 57. Person has sharp wound on side of neck blood
b. Denial is gushing out, the senior registrar will press on
c. Depressive mood which anterior tubercle
d. Rationalization a. 6
Ans: B b. 7
48. Pineal gland tumor compression in 10 years old c. 10
boy causes decreased secretion of melatonin d. 12
a. Delayed puberty Ans: A
b. Early puberty 58. Purkinje fibers are fastest in transmitting
c. Increase serotonin impulse because
Ans: B a. They have wider diameter
49. Person is in prolonged coma b. They have large number of sodium channels
a. Nucleus cerelous c. Intercalated junction
b. Periaqueductal Ans: A
Ans: B 59. A child has on/off bruises and epistaxis increase
50. Anti-manic effect of lithium takes how much time BT, normal PT and aPTT
a. 10 days a. vWb
b. 15days b. ITP
c. 21 days c. Hemophilia
Ans: B Ans: B
51. Wave of depolarization passes from endo to 60. Characteristic of vWb:
epicardium a. Prolonged APTT
a. RR interval b. Increase BT
b. QT interval c. Increase PT
c. QRS interval Ans: B
Ans: C 61. What is the composition of normal saline
a. 0.9 gm in 10 ml distilled water
Radiant Notes—FCPS PEARLS Golden File 4 By: Dr. Rafi Ullah
b. 9 gm in 1000 ml distilled water (ANS) 71. Regarding synthesis of aqueous humor
c. 0.9gm in 100 ml distilled water a. Ultrafiltration
Ans: B b. Ultrafiltration + active secretion
62. Maximum left ventricular pressure in a young c. Diffusion
healthy individual d. Active secretion
a. 25 Ans: B
b. 50 72. Premalignant condition
c. 80 a. Behcet’s
d. 120 b. Lichen planus
Ans: D c. Pemphigus vulgaris
63. A young athlete injured his ankle and develops d. Pemphigoid
ecchymosis in ankle...he can stand on his toes Ans: B
but it’s painful 73. Nonsmoker tobacco addict presents with growth
a. DVT in floor of mouth, diagnosed as sub mucosal
b. Achilles tendon damage fibrosis
c. Plantaris tendon damage a. Premalignant lesion
Ans: C b. Dysplasia
64. Cervical segment of spinal cord does not have c. Benign tumor
a. Lateral horn Ans: A
b. Ventral horn 74. About sickness
c. Dorsal root a. Feeling of unwell as perceived by patient
Ans: A b. Perceived by physician
65. What is not present at L4 c. Clinical
a. Lateral horn Ans: A
b. Ventral horn 75. High blood flow, oxygen consumption of kidney
c. Dorsal horn will be
Ans: A a. Remains same
66. What travels along with lateral corticospinal b. Decreased
tract? Ans: A
a. Rubrospinal tract 76. Na-K ATPase activity increased by
b. Vestibulospinal tract a. Dopamine
c. Spinothalamic tract b. Noradrenaline
Ans: A c. Adrenaline
67. L4 pain radiates to d. Insulin
a. In front of knee Ans: D
b. Lateral calf 77. Most likely regarding acute tubular acidosis
c. Post calf or medial calf can't remember a. Hyperkalemia
d. Post thigh b. Raised urea and normal creatinine
Ans: A Ans: A
68. Sesamoid cartilage in body 78. Patient has B.P 80/50, Cardiac output of 2L/min,
a. ALA of nose central venous pressure of 2 cm of water, lactate
b. Ear 30
c. Larynx a. Cardiac tamponade
d. Epiglottis b. Congestive cardiac failure
Ans: A c. Septicemic shock
69. Which of following has a strong bronchodilator d. Hypovolemic shock
effect? Ans: D
a. Albuterol 79. Tumor suppressor gene
b. Ipratropium a. P53 for Wilms tumor
c. Atenolol b. WT1 for pancreatic Ca
Ans: B c. APC for colon cancer
70. About fibrocartilage Ans: C
a. It contains an amorphous substance 80. Nucleus disappears in which stage
b. It is present inside disc of joints a. Late normoblast
c. Once damaged cannot regenerate b. Early normoblast
d. Contains elastic fibers c. Reticulocyte
Ans: B Ans: A
Radiant Notes—FCPS PEARLS Golden File 4 By: Dr. Rafi Ullah
81. Thyroid movement 92. Hemoglobin binds to:
a. Prevertebral fascia a. Haptoglobin
b. Pretracheal fascia b. RBC
Ans: B Ans: A
82. Working women in kitchen cut radial artery 93. Man living in desert, drinks a lot of water,
accidentally, response will be maximum absorption from?
a. β fibers a. PCT
b. Post ganglionic C b. DCT
c. Post ganglionic B c. CD
d. Delta Ans: A
Ans: B 94. Synergistic effect of loop and thiazide?
83. Right nasal and left temporal fibers gone a. Hypercalcemia
a. Lesion at left tract b. Hyperkalemia
b. Lesion at right tract c. Hypokalemia
Ans: A Ans: C
84. Pancreas cytoplasm stains blue due to 95. Thiazide + loop diuretic synergistic action
a. Lysosomes occurs on?
b. Mitochondria a. PCT
c. SER b. Collecting
d. RER c. DCT
Ans: D d. Thick ascending
85. With superior thyroid, which nerve lies Ans: C
a. Recurrent laryngeal 96. Synapses absent in:
b. External a. Ventral column
c. Jugular b. Dorsal Column
Ans: B c. Lateral Column
86. S4 is produced: d. Sympathetic chain
a. Ventricular filling e. Dorsal root ganglion
b. Ventricular filing during atrial systole Ans: E
c. Contraction 97. Which causes max volume increase:
Ans: B a. Hypersonic NaCl
87. Pregnant woman taken only iron throughout her b. Isotonic NaCl
pregnancy, what will be deficient? c. Dextrose
a. Folic acid Ans: A
b. Vitamin A 98. RTA, can see but can`t recognize:
c. Vitamin B a. Frontal
d. Vitamin C b. Parietal
Ans: A c. Wernicke’s area
88. Vitamin A deficiency d. Visual association area
Ans: Night blindness Ans: D
89. Seventeen dehydrogenase or something is 99. Heavy water used to measure
absent, which of following conversion reaction a. ICF
will be stopped: b. TBW
a. Cholesterol to progesterone c. ECF
b. Progesterone to prognolone Ans: B
c. progesterone to hydroxy-progesterone
100. Infective endocarditis
Ans: C.
a. Strep viridans
90. Have to prescribe vitamins: b. Strep
a. Eggs c. Staph
b. Milk Ans: A
c. Vegetables
101. Abscess of some area:
Ans: C
a. Staph
91. Tracheostomy, heat loss: b. Strept
a. Evaporation Ans: A
b. Conduction
102. Oxygen toxicity in sea divers, cause of death
c. Convection
a. Drowsiness
Ans: A
Radiant Notes—FCPS PEARLS Golden File 4 By: Dr. Rafi Ullah
b. Vomiting c. Night
c. Seizures d. Sleep
Ans: C Ans: A
103. Severe diarrhea: 114. H2 receptors decreases acid secretion via:
a. Metabolic acidosis normal anion gap a. Both ↓H+ secretion and vagal
b. Metabolic acidosis low anion gap b. Vagal
c. Metabolic acidosis high anion gap c. H+ inhibition
Ans: A Ans: A
104. Respiratory acidosis scenario: 115. Atypical lymphocytosis, sore throat
a. Asthma a. Monospot
b. COPD b. Mantoux
Ans: B Ans: A
105. Transverse diameter of rib cage: 116. Cervical branch of facial supplies:
a. Diaphragm a. Sternocleidomastoid
b. Bucket handle movement b. Platysma
c. Pump handle movement c. Genioglossus
Ans: B Ans: B
106. Dysphagia, endoscopy shows circular patch at 117. Mycobacterium leprae
mid esophagus biopsy shows polygonal cells a. Intracellular acid fast
with pleomorphic changes: b. Extracellular acid fast
a. H- pylori c. No Acid fast
b. Fiber diet Ans: A
c. Alcohol 118. Scenario upper respiratory infection lymphocyte
Ans: C normal decrease plasma
107. Patient had hypocalcemia and a. A-gamma was not in options
hyperphosphatemia, injection of PTH increased b. Common variable deficiency something
cAMP in kidney, cause: c. IgA deficient
a. Pseudo hypoparathyroidism Ans: B
b. Hyperparathyroidism after surgery 119. Travelers’ diarrhea DOC
c. Vit d intoxication a. Diphenoxylate
d. Vitamin D deficiency b. Norfloxacin
Ans A Ans: B
108. A long scenario with giant platelets 120. Female, which one of following having least
Ans: Bernard Soulier syndrome chances of Miscarriage at 17 – 26 weeks
109. Drug receptor interaction? a. Uterine abnormalities.
a. Heparin and protamine sulphate b. Fetal hydrops
b. Mannitol as a diuretic c. Rh incompatibility
c. Beta blocker Ans: C
Ans: A 121. Present and absent of vomiting data will be
110. Methyldopa MOA in hypertension: a. Nominal
a. Blocks adrenergic in CNS b. Ordinal
b. Directly act Ans: A
c. Block beta receptors 122. Surgeon doing open abdominal surgery, HO
Ans: A spills 1-L N/S drip on patients pelvic brim, fluid
111. Zona Glomerulosa secretion controlled by will go into?
a. Angiotensin 2 a. Right left paracolic gutter
b. Renin b. Spleen
c. Sympathetic c. Back
d. K Ans: A
e. Na 123. Basilar artery gives
Ans: A a. Anterior cerebellar artery
112. Ulcerative colitis, DOC: b. Posterior cerebral artery
Ans: Sulfasalazine c. Middle
Ans: B
113. Circadian rhythm bronchoconstriction maximum
a. Morning 124. B-lymphoma associated with which virus
b. Noon a. CMV
b. EBV
Radiant Notes—FCPS PEARLS Golden File 4 By: Dr. Rafi Ullah
c. HPV d. Supraspinatus
Ans: B e. Serratus anterior
125. Pap smear findings, nuclei large present. Advice Ans: E
to do Pap smear after 6 months: 136. Cells in retina, self-generating impulse
a. Metaplasia a. Bipolar
b. Hyperplasia b. Amacrine
c. Dysplasia c. Ganglion
d. Pre malignant d. Horizontal
Ans: C Ans: C
126. Scenario of Achondroplasia 137. Which vein directly drain to SVC
a. Autosomal Dominant a. Azygous
b. Autosomal Recessive b. Hemi azygous
Ans: A Ans: A
127. Physiologic shunt will be present when V/Q is 138. Creatinine phosphokinase is absent in??
a. Infinite a. Brain
b. Zero b. Muscle
c. Normal c. Liver
d. One d. Myocardium.
Ans: B Ans: C
128. Propylthiouracil mechanism of action 139. Small intestine
Ans: Inhibits thyroid hormone synthesis Ans: 5 to 7 meter
129. Female can't stand from sitting but can walk 140. Small intestine?
a. Gluteus maximus Ans: microvilli present
b. Gluteus minimus 141. Stroke volume increased but neither pulse
Ans: A pressure nor mean arterial pressure increased.
130. Longest pre erythrocyte phase schizonts in What will be the effect?
a. P. Malariae a. Pulse pressure raised and mean arterial
b. P. Ovale pressure increases
c. P. Falciparum b. Pulse pressure raised and mean arterial not
Ans: A raised
131. Injury near ankle can stand but painful walking c. Coronary blood flow increases
a. Achilles rupture d. No change
b. Plantar rupture Ans: D
Ans: B Medicine and Allied 1st MARCH 2017
132. Young type-I diabetic female with morning Evening
hyperglycemia, insulin regime 1. Carbohydrates source in diabetic patients
a. Long acting insulin twice daily
a. whole wheat
b. Short acting twice daily
b. Bread
c. Intermediate twice daily
d. Regular before each meal c. Cakes n biscuits
e. Short and intermediate twice daily Ans: A
Ans: A(Ref: pharmacology pharmacy technique page 2. Richest source of vitamin D
549 chapter 33 which states that long acting OR a. Cod liver oil
intermediate acting insulin twice a day) b. Milk
133. Pap smear Ca cervix scenario, most common Ans: A
causative agent 3. Myasthenia gravis patient taking neostigmine
a. HPV 16 and atropine developed some weakness
b. HPV 18 proximal and improved by giving Edrophonium
c. CMV what will we do
Ans: A a. Decrease dose of neostigmine
134. Lesion on vulva b. Increase dose of neostigmine
Ans: Condyloma accuminatum c. Stop atropine or inc or Dec atropine
135. Man trying to scratch his back and can`t, muscle Ans: B
damaged? 4. How to differentiate between normal and SIADH
a. Latissimus dorsi a. Plasma osmolarity
b. Pectoral is major b. Urine osmolarity
c. Subscapularis
Radiant Notes—FCPS PEARLS Golden File 4 By: Dr. Rafi Ullah
c. ADH levels in plasma b. Increase gastrin secretion
Ans: A Ans: A (because acid activate pepsinogen into
5. Son came with his father behavior odd then pepsin which in turn digest protein so absence of
asking apology what to give acid would affect protein digestion)
a. Lithium 16. Anti-manic effect of lithium takes how much
b. Haloperidol time???
c. Thioridazone a. 10 days
Ans: A b. 15 days
6. What is not a mechanism against hypothermia c. 5 days
a. Hyperventilation d. 21 days
b. Eccrine sweat Ans: B
c. Apocrine 17. Interstitial fluid estimation
d. Shivering Ans: Heavy water
Ans: A o the question is incomplete but if it says how
7. Partial alleviation of thirst by to estimate interstitial fluid volume answer Is
a. SIADH the difference between extracellular volume
b. gastric distension and plasma volume…interstitial fluid volume
Ans: B does not have any marker to calculate,
8. Which is not present in spinal cord at L4 heavy water is used to calculate
a. lateral horn extracellular volume
b. ventral horn 18. Histologically differentiate b/w gallbladder and
c. gray matter large intestine
Ans: A a. Columned lining
9. Fracture of sub-occipital which artery will b. No Muscularity mucosa
damage Ans: B
a. Occipital 19. Patient has oat cell carcinoma secreting ADH.
b. Auriculotemporal what he will have
c. Vertebral artery a. Hyperosmotic over hydration
Ans: C b. Hyposomotic over hydration
10. Which one causes innate immunity? Ans: B
a. NK cells 20. History of prolonged bleeding from umbilical
b. Complement system stump. Plus two cousins have history of
Ans: A prolonged bleeding. Which investigation
11. Cancer in humans due to a. PT
a. Apoptosis b. APTT
b. One less chromosome Ans: B
c. One extra chromosome 21. Basal ganglia damage will cause all except
d. Over expression of proto oncogene a. Involuntary movements
Ans: D b. Postural disturbance
12. Rt Hypoglossal nerve damage c. Intention tremors
a. Tongue deviated on Rt side with atrophy d. rigidity in limbs
b. Tongue deviated on same side without Ans: C
atrophy 22. Which vessel in Sub occipital triangle?
Ans: A a. Sub occipital nerve
13. Fibers of pain n warmth b. greater occipital nerve
Ans: C fibers c. occipital artery
14. First line of defense against microbial organism d. Post auricular artery
a. Skin e. Vertebral artery
Ans: E
b. complement
c. opsonin 23. PH. 7.36; HCO3 16 and there were 2 values
Ans: A pCO2. 40 and 30. Which condition?
15. In normal persons decrease acid secretion a. Normal
causes what b. Uncompensated respiratory acidosis
a. Abnormal protein digestion c. Compensated respiratory acidosis
Radiant Notes—FCPS PEARLS Golden File 4 By: Dr. Rafi Ullah
d. Compensated metabolic acidosis a. Secondary wave stared by swallowing
Ans: D center
24. Male non-smoker. One year cough history. X ray b. Secondary wave started by intrinsic
shows mass in peri-hilar which was squamous muscle
cell Ca. Cause? Ans: B
a. Radon 36. Diabetic gastro-paresis
b. Benzene Ans: Metoclopramide
c. Silica 37. Which is the secondary ossification center for
d. Asbestos bone formation
Ans: D a. Membranous
25. Communication b/w basilar and internal carotid b. Epiphysis
Ans: Post communicating artery c. Diaphysis
26. In Wilson's disease? d. Metaphysis
Ans: Increased copper e. Epiphyseal plate
27. Bundle of HIS supplied by? Ans: B (primary ossification center are
Ans: RCA diaphysis/shaft. While secondary ossification center
28. One scenario was all bleeding profile normal appear in the epiphyses)
except PT prolonged which factor deficient 38. Bronchopulmonary segments in right lung
a. 2 Ans: 3, 2, 5 (3 in superior lobe, 2 in middle lobe and
b. 5 5 in inferior lobe)
c. 7 39. Decrease hearing for lower frequency
d. 10 a. Ear wax
Ans: C b. Conductive deafness
29. Drug for hypertrophic cardiomyopathy? c. Sensorineural deafness
Ans: Verapamil d. Neural deafness
30. Stroncium-90 studied in area of nuclear bomb. Ans: C
Found in the food chain and contamination of 40. Which hormone inhibits hormone sensitive
milk. Which carcinoma it will cause? lipase
Ans: Osteosarcoma a. Insulin
31. Richest source of Vit D b. Glucagon
a. Margarine c. Adrenaline
Ans: A
b. Butter
c. Milk 41. Cold intolerance, coarse voice, weight gain, Dec
d. Cod-liver oil appetite diagnostic test?
Ans: D Ans: TSH
32. Patient is given oral med. Which drug is 42. Insulin is inhibited by
absorbed by orally? a. Beta agonist
Ans: Digoxin b. Beta blockers
33. Patient with a murmur. QRS complex 180 axis Ans: B
deviation. Diagnosis? 43. A person has previous history of jaundice 10
a. Aortic stenosis years ago. Now AST ALT both 100 plus. DB of 8
b. Aortic regurgitation histology shows ballooning degeneration. What
c. Mitral stenosis is pt likely to have further?
d. RBBB a. Alpha-1 antitrypsin deficiency
e. LBBB b. HBsAg
Ans: E c. Anti-mitochondrial antibodies
34. Patient was having nausea vomiting and vertigo. d. Raised ceruloplasmin
Ans: C
a. Vagus nucleus juxtaposed over
vestibular 44. Beta receptor stimulation causes
b. Dorsal nucleus a. Dec heart rate
Ans: A b. Piloerector
35. Large bolus stuck after eating what will happen c. Glycogenolysis
next? Ans: C
45. Adrenaline causes
Radiant Notes—FCPS PEARLS Golden File 4 By: Dr. Rafi Ullah
a. Dec heart rate Ans: C
b. Inc peripheral resistance 56. Patient had RTA. Now behave change lack of
c. mental alertness motivation. Lesion
Ans: B a. Frontal
46. SA node acts as pacemaker because b. Parietal
a. Strong outward K current c. Temporal
b. Located in right atrium Ans: A
c. Highest firing rate 57. MI 9th day suddenly die in washroom?
Ans: C Ans: Cardiac tamponade
47. Ear infection in oncology ward by pseudomonas, 58. Person taking ATT big toe swelling
fever caused by Ans: Pyrazinamide
a. TNF 59. Person taking ATT pins and needles sensation
b. IL-1 Ans: Isoniazid
Ans: A 60. Fever for 6 months hilar lymphadenopathy
48. Adult resting male heart rate 160. Bp. 130/90. caseous necrosis
CVP 8. Cause Ans: TB
a. Internal bleeding 61. 60 years old person generalized
b. Essential HTN lymphadenopathy peripheral film mature
c. Sympathetic over activity lymphocytes
Ans: C
Ans: CLL
49. After transplant within ten minutes cyanosed? 62. Connection b/w ocular movements and
Ans: antibody mediated rejection vestibular balance
50. 6 months after transplant sign symptoms of a. Medial longitudinal fasciculus
renal failure which improved on b. Medial leminiscus
immunosuppressive. Cause Ans: A
a. Cyclosporin toxicity 63. Taste fibers from 7, 9 and 10 nerves through
b. Acute humoral rejection which tract
c. Acute cellular rejection Ans: Solitary tract
Ans: C
64. Peripheral nervous system?
51. A girl with recurrent staph abscesses. Also had
a. Spinal nerves only
two bouts of aspergillosis. Cause
b. Spinal plus autonomic ganglia
a. NADPH Def. c. Spinal and cranial
b. IgA Def. Ans: B
Ans: A (ref first aid Ed: 2017 page 113)
65. Tactile sensation on tip of fingers
52. First barrier against micro-organisms
a. Meiseners
Ans: Skin
b. Merkel
53. Visual image of motor activity/actions to be Ans: B (Ref first aid 2017 page 464)
performed at? 66. Damage to Middle meningeal artery causes
a. Retina hematoma formation between
b. Ant Supplementary Motor Area a. Dura matter and arachnoid matter
c. Post Sup Mot Area b. Arachnoid matter and pia matter
d. Ant And Post Premotor Area c. Dura matter and calavaria
e. Motor Cortex d. Pia matter and cortex
Ans: D Ans: C
54. Levator ani nerve supply? 67. Emphysema?
Ans: Pudendal (It is primarily supplied by direct
a. Inc surface area Dec oxy tension
branches of the sacral plexus (S3-S5). To a small b. Dec surface area and tension
degree by the pudendal nerve) Ans: B
55. Patient dies on 4th day after MI. On histo 68. Cushing triad
predominantly Ans: Raise ICP, HTN, Bradycardia
a. Neutrophils
69. Occulocardiac reflex initial step of management
b. Lymphocytes
Ans: Remove stimulus
c. Macrophages
d. Plasma cells 70. Increased amniotic fluid. Raised AFP. Cause
Radiant Notes—FCPS PEARLS Golden File 4 By: Dr. Rafi Ullah
a. Anencephaly 83. Patient post thyroidectomy. Hoarseness of
b. Renal agenesis voice. Loosening of vocal cords. Damage to
Ans: A a. Cricothyroid
71. 1st pass metabolism b. Post Crico-arytenoid
Ans: Oral route Ans: B
72. Patients diseased or not studied for the risk 84. Na k ATPase increased by which hormone
factors whether they develop disease or not a. Dopa
a. Cohort b. Adrenaline
b. Case control c. Insulin
Ans: A Ans: C
73. Chi square 85. 12 years old child with MCV 126. Investigation?
Ans: 2 by 2 table a. Intrinsic factor antibody
74. Primary oocyte completes first meiotic division b. Serum B12 level
a. Just after ovulation Ans: B
b. Before ovulation 86. About human placenta
c. During 3rd intrauterine month a. 2 umbilical veins
d. During 6th intrauterine month b. 2 artery 1 vein
Ans: B Ans: B
75. Visited Thailand, diarrhea doughnut shaped cyst 87. Female 45 XO.
Ans: Cryptosporidium a. Gonadal dysgenesis and germinal
76. UTI scenario gram negative rod hypoplasia
Ans: E.coli b. Wide spaced nipple and low bridge nose
77. Respiratory acidosis cause? c. Short stature with simian crease
a. barbiturates poisoning Ans: A
b. aspirin 88. Pyogenic peritonitis cause
c. Nikethamide a. E.coli
d. high altitude b. Bacteroids
Ans: A c. Klebsiella
78. Traveler’s diarrhea. Treatment? d. Pseudomonas.
a. Diphenoxylate Ans: B
b. Nitrofurantoin 89. Right leg cordotomy done and pain relieved.
Ans: A Cause
79. Newborn with an Erythroblastosis Fetalis having a. Left dorsal column cut
Blood group B positive what is best for b. Left ventral spinothalamic cut
management? c. Left lateral spinothalamic cut
a. transfusion with O-ve Ans: C
b. exchange with AB-ve 90. A patient goes onto a shock like state
c. B negative a. CNS ischemic response
Ans: C b. Vasodilation
80. Secretary diarrhea cause Ans: A (if sudden then baro if potent then CNS
Ans: Cholera toxin (and if asked osmotic diarrhea ischemic if long lasting then angio-renin)
then Lactose intolerance) 91. Sacral autonomic
81. Mother has history of hemolytic disease of a. S2,3,4
newborn. Now 26 wk. pregnancy. What to do b. S3,4,5
a. Indirect coombs on mother Ans: A
b. Check Fetal Rh-d 92. Mannitol and inulin used for measurement of
c. Mother's serial antibodies Ans: ECF
Ans: A 93. Drug acting through H-k ATPase pump
82. Phrenic avulsed near origin. What will happen Ans: PPI
a. Decreased expiration 94. Foramen cecum is related to
b. Loss of respiratory stimulus a. thyroid
c. Loss of sensation in diaphragm in b. thymus
central tendon c. anterior pituitary
Ans: B > C (controversial), i am with option B Ans: A
Radiant Notes—FCPS PEARLS Golden File 4 By: Dr. Rafi Ullah
95. Patient sitting. Percussion from posterior side juvenile polyposis while adenomatous polyps are
along scapula. Upto which rib resonant true neoplasm)
a. 6 111.Carotid sinus
b. 8 a. Chemoreceptors
c. 10 b. Respond to changes in blood pressure
Ans: B Ans: B
96. Patient was given a drug which caused Inc Ca++, 112.After MI cyanosis?
Which drug was given a. Ventilation perfusion disturbance
Ans: Thiazide b. Slowing of circulation
97. Parasympathetic Ans: A
Ans: Decreases pupil size 113.Terminal cancer,, to reduce side effects
98. Epinephrine morphine is given along with
Ans: Causes glycogenolysis in liver Ans: Paracetamol (decreases morphine
99. Ant cardiac vein drains into consumption and decreases its adverse effects)
Ans: Rt. atrium 114.Which pain killer should be given to pt after
100.Farmer had HCC. Cause appendectomy
Ans: Aflatoxin a. Morphine
101.Collagen arranged in which layer b. Paracetamol
a. Reticular layer c. Ketorolac
b. Dermis Ans: C
Ans: A 115.Appendix is supplied by appendicular artery
102.Regarding hyaline cartilage which is branch of?
a. Collagen and elastic fibers visible Ans: Terminal branch of ileocolic artery (The
b. Collagen not visible appendicular artery is a branch of the ileal or
c. None posterior cecal branch of the ileocolic artery)
Ans: B (Collagen fibers are present but not visible) 116.Which one is a histo pathological sign of
103.Lung cancer scenario, showing squamous cell metastasis?
Ca cause a. Invasion
a. Asbestosis b. Increase nucleus cytoplasm ratio
b. Benzidine c. Atypia
Ans: A Ans: A
104.Fio2 safe level which does not cause Retrolental MEDICINE AND ALLIED 2nd MARCH
fibroplasia
Ans: 0.65 2017 EVENING
105.Excessive hemolysis 1. Inferiorly of the floor of fourth ventricle have
Ans: Unconjugated hyperbilirubinemia medially which structure
a. Pons
106.Cancer prevention mechanism b. Hippocampus
Ans: Apoptosis c. Caudate nucleus
107.Councilman bodies d. Amygdala
Ans: Apoptosis e. insula
108.Nephrotic syndrome. Basement membrane Ans: B
damage will leads to? 2. Site of action of thiazide diuretics
Ans: hypoalbuminemia a. Thick ascending loop
b. Early DT
109.Facial nerve?
Ans: B
a. Supplies Stapedius
3. Most common anomaly of great vessels found
b. Give chorda tympani in stylomastoid
is?
foramen a. TOF
Ans: A b. VSD
110.Hamartomatous polyps? c. ASD
a. Adenomatous d. Aortic aneurysm
b. Familial e. PDA
Ans: B (Hamartomatous polyps are benign Ans: E
Associated with Peutz-Jeghers syndrome and 4. Following is Ketone bodies
a. Acetoacetate
Radiant Notes—FCPS PEARLS Golden File 4 By: Dr. Rafi Ullah
b. Acetyl co a a. Brain
Ans: A b. Kidney
5. Most common cause of UTI c. Lungs
a. Klebsiella d. Skin
b. Strep. Pneumoniae Ans: B
c. E.coli 13. Common effect of nitroglycerin, histamine
Ans: C isoprenaline, theophylline
6. A man had hyperextension injury at cervical a. Tachycardia
CORD leads to b. Throbbing headache
a. upper motor neuron lesion in upper limb c. Dec force of contraction in heart
b. LMN lesion in lower limb d. Postural hypotension
c. LMN lesion in upper limb Ans: A
d. loss of pain in lower limb 14. 20 year asthmatic finding
e. loss of temp in lower limb Ans: FEV-1<75
Ans: C 15. A person had traumatic incident of upper limb
7. 55 years old man non-smoker has slight cough follow 3 months he feels pain and tenderness, a
for a week on x-ray there is subpleural coin swelling develops at the region, on x-ray
lesion of 2 cm in right upper lobe. diagnosis swelling is bright, after some time it resolved.
a. Granuloma Mechanism?
b. Silicosis a. Metaplasia
c. Squamous cell Ca b. Dysplasia
d. Small cell Ca c. Hypertrophy
Ans: A d. Hyperplasia
8. Stress related glycogenolysis in liver by? e. Apoptosis
a. Glucagon Ans: A
b. Cortisol 16. A 54 year woman has diplopia and weakness. On
c. Nor-epinephrine CT there is swelling in ant mediastinum. which of
d. Epinephrine following confirm diagnosis
e. Insulin a. Anti-acetylcholine receptor antibody
Ans: D (Epinephrine and glucagon, both of which are b. autosomal recessive antibodies
increased during stress, promote hepatic Ans: A
glycogenolysis, while with cortisol promotes hepatic 17. Research is done on function of thalamus MRI
gluconeogenesis.) shows different sensory stimuli. Which of
9. Insensible water loss in MODERATE climate following will produce minimal neurological
a. less than 200 activity in thalamus?
b. 200-400 a. Visual
c. 400-600 b. Olfactory
d. 600-800 c. Auditory
Ans: B d. Touch
10. A new born baby with tetany Ca++ level given, e. Vibration
on examination there is murmur baby is Ans: B
suffering from infection by pneumocystis carini 18. Farmer’s lung is due to?
Aspergillus herpes simplex .diagnosis? a. Coal dust
a. Adenosine deaminase deficiency b. Cotton fiber dust
b. DiGeorge 22q deletion c. Grain dust
c. B-cell /plasma cell defect d. Asbestos
Ans: B Ans: C
11. A woman had MI 48 hour later she collapsed. On 19. Apex of lung
ECG there is Irregular low voltage waves, a. 2.5 cm above mid clavicular line
pathophysiology? b. 2.5 cm above sternoclavicular joint
a. Dec CO c. Not extend to neck
b. Dec EF Ans: B
c. Inc refractory period 20. Long scenario cytokeratin stain for?
d. Inc speed of conduction of impulse Ans: Carcinoma
e. Re-entry current 21. Anesthesia of upper limb by axillary route. there
Ans: E is intact sensation on lateral surface of forearm
12. Patient visited hospital and have been diagnosed which nerve spared
with amyloidosis? Which organ will be most Ans: Musculocutaneous nerve
affected by this condition? 22. Hyponatremia most commonly occur in?
Radiant Notes—FCPS PEARLS Golden File 4 By: Dr. Rafi Ullah
a. Excessive water intake 36. Light thrown in right eye direct reflex is present
b. Hyperosmotic fluid administration but absence of indirect light reflex. Which
c. Post op patient structure is involved?
d. Gastro enteritis with fluid intake a. Right occulomotor nerve
Ans: D b. Left occulomotor nerve
23. Effect of vagus on heart rate? c. Pretectum
a. Contractility of heart Ans: B
b. Na influx 37. Beta endorphin present in
c. k efflux a. Hypothalamus
Ans: C b. Medulla
24. Cephalosporin MOA c. Pituitary
a. Transpeptidase inhibition Ans: A
b. Dec synthesis of peptidoglycan 38. Patient comes with tender right hypochondrium
Ans: A stool with occult blood on colonoscopy
25. Pt has c/o joint pain n on x-ray large silhouette of ascending colon was involved showing ulcers
heart seen? In childhood she has recurrent while other parts of colon spared. Diagnosis?
attacks of pharyngitis of B-streptococcal group, a. Necrotizing vasculitis
which valves is most commonly involved b. Crypt abscess
a. Mitral c. Ent amoeba histolytica
b. Aortic d. Carcinoma
Ans: A Ans: C
26. Scenario of sickle cell disease 39. In laparotomy how will surgeon identify left
Ans: HbSS kidney
27. Opportunistic organism after splenectomy Ans: Anterior relation with stomach, pancreas,
Ans: Streptococcus Pneumonia descending colon, spleen, jejunum
28. Anterior cruciate ligament damage direction of 40. Child with history of bed wetting though toilet
tibia over femur? trained. Investigation done at one side
Ans: Anterior duplication of ureter was found and one of them
29. Narrowest part of pediatric airway was opening in vagina cause?
Ans: cricoid Ans: Early division of ureteric bud
30. Regarding vertebral column 41. Motor end plate contains
Ans: cervical vertebra are 7 a. Acetylcholine esterase
b. Acetylcholine
31. Nerve involved in Bell’s palsy?
Ans: A
Ans: 7th cranial nerve
42. Coal mining sand blaster
32. Dilatation of aortic arch will compress
a. Silicosis
a. Left bronchus
b. Asbestosis
b. Trachea
Ans: A
c. Oesophagus
d. Left recurrent laryngeal nerve 43. Anxiety is decreased by activation of which
Ans: A receptors?
a. GABA
33. Axillary artery formed by
b. Glutamate
a. basilic vein and vena comitantes of brachial
c. Dopamine
artery
d. Glucocorticoid
b. cephalic vein
e. Nicotinic cholinergic
Ans: A
Ans: A
34. Jejunum supplied by
44. A person had injury which did not heal after 1
a. SMA
month what is missing
b. IMA
a. Skin appendages
c. Pancraticoduodenal artery
b. Granulation tissue
Ans: A
c. Fibroblast
35. Regarding vessels of lower limb d. Underlying connective tissue
a. Femoral vein is lateral to femoral canal Ans: C
b. long saphenous ends in mid-thigh
45. Air borne transmission occurs in?
c. Short saphenous starts from medial side of
a. Diphtheria
foot
b. Anthrax
d. cruciate anastomosis is b/w femoral artery
c. T B
and external iliac artery
Ans: C
Ans: A
46. Lymph flow will be decreased by increase of
Radiant Notes—FCPS PEARLS Golden File 4 By: Dr. Rafi Ullah
a. Hemorrhage Ans: D
b. Increase arterial pressure 57. Heart sound difference between S1 and S2?
c. Venous pressure a. High duration
d. Increase capillary permeability b. High pitch
e. Tumor c. High intensity
Ans: C Ans: A (s1 has long duration as compared to s2)
47. Air droplet infection can spread upto distance of 58. Deep inspiration
a. 1-2 feet Ans: Inc pulmonary compliance
b. 3-6 feet 59. Rectal prolapse
Ans: A (less than 3 feet) Ans: Trichuris trichuri
48. Dopamine half-life 60. HR 75 beats/min PR is 0.3, if HR inc 225 then PR
a. 2 Mins is
b. 9 mins a. 0.1
Ans: A b. 0.9
49. What structure Is Present In Floor Of Inferior c. 0.05
Horn Of Lateral ventricle medially Ans: A
a. Insula 61. Right lung different from left lung
b. MCA a. 12 Broncho pulmonary segment
c. Caudate nucleus b. Impression of ventricles
d. Hippocampus c. Larger in size
Ans: D d. Lighter in weight
50. A person can't recognize right half of body Ans: C (The left lung is smaller because of the space
where is lesion? taken up by the heart)
a. Premotor area 62. Dorsal column damage which will least affected
b. Cerebral cortex a. Touch
c. Neo-cortex b. Proprioception
d. Somatosensory association Area c. Asterognosia
Ans: D d. Vibration
51. Characteristics role of prostaglandin E2 and D2 e. Increase threshold for sensory stimuli
a. Vasodilation Ans: A
b. Vasoconstriction 63. A pregnant lady presented with lower abdominal
c. Increase permeability pain, on ultrasound placenta attaches to
Ans: A myometrium but does not penetrate through it??
52. Normal platelet Function required for? a. Placenta accreta
a. Clot retraction b. Placenta increta
b. Clot organization c. Placenta percata
c. Clot formation d. Placenta previa
Ans: A Ans: A
53. Hering beurer inflation reflex inhibits? 64. Pinworm infection
a. Apneustic center a. Ingestion of egg form
b. Pneumatic center b. Ingestion of larva form
c. Ventral respiratory group Ans: A
d. Dorsal respiratory group 65. Atrial repolarization on ECG seen in
Ans: D a. 1sT Degree Heart block
54. Brain bridge reflex function b. 3rd degree heart block
Ans: Increase heart rate c. HTN
55. GFR Ans: B
a. Increase with moderate efferent arteriole 66. QRS complex occur in
constriction a. Ventricular systole
b. Increase with severe efferent arteriole b. Ventricular depolarization
constriction Ans: B
c. Hydrostatic pressure in bowman capsule is 67. In a state, where there is fall in pH of plasma ,
only factor which it depends on increase in Bicarbonate and increase in PCO2 ,
Ans: A there is
56. Surfactant a. Met Acidosis with Resp compensation
a. Produced by type 3 alveolar cell b. Met acidosis with no compensation
b. Dec action in DM c. Resp acidosis with Met compensation
c. Works better with lecithin in myrintase d. Resp acidosis with no compensation
d. Inc amount of glycerol phosphocholine
Radiant Notes—FCPS PEARLS Golden File 4 By: Dr. Rafi Ullah
e. Partially compensated respiratory acidosis hemorrhages hyperplastic arteriosclerosis and
Ans: E (↓ Ph. ↑Co2= respiratory acidosis, with fibrinoid necrosis diagnosis?
compensation from renal by increasing bicarbonate, a. Fibromuscular dysplasia
but as it fail to bring pH. towards normal so partially b. DM
compensated respiratory acidosis) c. Systemic sclerosis
68. Slowest growing malignant thyroid tumor Ans: A
a. Papillary 79. Old male patient has history of headache in
b. Follicular temporal region there is also thick cord like
Ans: A vessel in that area. On biopsy giant cells are
69. Steady pressure is detected by seen?
a. Meissner a. ESR more than 110
b. Ruffini b. Steroid not effective
c. Pacinian c. Diagnosis made on Biopsy
Ans: B Ans: C
70. ST elevation in Leads II, III, AVF shows 80. Saliva prevents iron utilization by microbes via?
a. Anterior wall MI a. Lysozymes
b. Inferior wall MI b. Lactoferrin
Ans: B Ans: B
71. Pt with AML undergo bone marrow transplant 81. Which corynebacterium is common amongst
presented with pneumonia histology shows humans
large cell with intracellular inclusion, cause is? Ans: Diphtheria
a. CMV 82. Vagotomy done
b. Candida a. Decreased gastric acid and pepsin secretion
c. Pneumocystis b. Inc gastric acid and pepsin secretion
Ans: A Ans: A
72. Vitelline duct remnant? 83. Which is more in dialyzing fluid as compare to
Ans: Meckel diverticulum plasma
73. Which of following act through C-AMP a. Glucose
mechanism? b. HCO3-
a. BNP Ans: A
b. ANP 84. Which inhibits stomach emptying
c. ADH on DCT a. CCK
d. ADH on blood vessel b. Gastrin
Ans: C Ans: A
74. Spermatogenesis 85. Decreased gastric secretions by
a. Primary oocyte form secondary oocyte a. Secretin
b. Secondary oocytes form spermatids b. Gastrin
c. Spermatids form spermatozoa Ans: A
Ans: C 86. Rheumatoid arthritis
75. Hormone responsible for ductal growth and fat Ans: HLA DR4
deposition in breast 87. Ankylosing spondylitis
a. Progesterone Ans: HLA B 27
b. Estrogen 88. Antibody receptors are
c. Prolactin Ans: Hyper variable regions of H and L chains
Ans: B 89. Alpha adrenergic causes
76. Erythropoietin secretion is inhibited by a. Mydriasis
a. Cobalt b. Bronchodilation
b. Hypoxia Ans: A
c. Theophylline 90. Eye infection diagnosed as HSV conjunctivitis
Ans: C treatment
77. Which toxin responsible for scarlet fever a. Trifluridine
a. Exotoxin b. Acyclovir
b. Endotoxin Ans: A
c. Erythrogenic 91. 5 year old child was diagnosed with polyp and
Ans: C he was having rectal bleeding. Polyp was
78. Patient with malaise and raised blood pressure, removed from rectum by polypectomy what type
died b/c of hemorrhagic stroke in basal ganglia of polyp it could be?
on autopsy bilateral small kidneys with petechial a. Hypertrophic
Radiant Notes—FCPS PEARLS Golden File 4 By: Dr. Rafi Ullah
b. Hyperplastic a. Ruffini
c. Inflammatory b. Golgi tendon
d. Hemorrhagic c. Krause receptor
e. Polyp adenoma d. Merkel disc
Ans: C Ans: C (no option of pacinian, Remember highly
92. Patient with diagnosis to have stroke... on CT asked rapidly adapting receptors are Pacinian and
there was pale infarct... What could be cause? meiseners, while Golgi, merkels and Ruffini are slow
a. Hypertensive stroke adapting receptors)
b. Emboli / ischemic stroke 102. Thromboxane A2 produced by?
c. Rupture of berry aneurysm a. Platelets
d. Atherosclerosis b. Basophils
e. Rupture of arteriovenous fistula c. Histamine
Ans: B d. Mast cell
93. Hb 2nd trimester form from e. Endothelial cell
a. Spleen Ans: A
b. Liver 103. In cardiac achalasia what found?
Ans: B a. Abnormal pharyngeal phase of deglutination
94. Bundle of Hiss supplied by b. Absence of ganglion cells
Ans: RCA c. Myenteric plexuses with absence fibers of
95. Right hemoglobin dissociation curve occur by vagus nerve
Dec in Ans: C
a. Temp 104. About topically applied drugs what best
b. 2-3 BPG describes it?
c. PH. a. Cornea is hydrophilic
d. CO2 b. Cornea is hydrophobic
Ans: C c. Drug must be water and lipid soluble
96. Hanging death due to d. Absorption surface of cornea is 2X
a. Odontoid process Ans: C
b. Pedicles of c2 vertebra 105. Pneumatic bones are found in?
Ans: A a. Face
97. Stimulation of thirst center by b. Upper limb
a. Inc ADH c. Vertebral disc
b. Alcohol d. Lower limb
c. Inc aldosterone Ans A
d. Injecting hypertonic solution on 106. Ca channel blocker act on SA node
Osmoreceptors a. Verapamil
e. Angiotensin 2 b. Amlodipine
Ans: E c. nimodopine
98. Parasitic infection Ans: A
Ans: Eosinophilia 107. In hypovolemic shock thirst is increased by?
99. Man waiting for 60 minutes in moderate hot a. Angiotensin 1
environment... How body will regulate b. Angiotensin 2
temperature? c. Renin
a. Shivering d. Aldosterone
b. Erection of erector pili e. ADH
c. Overactive posterior hypothalamus Ans: B
d. Cutaneous Hyperemia 108. 20 year old asthmatic patient finding on
e. Vasoconstriction spirometry
Ans: D Ans: VT 450 FEV 2000 FVC 3000 (no recall of other
100. Young married woman delivered a healthy options in previous pprz there were options and best
baby boy... Now by which mechanism size of among those were VT 300 FEV-1: 2000. FVC: 3200)
uterus will be reduced or become normal 109. Graded potential has these features
a. Phagocytosis a. Is decremental
b. Proteolysis b. Is non decremental
c. Necrosis c. Saltatory conduction
d. Inflammation d. Follow all on none
e. Apoptosis Ans: A
Ans: E 110. Immunoglobulin produce at (it was not by)
101. Which of following is rapidly adapting? a. Plasma cell
Radiant Notes—FCPS PEARLS Golden File 4 By: Dr. Rafi Ullah
b. Spleen Ans: Unorganized tissue
c. Bone marrow 125. Antigen presenting cells and high
d. Thymus Phagocytic?
Ans: A a. Monocyte
111. GFR b. Macrophage
a. Inc by hydrostatic pressure in Bowman c. Eosinophils
capsule d. Neutrophil
b. Inc by afferent arteriole constriction Ans: B
c. Inc by efferent arteriole constriction 126. Ranitidine
Ans: C Ans: less CNS side effect than cimetidine
112. Urine hyperosmotic, plasma Na+ value Dec 127. Long thoracic nerve supply
cause is Ans: Serratus anterior
Ans: Inc ADH 128. Inc vascularity in
113. Renal disease Crescents are formed due a. Sarcoma
to?? b. Carcinoma
a. mesangial Ans: A
b. mesangial+ endothelial 129. Metastasis
c. endothelial Ans: Loss of E-Cadherin
Ans: B 130. During deep inspiration
114. On histo slide there is number of striated Ans: Inc pulmonary compliance
Ducts mostly serous acini were seen??? 131. Hemolytic anemia
a. Parotid gland Ans: Un-conjugated hyperbilirubinemia
b. Submandibular
132. Partial thyroidectomy done for?
c. Sub lingual
Ans: To save parathyroid gland
Ans: A
133. First branch of abdominal aorta?
115. Left geniculate body lesion?
Ans: Inferior phrenic
a. Left homonymous hemianopia
b. Right homonymous hemianopia 134. FFP given for?
c. Bitemporal Ans: Warfarin toxicity
d. Binasal 135. Internal carotid lies
Ans: B Ans: first lateral to external then post and then medial
116. Submandibular gland divided by which 136. Which of following is appropriate for fetal
muscle? skull?
Ans: Mylohyoid a. Ant fontanelle formed by 3 bones
117. Urinary bladder separated from rectum by b. Vault is larger than face
Ans: B
Ans: Denoviller’s fascia
118. Patient has fever from one week. On 137. True about bronchioles?
auscultation, lungs fields clear. There is a rustle a. Developed prenatally
murmur on the left of sternum. Diagnosis? b. Have no cartilage in walls
Ans: B
a. Pericarditis
b. Pleural effusion 138. Glucuronidation of drug causes?
Ans: A a. Activation of drug
119. HDL b. Lipid soluble
c. Water soluble and inc renal clearance
Ans: High protein
Ans: C
120. Preload indicator
139. EBV causes
Ans: LV EDV
Ans: Burkitt lymphoma
121. Pacinian
140. Selective Cox 2
Ans: Vibration and Pressure
Ans: Meloxicam
122. Merkel diverticulum
141. Morphine
Ans: Remnant of Vitellointestinal Duct
Ans: In terminal cancer patient
123. Primary active transport characteristic
finding 142. GI bleed DOC?
a. Inj Octreotide
a. Uphill.
b. Tranexamic acid
b. Cleavage of ATP for energy
Ans: A
c. Carrier protein
Ans: B (if only active transport mentioned then click 143. Inverse square root of variance is?
uphill) Ans: Standard deviation
124. Hamartoma 144. Dr should start conversation with patient
Radiant Notes—FCPS PEARLS Golden File 4 By: Dr. Rafi Ullah
Ans: Asking name age b. Inc hydrostatic pressure
145. Superior colliculus? c. Dec oncotic pressure
a. Vision Ans: B
b. Auditory 163. Fibroblasts
Ans: A Ans: Amorphous extracellular matrix
146. Anterior Choroidal artery branch of 164. Fastest fibers
Ans: Internal carotid Ans: Purkinje fibers
147. Which pass through cavernous sinus 165. Cause of atheroma?
Ans: Abducent nerve a. Diastolic HTN
148. Mass moves with swallowing due to b. Systolic HTN
Ans: Pretracheal fascia c. Alcohol
149. Leukocyte adhesion molecule d. Cholesteatoma
Ans: LFA-1 Ans: A (focus on the option D, Cholesteatoma is an
150. Lymphatic related ear diseases, but if in options somehow
Ans: Cystic Hygroma cholestermia is present click that)
151. Sympathetic muscarinic? 166. Sympathetic stimulation causes
a. Sweat gland Ans: Inc blood flow and Inc GFR
b. Adrenal medulla 167. Vitamin D3?
Ans: A a. Animal vitamin
152. Taste sensation b. 1,25 Cholecalciferol
Ans: Tractus Solitarius c. Plant vitamin
d. 25.hydroxy Cholecalciferol
153. Philadelphia chromosome
Ans: B
Ans: CML
168. Function of Vit D
154. Scenario of SLE
Ans: Inc renal Ca reabsorption
Ans: Anti-Ds DNA (most specific)
169. C wave occur during?
155. Rete ridges?
a. Opening of semilunar valve
a. Verrocous
b. 1st heart sound
b. Squamous cell CA
c. RV contraction
Ans: A (Ref: Philippine journal of pathology Dr.
Ans: C
Manas Bajpai)
170. Adrenergic stimulation causes
156. Inverse reflex is?
Ans: Gluconeogenesis and glycogenolysis
a. Is polysynaptic
b. Is Disynaptic involving 1 neuron 171. Tumor immunity
c. Monosynaptic Ans: NK cell
Ans: B 172. Dexamethasone as compare to
157. Earthquake is hydrocortisone?
a. Social crisis a. Immunosuppression
b. Situational crisis b. Hyperglycemia
c. Emotional crisis Ans: A (dexamethasone is along acting, causes
Ans: B immunosuppression much more than
hydrocortisone, although it does causes
158. Left deviation?
hyperglycemia but immunosuppression effect is
a. RBBB
more)
b. Right sided MI
Ans: B 173. Central chemoreceptors respond to?
a. CSF PH
159. Hyperkalemia cause
b. PCO2 of cerebral vein
Ans: Inc aldosterone
c. Dec pO2 in arterial blood
160. Thiazide causes d. Inc Co2 in arterial blood
Ans: Hypokalemia Ans: A (ref BRS)
161. Patient present with dizziness vertigo 174. Case of Rheumatic heart disease most
polydipsia polyuria polyphagia where is lesion common cause
a. Pituitary gland Ans: Streptococci viridian
b. Hypothalamus
175. Stratified columnar epithelium found in?
c. Alpha cell of pancreas
a. Esophagus
d. Beta cell of pancreas
b. Stomach
Ans: D
Ans: A
162. Cause of edema in CCF?
176. Histamine releasing opioid?
a. Inc permeability
Ans: Morphine
Radiant Notes—FCPS PEARLS Golden File 4 By: Dr. Rafi Ullah
177. Endotoxin 5. A girl after fight with her best friend, was unable
a. Release after bacteria death to extend her all fingers. She also noted that
b. Strongly antigenic she can’t abduct her thumb too, nerve injured is
Ans: A a. Ulnar nerve injury
178. Injury to lateral hypothalamus causes b. Median nerve injury
a. Increases appetite c. Radial nerve injury
b. Dec hunger Ans: C
c. No effect on hunger 6. A girl is unable to comb her hairs, Nerve injured
Ans: B a. Accessory nerve
179. Radiation cause malignancy b. Cranial part of 11 nerve
Ans: 5-10 years c. Spinal part of accessory nerve
180. Eclamptic patient on OT table drug of choice Ans: C
for HTN? 7. A surgeon was taking lymph node biopsy from
a. Sodium nitroprusside posterior triangle, he was skilful but that
b. IV hydralazine morning he fought with her wife. Which nerve
c. Methyldopa can be damaged by his lack of attention during
Ans: B surgery
181. 4 year old girl complains that she is a. hypoglossal
abnormally tall and also secretion from her b. accessory nerve
mammary glands on x-ray growth of pituitary c. spinal part of accessory
sella turcica which cell increase d. Ansa cervicalis
a. Chromophobes Ans: C
b. Eosinophil 8. Related to inter coastal veins and arteries
c. basophils a. branches of thoracic aorta
Ans: B b. Branches of internal thoracic artery
c. 1st 5 are branches of Costocervical trunk
Medicine and Allied 3rd March 2017 d. TWO anterior veins and one posterior vein.
Morning Ans: D
1. The respiratory zone where gaseous exchange 9. A branch of maxillary artery
occurs extends beyond Ans: Inferior orbital artery
a. Respiratory bronchioles 10. Extensive eversion can damage to
b. Terminal bronchioles a. Lateral ligament of foot
c. Alveolar ducts b. Triangular ligament
d. Alveoli c. Deltoid ligament
Ans: B Ans: C
2. During strenuous exercise, the respiratory rate 11. Pons k connections
increases, this is because a. Contralateral cerebral cortex
a. Pco2increases b. Ipsilateral cerebellum
b. Po2 decreases Ans: A (Not sure, if have to choose between these
c. PH. of blood increases two i prefer A)
d. Hypoglycemia occurs 12. Left sided facial palsy plus UMN lesions sign on
Ans: A (Exercising muscle  ↑ CO2 production and same left side
↑ O2 consumption. ↑Ventilation rate to match O2 a. Cerebral lesions
consumption and CO2 production) b. Pons lesions
3. Baroreceptors detects Ans: A
a. Decreased blood pressure 13. Lateral ventricle relations
b. Increase blood pressure Ans: Hippocampus
c. Change in arterial blood pressure
14. Hypogonadism and anosmia lesions
d. Change in venous blood pressure
a. Hypothalamus (kallmann)
Ans: C (Aortic Arch baroreceptors Respond to
b. Amygdala
increases, but not to decreases, in arterial pressure
c. Hippocampus
while Carotid Sinus baroreceptors Responds to
Ans: A
both, increase as well as to decrease in arterial
pressure 15. Contralateral loss of pain and temp at T9 and
ipsilateral loss of proprioception below T8,
4. Pulse wave velocity can be used to
bilateral normal reflexes
a. Detect blood pressure
a. Lesion below T9
b. Detect blood flow to the extremity
b. Lesion of spinothalamic tract at T8
Ans: B
Ans: B
Radiant Notes—FCPS PEARLS Golden File 4 By: Dr. Rafi Ullah
16. Two kids bet on eye blinking, the one kid lost as Ans: C
he blinked 1st, this blinking was due to 27. Drug of choice for absence seizure
a. Thoracic segment of spinal cord a. Valproic acid
b. Cervical segment of spinal cord (superior b. Hyperventilation
cervical ganglion) c. Ethosuximide
c. Midbrain Ans.C
Ans: B 28. Regarding reserpine
17. Sensory inputs from face is relayed in the a. Absorption decreased by food
a. Main sensory nucleus of trigeminal nerve b. has active metabolites
b. Mesencephalic Ans: A (Not sure, damn CPSP questions)
Ans: A 29. Side effect of xylocaine
18. CSF is absorbed by a. Syncope
a. Ependymal cells b. Tachycardia
b. 3rd ventricles c. Hypotension
c. Arachnoid granulations d. Rashes
Ans: C Ans: A
19. Epidural anesthesia is injected in 30. Drug receptor interaction
a. Epidural space a. Heparin and protamine sulphate
b. Subdural space b. Mannitol as a diuretic
c. Subarchanoid space c. Beta blocker
Ans: A Ans: A (heparin antidote is protamine sulphate)
20. Neuronal AP is due to 31. Traveler’s diarrhea resistant to metronidazole
a. Rapid Ca influx a. Cephalosporin
b. Influx of K b. Ofloxacin
c. Leakage of K c. Norfloxacin
d. Influx of Na Ans: C (both b and C correct)
Ans: D 32. Long scenario patient with endocarditis and
21. Regarding autonomic nervous system penicilin allergic. Next drug
a. All post ganglionic sympathetic are nor- a. Clindamycin (opposition favorite)
epinephrine secreting b. Vancomycin and gentamycin
b. All parasympathetic are epinephrine Ans: B
dependent 33. Variance
c. All parasympathetic secrets ACh secreting a. Shows deviation and variability in between
Ans: C individual measures
22. Melanocytes are b. Shows variation from the mean of sample
a. Ectodermal Ans: B
b. Mesodermal 34. Primary prevention for hyperlipidemia
c. Neural crest cells derivative a. Avoid oily foods
Ans: C b. Statin administration
23. Diaphragm is derivative of Ans: A
a. Neural crest cell 35. The unwanted effects of hyperbaric oxygen
b. Cervical somites administration can be avoided by administering
c. Thoracic somite’s derivative a. Vit-D
Ans: C b. Vit-A
24. Regarding C7 vertebra c. Vit- E
a. Heavy body Ans: C
b. Kidney shaped 36. To measure cardiac output, which dye is used?
c. Longest spine a. Cardio blue
Ans.C b. Apple green
25. Clavulinic acid helps penicilins in c. Evans blue
a. Cell wall inhibition d. Cardiogreen
b. Keep busy bacteria Ans: D (both Evans blue and cardiogreen are
c. Beta lactamase inhibitor used, cardiogreen > Evans blue)
Ans: C 37. Proteins normally don’t leaks out in tubular
26. Mechanism of action of cephalosporin fluid due to endothelium is fenestrated
a. Cell wall inhibitor a. Basement membrane is thick
b. Beta lactamase inhibitor b. Basement membrane is negatively charged
c. Peptidoglycan inhibitor Ans.B
d. Folate inhibitor
Radiant Notes—FCPS PEARLS Golden File 4 By: Dr. Rafi Ullah
38. Long scenario on minimal change disease what Ans: Cataract
u can see on microscopy? 53. Parasite
a. Endothelium damaged a. Clostridium
b. Tram trick appearance b. Confusing names
c. Effacement of foot process c. Toxoplasma
Ans.C Ans: C
39. Typical difference btw both kidneys 54. Rubella infection to mother and u want to
Ans: Hilum decide the abortion, what will support Ur
40. Plane section passing through pylorus plane decision
Ans: Body of gall bladder can be seen a. IgM
41. Adductor canal b. IgG
a. Medially by sartorius Ans: A
b. Present in hamstring 55. Difference between osmolarity of intracellular
Ans: B and extracellular
42. Action of CCK Ans: 1 mOsm/l
a. Increased stomach motility 56. Kcal required daily
b. Inhibit gall bladder Ans: 25-30
c. Increase Intestinal motility 57. Oblique fissure
Ans.C a. t3-t6
43. Intrinsic waves of GIT b. t2-t6
Ans: 12 for duodenum Ans: A
44. The wave generated in esophagus to propel the 58. Long scenario of Down syndrome, and u took
bolus is villous sample, what do u want to see
a. Mass movement a. Cytogenetic abnormality
b. Primary peristalsis b. Sex of the baby
Ans: B Ans: A
45. GIT motility 59. High output cardiac failure
a. Same as of stomach Ans: Thiamine deficiency
b. Is not dependent on hormones 60. Chronic alcoholism, sign and symptoms
c. Doesn’t occurs in absence of food Ans: Thiamine
Ans: correct option missing poor recall 61. B12 deficiency signs symptoms plus abdominal
46. Glucose absorption in GIT is symptoms, what will u check?
a. Primary active a. IF level
b. Sec Active b. B12 level
Ans: B Ans: B
47. Most potent response in hypovolemic shock 62. With which type of blood, transfusion reactions
a. Sympathetic will be most severe
b. Brain ischemic a. A+ To 0-
Ans.B b. O+ To B+
48. Asbestosis c. A- To A+
Ans: Mesothelioma Ans: A (as a general rule u cannot give positive to
49. Benzidine negative, and one more thing O -ive is universal
a. Carcinogenic donor but can only receive from O negative)
b. Lithium 63. Graft vs. host rejection, what will be present
c. Alcohol Ans: Hyperheamogluninemia
Ans: A (causes e.g. bladder carcinoma) 64. T-cells
50. Long scenario, but PCo2 will be decreased to a. Mature in bone
compensate for b. No rule in apoptosis
a. Respiratory acidosis c. Responsible to kill cancer cells
b. Metabolic acidosis Ans: C
c. Metabolic alkalosis 65. T-cells
Ans: B a. Component of innate immunity
51. The pulmonary artery pressure will be high then b. Need co-stimulatory signals for activation
aortic pressure in Ans: B
a. Right to left shunt 66. Related to AIDS
b. Left to right shunt Ans: Kaposi sarcoma
Ans.B 67. AIDS defining cancer
52. Rubella a. Esophageal carcinoma
Radiant Notes—FCPS PEARLS Golden File 4 By: Dr. Rafi Ullah
b. Malignant melanoma 79. Regarding O2 carrying capacity of Hb
c. Kaposi sarcoma a. Carries 134 ml of o2 per mg
Ans: C b. 1.34ml per gm of Hb
68. Nasopharyngeal carcinoma Ans: B
a. Herpes simplex 80. The plasma oncotic pressure is mainly due to
b. Radiation a. Albumin
c. EBV b. Electrolytes
Ans: C c. Clotting factors
69. Microscopic findings were epithelium and rete Ans: A
ridges 81. Albumin to globulin ratio
a. verrocous carcinoma Ans: 0.8-2.0
b. Squamous cell carcinoma 82. A long scenario on SLE and end Mein what test
Ans: A u will order for screening?
70. Long scenario, Estradiol present but no a. ANA
progesterone for last 3 days in a normal adult b. DsDNA
female married. Ans: A
a. Pregnancy 83. Ventilation related hospital infection
b. Ovulation a. Pseudomonas
c. Menstruation b. Staph. Aureus
Ans: C Ans: A
71. Cause of decrease cardiac output 84. Meningitis, a child of unknown age
a. Thiamine deficiency a. Bacteroids
b. Hypervolemia b. Neisseria meningitis
c. IVC obstruction c. Streptococcus
Ans: C Ans: C
72. The percentage of blood pumped out by heart 85. Farmer having a lesion (ulcer) on legs
out of total blood in the end of diastole a. Cutaneous larvae migrans
a. Ejection fraction b. Leishmaniasis
b. Cardiac output Ans: A
Ans: A 86. Long scenario of a school teacher who after
73. Long scenario, a boy in OT room waiting for his death was diagnosed as a case of endocarditis
turn to be under the blade of a well-known and lymphocytic infiltration of cardiac tissues
surgeon, sweating, tachycardia, B.p a. Viridians
120/80…pulse 90 b. Coxsackie virus
a. Pheochromocytoma c. CMV
b. Amphetamine Ans: C
c. Stress mental 87. Long scenario of infective endocarditis. Which
Ans: C will be more diagnostic
74. Hypovolemic shock, which will happens 1st? a. Gram staining
a. Arteriolar constriction b. Echocardiogram
b. Renal absorption increases c. Blood culture
c. Venous and venules constrict Ans: C
Ans: C 88. Long scenario on rheumatic heart disease with
75. Which type of collagen is present during wound a mid-diastolic murmur in history before attack.
healing? What factor do u think predisposed her to
a. Type 1 rheumatic heart disease?
b. Type 3 Ans: Infective endocarditis
Ans: B (type 3 in early wound healing, while type 1 in 89. Clostridium tetani
late wound healing) a. Gram negative
76. ICAM, VCAM b. Spore forming
a. Platelets adhesions Ans: B
b. Leukocytes adhesions 90. Cyanosis
Ans: B Ans: Hb less than 5mg/dl
77. Most important function of endothelium in 91. Upper motor neuron signs and symptoms
hemostasis Ans: Pyramidal tract lesions
Ans: Prostacyclin production 92. Sickle cell disease
78. Warfarin overdose, 1st to give a. Point mutation
Ans: FFP b. Mismatch repair failure
Radiant Notes—FCPS PEARLS Golden File 4 By: Dr. Rafi Ullah
c. Missense mutation b. Dec estrogen
Ans: A Ans: A
93. Scenario on gall stones, regarding above 106. Hematocrit contain
scenario a. RBC only
a. Jaundice will not occur b. RBC ,platelets, WBC
b. Unconjugated bilirubin high in stool Ans: B
c. Fats content of stool high 107. Which drug can prolong the action of
Ans.C warfarin
94. Typical location of SA node a. Rifampicin
a. Inferior part of crista terminalis b. Cimetidine
b. Superior part of crista terminalis Ans: B
Ans: B 108. Neuromuscular junction
95. Increase bleeding time, all rest bleeding profile a. Clostridium tetani can act
normal b. Clostridium botulinum inhibits release of Ach
Ans: Thrombocytopenia Ans: B
96. Urine pH >5, despite low plasma NH3, type of 109. Atrial repolarization can be seen on ECG
acidosis during
a. RTA-2 a. Partial heart block
b. RTA-1 b. During sleep
Ans: B c. Complete heart block
97. Basilar artery infarction Ans: C
Ans: loss of vision… 110. QRS complex
98. Micturition reflex a. Ventricles depolarized
Ans: Inhibited during ejaculation b. Ventricular depolarization
99. TB granuloma Ans: B
a. Caseous necrosis 111. Maximum pressure in aorta during
b. Epitheloid cells a. Ventricle systole
c. Giant cells b. At the end of diastole
Ans: A c. Slow ejection phase
100. Regarding atypical mycobacteria Ans: C
a. They can be cultured on Loffler media 112. DiffusionofCo2acrosspulmonaryalveolidepen
b. Resistant to anti-Tuberculous typical drugs dson
Ans: B a. Charge on Gas molecules
101. Insulin regarding b. Humidityofco2
a. Uptake of glucose by liver and muscles c. Surface area available
b. Increase during exercise Ans: C
c. Inhibits gluconeogenesis 113. Progressive lengthening of PR interval with a
Ans: C drop of QRS
102. Scenario of Diabetic patient was obese and a a. Mobitz type 2
drug was give that causes diarrhea b. Mobitz type 1
a. Pioglitazone Ans: B
b. Stiglaptin 114. Most common infarct
c. Acarbose Ans: LAD
d. Metformin 115. Sternal angle
Ans: D a. Xyphoid is at the lower end
103. Scenario of a young diabetic, which cells b. 2 coastal cartilage
have lost their function Ans: B
a. Alpha cells 116. Which structure lies posterior to aorta
b. Receptors are lost a. Azygous vein
c. Beta cells b. Subclavian artery
Ans.C c. Brachiocephalic trunk
104. Cancer chemotherapy causes decrease in Ans: C
size of a tumor was a long scenario, mechanism 117. Receptors on hairless skin
a. Necrosis a. Meissener’s
b. Apoptosis b. Merkels
Ans.B Ans: A
105. Breast atrophy in young female is due to 118. Receptors for a tunic fork of……frequency
a. Dec Estrogen and Progesterone a. Merkels
Radiant Notes—FCPS PEARLS Golden File 4 By: Dr. Rafi Ullah
b. Pacinian c. Lymph node
Ans: B Ans: C
119. Receptors for cold 132. Damage to lateral hypothalamus
a. C-fibers a. Loss of anorexia
b. A-delta b. Loss of appetite
Ans: B (both A and C) Ans: B
120. A long scenario of cretinism in a baby what 133. Sole supply of mediastinal pleura
will be the condition of thyroid a. Phrenic nerve
a. Atrophy b. Intercostal nerves
b. Hyperplasia c. Pericardiophrenic nerve
Ans: A Ans: A
121. Long scenario of Cushing and symptoms, in 134. During rapid ascent from a deep water, the
the end there was adrenal mass light headedness and vomiting is due to
a. Cushing disease a. Vestibular irritation
b. Cushing syndrome b. Unequal pressure between two ears
Ans: A Ans: B
122. An enzyme that takes a radicle from one 135. Example of carrier mediated counter
molecule and attach to another transports?
Ans: Transferase a. Na+-glucose transport
123. Strongest stimulus for aldosterone b. Active transport
a. Na c. Passive transport
b. Hyperkalemia d. H+ transport
Ans: B Ans: A
124. Hyperkalemia’s induced by 136. Maximum Hb concentration
a. Administration of insulin a. Proeryhtroblast
b. Administration of aldosterone b. Normoblast
c. Exercise c. Reticulocyte
Ans: C d. Late normoblast
125. First mechanism against cold Ans: D
a. Vasoconstriction 137. Regarding Hodgkin lymphoma
b. Sweating a. B-symptoms are present in mental zone
c. Shivering b. It is associated with CMV infections
Ans: C c. Contagious spread
126. A scenario and CSF showed blood Ans: C
a. Subdural hemorrhage 138. Transmission of AIDS
b. Intradural hemorrhage a. Nasopharyngeal secretions
c. Subarachnoid hemorrhage b. Mother to fetus
Ans: C c. Oro-fecal
127. S3 heart sound Ans: B
a. Hypertrophic ventricles 139. Typical Achondroplasia scenario and asked
b. Hypervolemia what is the mood of inheritance
Ans: A Ans: Autosomal dominant
128. Serological diagnosis 140. Dryness of nose, eyes
a. Hydatid cyst a. Otic ganglion
b. Amoebic liver abscess b. Sympathetic chain
Ans: B c. Pterygopalatine ganglion
129. During cell division, chromosome arranges Ans: C
in equator during 141. A small child of 3 yrs. death due to unknown
a. Anaphase cause, coronary artery dilatation and rupture
b. Metaphase a. Takayasu arthritis
Ans: B b. Kawasaki disease
130. Aortic insufficiency Ans: B
a. Rheumatic fever 142. During bronchoscopy which lobe can be
b. Ankylosing spondylitis visualized 1st?
Ans: B a. Inferior lobe
131. Germinal centers, lobules and histology b. Apical superior
a. Thymus Ans: A
b. Tonsils 143. Regarding compact bone
Radiant Notes—FCPS PEARLS Golden File 4 By: Dr. Rafi Ullah
a. Haversian canal is oblique channel c. Dal-mash
b. Osteoblast are present in lacuna Ans: A (1 gm of fat gives two times the energy of
c. Lamella are arranged in regular fashion proteins or carbohydrates)
Ans: C 158. Steroids use…long scenario
144. COPD is diagnosed when FEV-1 a. Lymphocytosis
a. Less than 90 b. Erythroblastosis
b. More than 80 c. EOsinophilia
c. Less than 0.8 d. Neutrophilia
Ans: C Ans: D
145. Dialysis associated amyloid 159. Conversion of 25-OH-D3 to 1, 25-OH-D3
Ans: Beta-2-amyloid a. Lungs
146. Pigment layer of retina b. Blood
a. Store solar energy and convert it to heat c. Kidneys
b. Takes neurotransmitter from blood and stores Ans: C
Ans: B 160. Long scenario on a boy who moves his arm
147. HLA sampling, best is suddenly on a wide angle (hemiballismus)
a. Buccal mucosa a. Negri bodies
b. RBC b. Synuclein
c. WBC’s c. Subthalamic defect
Ans: C Ans: C
148. When antibiotics should be given to a patient 161. Epithelium changes shape with the activity
undergoing surgery of gland
Ans: OT-Table Ans: Thyroid follicles
149. Hypovolemia, blood loss to replace the fluid 162. Scenario on meningitis with just high
of intravascular compartment protein, low glucose
a. N/saline Ans: Bacterial meningitis
b. 5%dextrose 163. Rupture of membranous urethra fluid will
Ans: B accumulate in
150. Ascending reticular formation located in a. Superficial Perineal pouch
Ans: floor of aqueduct of Sylvius b. Deep Perineal pouch
151. During memory Ans: B
Ans: Structural changes in synopsis 164. Absence and presence of vomiting data type
152. Liver damage and then regeneration by is
a. Fibroblast Ans: Nominal
b. Growth factors 165. Repeated abortion, autoimmune signs
c. Hepatocytes a. SLE
Ans: C b. Antiphospholipid syndrome
153. Endocarditis typical tooth extraction history Ans: B
Ans: Strep viridans 166. Effect of parasympathetic on Heart
154. Hashimoto thyroiditis Ans: Prolong PR interval
a. Antithyroglobulin antibodies 167. Vit-D acts on
b. Antimitochondrial antibodies a. Proteins
c. Anticentromere antibodies b. Genes
Ans: A Ans: B
155. Least likely feature of carcinoma 168. Effect of breast feeding
a. Invasion Ans: Uterine contraction
b. Metastasis 169. Half-life of dopamine
c. Cell polarity a. 9 mints
Ans: C b. 2 mins
156. Hallmark of carcinoma Ans: B
a. N/c ratio 170. Textile industry
b. Pleomorphism Ans: Byssinosis
c. Invasion 171. After 12 hours along with neutrophils what
d. Metastasis will be present
Ans: D a. Membrane attack complex
157. High energy. b. c3b
a. Fats c. c5a
b. Protein Ans: A
Radiant Notes—FCPS PEARLS Golden File 4 By: Dr. Rafi Ullah
172. Posterior perforation of duodenum artery b. Hypoglycemia
involved Ans: A
Ans: Gastroduodenal artery 8. A boy has pain on nose and area between the
173. Post streptococcal glomerulus-nephropathy mouth and periorbital and area below temporal
scenario region what is the nerve involved?
a. Subepithelial deposits a. Mandibular branch of trigeminal
b. Mesangial proliferation
b. maxillary division of trigeminal
Ans: A
Ans: B
174. Pivot joint
a. Wrist joint 9. Increased electrical activity seen in epilepsy is
b. Knee due to alterations with which transmitter
c. Atlantoaxial a. Gaba
Ans: C b. glycine
175. Most carcinoma have c. MAO
Ans: Cytokeratin Ans: A
176. Hypersexuality, hyperorality and 10. Dose of streptomycin in 45 years old with 45 kg
disinhibiting weight
a. Kallmann syndrome a. 75g/kg
b. Klüver-Bucy syndrome b. 3gm/kg
Ans: B c. according to plasma level
d. 0.75-1gm intermittent
3rd March 2017 Medicine Evening Ans: D (normal dose is 20-40 mg/kg old age caution
1. How many somites are formed on 30th day? is taken when more than 59 years)
a. 30 to 32 11. Turner
b. 40 to 42 a. 45X0
c. 35 to 38 b. 46XY
Ans: C (range is 32-36) Ans: A
2. Vertical diameter of the thorax is increased by 12. What type of bones are ossicles?
a. diaphragm a. Flat
b. bucket handle pump b. Short
c. vertebra c. Sesamoid
d. ribs d. Irregular
Ans: A Ans: D
3. Salivation is increased by 13. A quack gave some drug causes elevated liver
a. food in mouth creates unconditioned reflex enzyme monitoring is by
b. smell of food a. AST
c. diminished by bilateral damage to 9th nerve b. ALT
Ans: B Ans: B
4. Pregnancy me rubella termination of preg 14. If one liter of 150mmol Na is infused. What will
Ans: IgM be result?
5. A non-pregnant patient has fasting glucose a. Inc ICF vol
122,after OGT one hour 198 and two hours b. Inc ICF Osm
194....what’s the diagnosis Ans: A
a. Something lag 15. Short bones are located in
b. Overt a. Ear
c. normal glucose tolerance b. Face
d. gestational Ans: A
e. secondary diabetes 16. Most commonly fractured bone in hand
Ans: B a. Scaphoid
6. Which Ig in pregnancy is indication of b. Lunate
termination in pregnancy Ans: A
a. IgM 17. Hip is related to
b. IgG a. sciatic nerve
Ans: A b. femoral nerve
7. Side effect of Biguanides? c. gluteus max inferiorly
a. Diarrhea
Radiant Notes—FCPS PEARLS Golden File 4 By: Dr. Rafi Ullah
d. obturator internus posteriorly c. Factor 8
e. obturator externus Inferiorly d. Factor 6
Ans: E Ans: A
18. IgA nephropathy 29. Which is not graphic presentation on ECG?
a. Inc mesangial matrix a. AV fiber
b. glomerular sclerosis b. Right atrium
c. thickening c. Left atrium
Ans: A d. SA node
19. Act through cGMP Ans: D
a. thyroxine 30. About adrenal gland
b. growth a. pyramidal in shape
c. Cortisol b. Perinephric fat something
d. ANP c. Cortex and medulla are derived from same
e. Oxytocin origin
Ans: D Ans: A (regarding option C the medulla originates from
20. Lithium therapy should be stopped neural crest, the adrenal cortex develops from the
Ans: Coarse tremor intermediate mesoderm.)
21. A female pregnant with mitral stenosis came to 31. Rt kidney different from left
you with atrial fibrillation. What will you do? Ans: Relation of structure at hilum
a. Observe 32. Lipid metabolism which Vit
b. Oral anticoagulant Ans: biotin
c. Iv heparin 33. Pretracheal fascia
d. Heparin and oral anticoagulant a. Thyroid
Ans: C b. Trachea
22. Pulmonary capillary wedge pressure is measure Ans: A
of.... 34. damage valve sequence of events
a. Indirect left atrial pressure Ans: damage valve- thrombosis --bacteremia –
b. Indirect Right atrial pressure perforation
Ans: A 35. Which is through Orofecal route
23. Abdominal aorta a. Hep-E
a. commences at L1 b. Filariasis
b. gives phrenic arteries at commencement c. Legionellosis
c. gives renal arteries bilateral d. Pertussis
Ans: C Ans: A
24. which of the following takes part in formation of 36. meningitis patient with increase lymphocytes,
Circle of Willis glucose 40mg
a. Middle cerebral and basilar artery a. viral
b. Anterior communicating and post b. Tuberculous
communicating artery Ans: B
Ans: B 37. Post splenectomy elderly male... cause of
25. A patient on ATT developed blurring of vision? infection
a. Rifampicin a. strep pneumonia
b. Ethambutol b. Neisseria
Ans: B c. H-influenza.
26. GFR increases by which Ans: A
a. increase pressure in bowmen capsule 38. A patient with hemophilia-A treatment
b. by increase in plasma osmotic pressure a. FFP
c. Inc afferent arteriolar resistance b. whole blood
Ans: B c. cryoprecipitate
27. Vein which directly drain in SVC Ans: C
Ans: azygous vein 39. Hepatic disease we measured
28. PT monitors? Ans: PT
a. Prothrombin and fibrinogen
b. Factor 9
Radiant Notes—FCPS PEARLS Golden File 4 By: Dr. Rafi Ullah
40. A woman with splenomegaly, conjugated 51. Which of following has more HCC chances in
bilirubin 1% with urinary bilirubin ...hepatic developing countries?
abnormality is due to a. Hep C
a. Hemochromatosis b. Hep B and Hep C
b. hemolysis Ans: B
c. benign cholestasis 52. An epileptic pt receiving carbamazepine is
Ans: depressed what should be kept in mind for anti-
41. About spinal accessory nerve... depressants
a. through substance of sternocleidomastoid a. Increased toxicity with fluoxetine
b. Ant to sternocleidomastoid b. MAO
c. Superficial to sternocleidomastoid Ans: B (Do not use carbamazepine if you have taken
d. Crosses transverse process of Atlas# an MAO inhibitor in the past 14 days. A dangerous
Ans: A drug interaction could occur)
42. For fetal heart maturation 53. Commonest CHD
a. growth hormone Ans: VSD
b. thyroid hormone 54. Compression causes Wasting of the thenar
Ans: B muscles, damage is to?
43. Growth hormone decrease in Ans: median nerve
a. starvation 55. Doctor patient consultation
b. sleep Ans: Active listening
c. Somatomedins 56. Greater and lesser sac are connected through
Ans: C Ans: epiploic foramen
44. Difference between cimetidine and ranitidine? 57. What structure you come around during normal
a. Cimetidine is less CNS toxic inguinal hernia?
b. Ranitidine can be given parenteral only a. Obturator nerve
c. Less efficacy of cimetidine b. Pampiniform plexus
Ans: C (remember ranitidine is less CNS toxic) c. Pudendal nerve
45. At 5th week of gestation what appears Ans: B
a. rotation of stomach 58. Serotonin and nor-epinephrine degraded by
b. limb buds Ans: MAO
Ans: A 59. A person has kidney transplant. 6months later
46. Regarding H2 blocker adverse effects? he developed fever allergy etc. What type of
a. More with short time high dose sensitivity is this?
b. More with long time low dose Ans: Type 1
Ans: B 60. Testicular tumor scenario what was the marker
47. Fetal movements perceived at a. CEA
a. 5th month b. HCG
b. 6th month c. AFP
Ans: A d. HDL
48. A boy with fever right knee pain which is hot Ans: C
tender limited movement. Organism involved? 61. Terminal ileum resection leads to def of
Ans: Staph aureus Ans: bile salts
49. A person with kidney problem taken long 62. A boy has developed night blindness. What’s the
treatment given no response on urine R/E WBC reason?
cells found. Ultrasound showed some renal a. Corneal keratinization
parenchymal disease. What’s the lesion likely b. Cataract
Ans: Chronic active c. Glaucoma
50. Common mode of transmission of Hep B and d. Vitreous hemorrhage
Hep C? Ans: A
a. Syringes 63. Sertoli cell function
b. Transfusion a. Blood Testis barrier
Ans: A (As now a days every blood is screened, and b. Testosterone
even if not needle stick injury via syringes and Ans: A
reusing same syringe is among the highest) 64. Right optic tract lesion
Radiant Notes—FCPS PEARLS Golden File 4 By: Dr. Rafi Ullah
Ans: Left Homonymous hemianopia a. Cervical rib
65. Memory loss area of brain effected b. 1st rib
Ans: hippocampus c. 2nd rib
66. Aphasia brain area associated Ans: A
Ans: temporal 79. A patient has given Anti globulin something,
67. pleural pain to shoulder hypersensitivity reaction
a. C2, C3, C4 a. Type 1
b. C3, C4, C5 b. Type 3
Ans: B c. Type 4
68. meningitis loss of hearing d. Type 2
a. neural pathway Ans: B
b. damage to organ of corti 80. In CHF. Captopril works as?
Ans: B (if asked bilateral than A) a. Decrease stroke volume
69. Most abundant collagen in dermis b. Cardiac output
a. type 1 Ans: B (cardiac output, stroke volume, EF increases)
b. type 3 81. Sodium nitroprusside function?
Ans: A a. Direct on vessels
70. Most teratogenic b. Centrally
a. Coffee Ans: A
b. Alcohol 82. What is live attenuated vaccine, In which
c. Smoking organism cannot reverse to its pathogenic form
Ans: B Ans: long-term immunity
71. Ventral Corticospinal ends at 83. erythropoietin stimulus
Ans: mid thoracic Ans: hypoxia
72. Skin have which collagen 84. IgA nephropathy
a. type 1 Ans: mesangial deposits in glomerular basement
b. type 2 membrane
c. type 3 85. Difference between ECF and ICF is?
Ans: A a. Low serum Ca Na and high K in ICF
73. Cardiac reserve b. Low Ca high Na High k in ICF
a. amount of blood that can be pump after Ans: A
normal flow 86. vertebral arteries give rise to
b. cardiac out per total body surface area Ans: basilar in cranium
Ans: A 87. Infra-temporal fossa is also termed as
74. Cardiac index is Ans: parapharyngeal
Ans: cardiac output to body surface area 88. Deglutition
75. Unmyelinated nociceptor afferent fibers relay in Ans: Vocal cord approximated
spinal cord where? 89. Severe diarrhea
a. Ventral Spinocerebellar a. Metabolic acidosis
b. Dorsal Spinocerebellar b. Metabolic alkalosis
Ans: A Ans: A
76. Selective proteinuria is found in 90. Bundle of his is supplied by
a. SLE a. RCA
b. Mesangial something b. LAD
c. Minimal change c. Rt marginal artery
Ans: C Ans: A
77. SLE chance genetic potential 91. Weight gain in the body is regulated weekly by?
a. Family member may get. a. Food
b. monozygotic twins b. Work output
c. more in females Ans: B
Ans: B 92. Which cannot be measured by spirometry
78. A patient has problem in fore arm and wrist on Ans: RV
examination found that upper arm vessels are 93. Cushing reflex
also disturbed. What is the site of lesion? Ans: DEC Heart rate
Radiant Notes—FCPS PEARLS Golden File 4 By: Dr. Rafi Ullah
94. fetal circulation 108. Cancer patient...having 5 yrs. survival less
a. inc systemic pressure than50%.
b. inc pulmonary pressure a. Cachexia
Ans: B b. Murmur
95. Regarding pulse pressure it is decrease in Ans: A
Ans: decrease in systemic pressure 109. Rapidly adapting sensory receptors
96. Anaphylactic and hypovolemic shock difference a. Pain
a. CO b. Cold.
b. HR c. Touch.
c. Systemic vascular resistance d. Spindle
Ans: A Ans: C
97. A person with MI admitted in ICU deteriorated 110. Graft rejection after 6 months
and hypotensive and cyanosis of fingers, what’s a. Acute Humoral Rejection.
the cause? b. Acute Cellular Rejection.
a. Ventilatory c. Chronic Rejection.
b. V/Q mismatch d. Hyper acute Rejection.
c. Right to left shunt Ans: B
Ans: B 111. Turner syndrome
98. RA Ans: 45X0
Ans: HLA DR4 112. Mitral stenosis murmur best heard at
99. organelle replicating Ans: apex
Ans: mitochondria 113. Anti-lymphocyte globulin given to patient
100. Trachea when enters thorax has following on and reaction occurred after many days. Which
its right? type of hypersensitivity reaction
a. Right recurrent laryngeal nerve a. Type 1
b. Vagus b. Type 2
Ans: B c. Type 3
101. In which disease is man reservoir d. Type 4
Ans: Hydatid e. Arthus reaction
Ans: C
102. Max heart rate Inc in?
a. Atropine infusion 114. Genesis for Long term Aspirin therapy.
b. Exercise Inhibition of.
c. Anxiety a. Leukotrienes.
Ans: B b. Thromboxane A2.
c. PGI2
103. A patient has embolus obstructing
Ans: B
pulmonary artery. What of the following will be
increased? 115. Bronchogenic Ca.
Ans: VQ Ans: tobacco smoking.
104. Relation of trachea in superior 116. Klinefelter’s syndrome.
mediastinum....what lies right to it Ans: Leydig cells hyperplasia.
Ans: Vagus 117. irregular p/v bleeding 42 yr. old uterine
105. Pelvic outlet most appropriate hyperplasia
a. AP diameter more than transverse a. inc progesterone
b. Transverse diameter more than AP diameter b. inc estrogen
Ans: B Ans: B
106. Emergency protocol 118. Alpha adrenergic effect
a. To keep only patient inside Ans: mydriasis
b. Psychological support to family 119. Mechanism of action of Ondansetron
Ans: A Ans: serotonin type 3 receptor antagonist
107. Free nerve ending 120. Anti-HTN of choice prior to surgery
a. only pain pheochromocytoma
b. Non-encapsulated a. both alpha & beta
Ans: B b. beta
c. methyldopa
Radiant Notes—FCPS PEARLS Golden File 4 By: Dr. Rafi Ullah
Ans: A Ans: phenyl alanine
121. Histopathological feature for metastasis. 134. Healthy adult 25 years loses blood of500ml
a. Atypia Ans: BP remains normal
b. Invasion 135. Early leprosy investigation of choice
c. Pleomorphism Ans: nasal scrapping
Ans: B 136. In liver glycogen is not converted in glucose
122. Lumbar vertebrae gives attachment to. Ans: enzyme deficiency
a. Extensors 137. Type of gangrene in diabetes
b. Transverse abdominis. a. Dry gangrene
c. Inferior oblique b. Wet gangrene
Ans: A Ans: A
123. Metastasis occurs by 138. Quack give some medicine which causes
a. detachment of tumor cells hepatic necrosis which of the following will
b. loss of e cadherin increase
Ans: B a. ALT
124. Histology of Macula densa...?? b. AST
Ans: I marked deep basal plasma lemma c. LDH
125. Which hormone causes protein fat Ans: A
carbohydrate storage 139. Heart rate increases markedly due to rise in
Ans: Insulin body
a. Temp
126. Open wound healing b. Anxiety
Ans: myofibroblasts contraction c. Emotion
127. Boy with fever, painful swollen joint(knee) d. Exercise
unable to move, causative organism Ans: D
a. strep viridans 140. Vit C deficiency
b. staph aureus a. Dec collagen
Ans: B b. defective collagen
128. A patient with seizures on carbamazepine. Ans: B
regarding antidepressants
Ans: TCA contraindicated. 141. alpha waves on EEG
Ans: disappears when eyes are opened
129. Reabsorption of potassium decreases 142. Colon carcinoma
Ans: decrease Na excretion in renal tubules (i Ans: autosomal dominant
marked) 143. Hemophilia
130. Slowly growing thyroid tumor Ans: X-linked recessive
a. Medullary 144. Protooncogens
b. Papillary Ans: point mutation
c. Follicular
d. Anaplastic Medicine and Allied 5th May Morning
Ans: B
131. Common most mode of inheritance
2017
1- Scenario was patient with pallor lethargy Hb
a. X-linked 7.5g/dl, WBC 3.5*10^9 where neutrophils were
b. autosomal dominant 11% and lymphocytes 88%, PLT 50K, most
c. Autosomal recessive appropriate for diagnosis
d. Multifactorial a. Bone marrow
Ans: D b. Coombs test
132. Quickening occurs at which month Ans: A
a. 4th month 2. 25 year old lady using some med presents
b. 5th month with bradycardia, Dec cardiac output pale skin
c. 6th month and increased peripheral vascular resistance.
d. 7th month The drug used is
Ans: B a. Epinephrine
b. Nor Epinephrine
133. Essential amino acid
c. Phenylephrine
Radiant Notes—FCPS PEARLS Golden File 4 By: Dr. Rafi Ullah
d. dopamine b) h influenza
e. Nifedipine c) strep pyogenes
Ans: C d) Klebsiella
3- In the presence of oxygen, end product e) Nesseria
of glycolysis is Ans: A
a) Lactic acid 12-ECG shows P wave is converted into small waves
b) 1 molecule of PA and QRS comes at irregular interval
c) 2 mol of PA a) A fib
Ans: C b) Atrial flutter
4- 08 months old child having esophageal c) Ventricular fibrillation
candidiasis, chest infection from pneumocystis d) wolf Parkinson white
carini, thin thymus, diminished body Lymph Ans: D (not sure about this-something wrong in
nodes and no germinal centers in Lymph nodes, statement)
causes is 13-RE entry circuit
a. antibody to B and T cells a- Ventricular hypertrophy
b. 3rd and 4th pharyngeal pouch defect b-a fib
c. HIV c-conduction block
d. failure of conversion of plasma cells to B cells d- paroxysmal
Ans: B Ans: B
5- Nerve block for upper abdominal viscera 14-Shwartzman reaction
a. Epidural a) complement mediated
b. Celiac block b) autoimmunity
c. Lumbar sympathetic c) endotoxin mediated something
Ans: B d) antibody mediated
6- About REM sleep Ans: C (ref: atlas of immunology 3rd Ed Pg.: 427)
a) nightmares occurs 15- HIV positive patient with chest infection is
b) BDZ Dec REM sleep prophylactically treated with
c) Locus ceruleus ceases NE firing a) folate synthesis inhibit
d) nucleus Raphe ceases serotonin firing b) cell wall synthesis inhibitor
Ans: both A and B (i prefer B) c) protein synthesis inhibitor
7- Patient with sweating palpitation -TSH was low d) TMP-SMX
that is raised after TRH administration. Cause is Ans: D (ref: first aid page 150 Ed: 7)
a. Hyperthyroidism with problem in thyroid 16- Pyrimethamine gives its anti shizontal action by
b. hyperthyroidism with problem in pituitary a) Tetrahydrofolate inhibitor
c. hyperthyroidism with problem in hypothalamus b) Decreasing glucose utilization by organism
d. hypothyroidism with thyroid problem c) antimetabolite that inhibit parasite folic acid
e. hypothyroidism with hypothalamus problem synthesis
f. hypothyroidism with pituitary problem Ans: C
Ans: A 17- Epithelium found upon surfaces exposed to
8- About dopamine receptor friction, which it serves to protect against
a) D1 acts via adenylyl cyclase mechanical injury,
b) D2 subtypes has no difference a) Stratified squamous keratinized
c) D1 causes Extrapyramidal symptoms b) Stratified squamous non keratinized
Ans: A c) Stratified columnar
9- Patient with pul: edema treated with a Ans: A
vasodilator that also causes decrease in preload 18- A child with decreased chest sounds on left
a) ACE inhibitors side? And decreased chest movements on left
b) sodium nitroprusside side, improves on holding him up and intestine
c) Glyceryl-Tri nitrate is present on left side of chest. Cause is
d) Hydralazine a. Patent pleuro peritoneal something
Ans: D b. Hiatus hernia
10- Ejaculation by c. diaphragm penetration
a) Sympathetic d. Dextrocardia
b) Parasympathetic Ans: B
c) Pudendal 19- A drug which blocks adenylyl cyclase and
Ans: C (ref first aid pg. 593, Ed 2017) inhibits cAMP can be used in the treatment of
11- 6months to 2years of age cause of a) Corynebacterium diphtheria
meningitis b) Pseudomonas aeruginosa
a) S pneumonia c) Vibrio cholera
Radiant Notes—FCPS PEARLS Golden File 4 By: Dr. Rafi Ullah
Ans: C 28-EGFR inhibitor can be given in treatment of which
20- Athetosis caused by failure of of the following cancers of lung
a) Globus pallidus a) Small cell carcinoma
b) caudate nucleus b) Large cell carcinoma
c) putamen nucleus c) Adenocarcinoma
Ans: C (while some friends are with option A , i am d) Squamous carcinoma
with C as it is due to defect in corpus striatum which e) Atypical carcinoid
is made from putamen and caudate, so option A Ans: C
doesn’t ring a bell also remember Putamen (motor) 29- Giant “a” wave in cardiac cycle in with right
while Caudate (cognitive). ventricular hypertrophy in
21-Thalamic hemorrhage Ans. Tricuspid stenosis
a) hyperesthesia 31- After an abdominal surgery blood was
b) numbness transfused to a patient, after which he developed
Ans: A jaundice, vomiting and lethargy. Hepatitis was
22- Direct Tributaries of brain veins diagnosed. Which is most commonly transmitted
a) Dural venous sinuses via blood transfusions.
b) internal jugular vein a) Hep A
c) emissary veins b) Hep C
d) diploic veins c) Hep D
Ans: A d) Hep B
23- Withdrawl reflex by Ans: D
a) pacinian corpuscles 32- Which will LEAST likely cause increased
b) merkel disc bleeding time
c) free nerve ending a) Prolonged use of aspirin
d) hairy cell b) Von Willebrand disease
e) Meissner c) Bernard Soulier
Ans: C d) Deficiency of factor 9
24- Lacrimal gland receives parasympathetic from e) Idiopathic thrombocytopenia purpura
Pterygopalatine ganglion that receives Ans: D
preganglionic fibers from 33- A patient presented with BP 70/40, CO 2L/min,
a) deep petrosal nerve CVP 2 cm, serum lactate increased. Cause
b) lateral petrosal nerve a) CCF
c) Greater petrosal nerve b) Cardiac temponade
Ans: C c) Septicemia
25- Regarding parotid gland d) Hypovolemic shock
a) no relation with pharynx Ans: D
b) have loose capsule 34-A patient presented with MI. After 4 days,
c) not related to any ear structure pericardial fluid drained, which is a marker for
d) deep processes relations with nerves decreased tissue perfusion
Ans: D Ans. Serum lactate.
26- Scleral icterus for 4 days. Malaise for one month. 35- Synapses are not present in
Rest of physical exam unremarkable. Lab a) Lateral gray horn
findings show Hct 25%. b) Ventral gray horn
a) Intravascular hemolysis c) Dorsal root ganglion
b) Increased iron absorption d) Dorsal root of spinal cord
Ans: A e) Ventral root of spinal cord
27- MCV would remain normal in which one Ans: C
a) Acute blood loss after 2 days 36-A 25 years old boy is treated for acute sever
b) Resection of terminal ileum Ulcerative colitis by
c) Gastrectomy a) sulfasalazine
Ans: A b) I/v hydrocortisone
27- Most common findings in autoimmune disease c) azathioprine
a) Weight loss d) metronidazole
b) Fever Ans: B
c) Arthritis 37- Skilled movements controlled by
d) Myalgia’s a) Corticospinal tract
e) Hematological abnormalities b) Cerebellum.
Ans: E Ans: A
38-Regarding Second heart sound:
Radiant Notes—FCPS PEARLS Golden File 4 By: Dr. Rafi Ullah
a) Low frequency than S1 Ans: B
b) Less duration than S1 50-First step in acute inflammatory response
Ans: B a) Pavementation
39-Brucellosis is transmitted by b) Margination
a) Man to man c) Migration
b) Raw milk Ans: B
c) Raw fish 51-Rt side radical mastectomy post patient develop
d) Fleas painless RT arm swelling due to
Ans: B a) Vascular insufficiency
40-A pregnant female carrier of hemophilia, b) Increase plasma cld osmtc pres
considering it is an x-linked recessive, which c) Lymphatic obstruction
information will be the most beneficial on Chorionic d) Hydrostatic pres Inc.
villous sampling Ans: C
a) Sex of baby 52- Child with red tissue in lower abdomen leaking
b) Trisomy urine and hypospadias, most likely
c) cytogenetic a) Bladder exstrophy
Ans: C b) cranio caudal defect
41-Patient was stressed about his disease and most Ans: A
concerned question is 53- Child with fever and posterior cervical
a) What is wrong with me lymphadenopathy. Which structure is most
b) Which medicines will I take likely to get damaged?
c) How much will the treatment cost a) CN 11
d) When will I get back to work b) CN IX-X-XII
Ans: A c) anca cervicalis
42. Removal of norepinephrine from synaptic cleft Ans: A
a) Reuptake 54-First rib fracture causes damage to
b) diffusion + reuptake +enzymatic degradation a) axillary artery
c) Diffusion and enzyme degradation b) subclavian artery
Ans: B c) brachial artery
43-Aspirin toxicity can be treated with Ans: B
a) NH4CL 55- An accessory cervical rib arising from
b) NaHCO3 transverse process of cervical vertebrae may
Ans: B cause damage to
44- An old man living beside a river, suffered a) C6
blindness Cause is b) C7
a) Oncherosis c) C8
b) vocheria d) T1
Ans: A Ans: D
45-Sequence of events 56-ECG shows prolonged PR. Echocardiography
Ans- Damaged valve, thrombus, bacteremia, perforation. shows no ventricular hypertrophy. Left axis
46-which is not sign of malnutrition deviation
a) dry pigmented skin a) Unifasiculus
b) leucopenia b) Bifasiculus
c) pitting edema c) Trifascuculus
d) cheilosis Ans: A
e) stomatitis 57- In strenuous, Increase ventilation is due to
Ans: B a) Dec arterial PO2
47-Erythropoitin secreted by b) Dec arterial PCO2
a) juxtaglomerular c) Dec Arterial Ph.
b) macula densa d) collateral stimulation from higher centers
c) peritubular capillaries interestitium Ans: D
Ans: C 58-Female with symptoms of chest infection. She
48- Cause of diplopia goes on vacations and gets relieved her symptoms.
a. one eye myopic n hypermetropic When she came back from vacations she develops
b. both eyes myopic and extronopia again cough fever. CXR chows diffuse pulmonary
Ans: A nodules. The most likely reason is
49-Which one is most important in K uptake in cells. a) Mast cell degranulation
a) Aldosterone b) interstial fibrosis
b) Insulin. c) Langerhans cell histiocytosis
Radiant Notes—FCPS PEARLS Golden File 4 By: Dr. Rafi Ullah
Ans: C (although few frnds says option A, but it a) Failure to absorb glucose in PCT
seems to me as Pulmonary Langerhans cell b) Decrease threshold for glucose
histiocytosis (eosinophilic granuloma) c) decrease ability to absorb glucose
59- Child with fever arthritis, rash and blindness d) she has no glucose in urine
a) Henoch-Schönlein purpura Ans: B
b) Lyme disease 70- Regarding super antigen
c) chronic juvenile arthritis a) binds directly to T cells
d) rheumatic fever b) causes cytokine storm from T cell
Ans: C Ans: B
60- A premature baby is born with decrease 71- Electrical muscle activity of duodenum
surfactant. What would be the result when a) greater than last portion of small intestine
comparing to normal neonate b) same as stomach
a) Dec surface tension and Inc compliance c) Affected by circulating levels of hormones
b) Increase surface tension and Dec compliance Ans: C
c) increase surface tension and Inc compliance 72. Radiation therapy, lesions on skin due to
Ans. B Ans: Endarteritis obliterans
61- Patient with mid-systolic murmur. Echo shows 73- MCC of MI due to
leaflet of mitral valve displaced upward and a) Atheroma
aortic insufficiency. Lens dislocation, Defect in b) Thrombosis
a) collagen Ans.B
b) fibrillin 74- Electrical activity of stomach
Ans: B a. higher at pyloric region compared to fundus
62- Directly related to hip joint b. start at pyloric region
a) inferiorly obturator externus c. start at body and moves toward pylorus
b) sciatic nerve anteriorly d. lowest at pyloric region
c) psaos bursa posteriorly Ans: A (basic electrical rhythm increases towards
d) gluteus medius inferiorly/posteriorly antrum)
Ans. A 75-MOA of BDZ
63- Deep inguinal ring in a) potentiate GABA
a) transversalis fascia b) inhibits GABA
b) ext. oblique c) activates Glutamate
c) internal oblique Ans: A
d) transverse abdominis 76-Early diagnosis and treatment
Ans: A a) primary prevention
64- Great saphenous vein b) secondary prevention
a) forms behind medial malleolus c) tertiary prevention
b) ends at 3.5cm below and lateral to pubic d) hospital services
tubercle e) social services
c) receives only one tributary before joining femoral Ans: B
vein 77- Superior border of inguinal canal
Ans: B Ans: conjoint tendon
65-Regaring lumbar plexus 78- Gas gangrene caused by
a) obturator nerve arises from it a) Clostridium biregrengi
b) forms by upper 4 lumber nerves b) clostridium welchii
c) inferior hypogastric arises from L4 Ans: B (perfringes not in option)
Ans: Both A and B, I prefer B (FormationAnterior 79- Characteristic feature of c.botulinum ingestion
rami of upper four lumbar nerves L1, L2, L3, L4) a) diarrhea
67- Phrenic nerve is related to b) resp: Failure
a) Pericardiophrenic artery c) seizures
b) Musculophrenic artery Ans: B
Ans: A 80- Medial leminiscus formed by
68- Muscle of sadness and fright a) pyramidal tract
a) Platysma b) Ant:spinothalmic tract
b) orbicularis oculi c) Internal arcuate fibers
c) masseter Ans: C
Ans: A 81- Long term smoker feeling difficulty in swallowing
69- Patient with DM has glycosuria. Her blood sugar with solids. Appropriate statement
level is 100mg/dl. What os more appropriate a) Dec antigravity
in this patient
Radiant Notes—FCPS PEARLS Golden File 4 By: Dr. Rafi Ullah
b) Dec peristalsis c. Melatonin
c) damage to neurons Ans: C
Ans: B 97. 41 % Hematocrit
82- Hypertensive on medication gets her LFTS a. RBC’S
dearranges. Culprit is b. WBC’S
a) methyldopa c. ALL CELLS
b) Hydralazine Ans: C
c) Nifidipine 98. Carpel tunnel syndrome
Ans: A Ans: myxedema
83- MCC of DVT 99. Most common symptom of autoimmune
a) immobilization diseases...?
b) surgery a. hematological changes
c) Protein C deficiency b. arthritis
Ans: A c. fever
84-Drug of choice for Hypertrophic obstructive Ans: B
Cardiomyopathy 100. Life span of sperm
a) Verapamil a. 24.48 hr.
b) Diltiazem b. 24.72 Hr.
c) Nicardipine Ans: B
Ans. A 101. Cortisol.
85- anti t.b causing color blindness Ans: Zona Fasciculata
a. INH 102. Staging
b. Rifampicin a. extent of invasion
b. lymphovascular invasion
c. Ethambutol c. hyperchromazia
Ans: C
d. inc no. of mitosis
86-Similarity between LMWH and heparin Ans: A
a) Dose dependent clearance
103. Mean arterial pressure………….
b) Same plasma protein binding
Ans: Diastolic pressure + 1/3rd of pulse pressure
c) both act same on Xa
Ans: C 104. Child with megaloblastic anemia scenario
investigation?
87- Most Radiosensitive
Ans: Vit 12
a) Glioma
b) Craniopharyngioma 105. Cells diff in thymus to become
Ans: A a. TH-1
b. TH-2
88. Withdrawal reflex is mediated by
c. cytotoxic cells
Ans: nociceptor
Ans: C (it differentiated into both helper as well as
89. Infectious mononucleosis dx……….. cytotoxic, but if u have to choose one choose cytotoxic)
Ans. heterophile sheep test
106. Renal column….
91. Nucleus pulposus is remnant of? Ans. interlobar artery
Ans. Notochord
107. Delayed effect of insulin
92. In spinal tap, CSF was blood stained what will be Ans: m- RNA transcription of lipogenetic enzymes
the diagnosis?
108. Nasal septum develops from
Ans. Subarchanoid hemorrhage
a. medial nasal process
93. Lateral nucleus b. frontal nasal process
Ans. hunger loss c. lateral nasal process
94. Potent stimulus for renin Ans: A
a. Hypotension 109. Achondroplasia
b. Dec tubular NaCl load a. autosomal dominant
c. Sympathetic nerves b. autosomal recessive
Ans: C c. X linked recessive
95. Esophageal compression Ans: A
a. Lt atrium 110. Insulin decreases
b. Rt atrium a. Glucagon synthesis
Ans: A b. Lipolysis
96. which hormone inc at night and inhibit GnRH c. Ketogenesis
Hormone Ans: C
a. Growth hormone 111. Digastric muscle middle attachment
b. Prolactin Ans. Hyoid
Radiant Notes—FCPS PEARLS Golden File 4 By: Dr. Rafi Ullah
112. What’s present between individual muscle 128. Surfactant def…..
fiber Ans. Inc surface tension and Dec compliance
a. Basal lamina 129. Anti T.b multi drug…..
b. Endothelial Ans. to Dec resistance
c. Epimysium 130. Rickettsia……
d. perimysium Ans. Neg cell wall bacteria
e. Endomysium 131. Alleles….
Ans: E Ans. non identical genes on same locus
113. Carcinoma is caused by which 132. Intestinal motility increased by
inorganic compound? Ans: hormones…
a. asbestoses
133. Regarding autonomic nervous system
b. Silica a. All sympathetic nerves are adrenergic
c. Dust b. All sympathetic nerve are cholinergic
Ans: A c. All parasympathetic nerves are cholinergic
114. Farmer's lung Ans: C
a. Sugarcane dust 134. Plasma proteins bind drug………
b. Grain dust Ans. inactive till reach liver
Ans: B 135. Micelles formation
115. muscles of back innervated by Ans: for carrying fatty acids
Ans. dorsal rami 135. Sympathetic through alpha-adrenergic
116. Which is reversible? a. Control radial muscle of iris
a. Anaplasia b. Glycogenolysis
b. Metaplasia Ans: A
Ans: B 136. High membrane permeability
117. Regarding defecation reflex Ans. water
a. Anorectal 137. TMJ fracture…..
b. rectoanal Ans. lateral pterygoid muscle
c. haustration 138. Can be used in emergency without cross match
Ans: B Ans: O negative
118. Regarding SLE 139. MI scenario after 4 days
Ans: HLA b27 not in SLE Ans. neutrophils + macrophages…
119. SA node 140. Langerhans cells in pancreases more…..
Ans. generate impulses Ans. Tail
120. which one is benign tumor 141. Cervical Ca
Ans: Warthin tumor Ans. HPV
121. Irregularly irregular QRS complexes 142. Dec venous tone
a. AF Ans. Dec atrial pressure
b. VT 143. Antivirals
c. ventricular fibrillation Ans: Inhibit protein synthesis…………….
Ans: A (incomplete stem-poor recall) 144. Viruses causes cancer by
122. RT side radical mastectomy post patient Ans: Alteration in protein synthesis
develop painless RT arm swelling due to... 145. Counselling
a. Vascular insufficiency Ans: Done for patients to help themselves
b. Increase plasma cld osmtc pressure 146. Sepsis dx….
c. Lymphatic obstruction Ans. WBC’s 1500000 (if blood culture in option prefer
d. Hydrostatic pressure Inc... that)
Ans: C 147. Marathon racer taking distilled water,
123. Regarding Vit-C Ans. Inc ECF volume (more water is lost than salt,
Ans: Hydroxylation of proline ↓ECF VLOUME and ↑ ECF osmolarity because more
124. Patient is pregnant and suddenly faints. water is lost than salt, so osmoles concentration is high
She'll be positioned in left lateral and not right comparatively----as a result water shifts out of cell ICF to
lateral to avoid compression to which abdominal ECF( because water molecules move from low solute to
structure. high solute concentration----as a result ↑ECF volume,
Ans: IVC…. ↓ICF volume)
126. Produced by intestinal bacteria 148. Ca deficiency …..
Ans: Vit k… Ans. difficult labor
127. Radiosensitive Structure………. 149. TB diagnoses
Ans. lymph node a. AFB
Radiant Notes—FCPS PEARLS Golden File 4 By: Dr. Rafi Ullah
b. Caseous necrosis Medicine and Allied 5th May 2017
Ans: A
150. Clinically GFR is calculated by?? Evening,
Ans: creatinine Clearance 1. Maximum feedback gain in controlling
151. Overexcited chemoreceptors….. blood pressure is by which one of the
Ans. pco2 above 60 following
152. Trachoma a) CNS ischemic response
Ans: chlamydia b) Baroreceptors
Ans: B
153. Long pro erythrocytic phase….
Ans. p malaria 2. Parasympathetic supply was asked of
rectum
154. Drug of choice in hypertrophic obstructive
a. S 1,2, 3
cardiomyopathy
b. S 2,3, 4
Ans: verapamil
c. S 3,4, 5
155 Acute overdose TX with bicarbonate d. L2
Ans. Phenobarbitone e. S2
156. CNS arise from……………… Ans: B
Ans. ectoderm 3. Elongated body and gynecomastia was
157. Common side effect theophylline and nitrates? present in the scenario. Diagnosis was
Ans. Tachycardia asked
158. Axillary artery……….. a. Klinefelter
Ans. posterior to pectoralis minor b. Turner’s
159. Respond to Dec pco2 c. Edward
Ans. carotid chemoreceptors d. Patau
160. Cell survival is due to Ans: A
Ans: p53 4. A man was involved in an RTA with
161. Broca s area………… multiple rib fractures. He has difficulty in
Ans. INF frontal gyrus breathing. You notice that his abdomen is
162. Psycho-physio action showing brain activity moving more during breathing. Which of
Ans. pupillary response the muscle is helping him breathe?
163. Sympathetic cholinergic a. Diaphragm
Ans. sweating b. External intercostal
c. External oblique
164. Metastasis
d. Internal intercostal
Ans: loss of E cadherin’s
e. Transversus thoracic
165. Inhibit GH…………. Ans: C
Ans. brain somatostatin 5. Minimum absorption of drug from its site
166. Not to atrium drain………… of absorption, for its desirable effect
Ans. pulmonary veins occurs if
167. Root of spinal nerves a. It undergoes extensive first pass metabolism
Ans. enter through intervertebral foramina b. It makes active metabolites in liver
168. Surfactant action c. It has high renal clearance
Ans. prevent alveoli collapse d. Drug has narrow therapeutic index
169. Pregnant lady with obstructive jaundice best e. If it's used for local action
test? Ans: D
Ans: GGT 6. What is present in lateral sulcus of brain??
170. Thyroid is in a. Insula
Ans. Pretracheal fascia b. Splenium
171. Person taking fats, he should take Ans: A
Ans. biotin 7. A woman is complaining of decreased
172. T cells…. sensations, in medial half of hand. Movements
Ans. MHC 1 of short muscles of right hand affected. On
173 slow Ca thyroid examination pulses of her right hand are weak.
Ans: Papillary Reason for this?
A. Cervical rib
174. Diarrhea……………
B. C8 damaged
Ans. metabolic acidosis with normal anion gap
Ans: A
8. Sensory supply of neck from chin to
supraclavicular notch?
Radiant Notes—FCPS PEARLS Golden File 4 By: Dr. Rafi Ullah
a. C3, C4 Ans: A
b. C1, C2 18. Parasympathetic increases salivary
c. C2, C3 secretion, through which of the following
Ans: C will it act on blood vessels?
9. A man is sweating 2 liters and infused 1 liter a. Substance P
N/s. What will change b. VIP
a. Increase ICF c. Gastrin
b. Increases ECF Ans: B
c. Inc osmolarity 19. Long stem. A lady in oncology ward
Ans: A developed fever. Pseudomonas was
10. A boy is busy in a well-ventilated room. He is identified. What is responsible for
sweating and hyperventilating. Body this
temperature is 26’c. HR 120, BP 150/110. A. TNF
What has happened? B. Granulocyte stimulating factor
a. Heat stroke Ans: A
b. Exercise 20. Regarding Corynebacterium Diphtheria,
c. Excitement following is true
d. Sympathetic over activity 1. It acts by exotoxins
e. Mental stress 2. Primary site of infection is always throat
Ans: B Ans: A
11. Parasite rate is calculated for which age 21. Long stem about cancer. In the end it was
group? ask which cells will combat cancer cells.
a. Infants Ans: NK cells
b. Less than 5 years 22. A man has presented with water
c. 5-15 years intoxication. Which of the following is
d. Less than 15 years the characteristic sign of water
Ans: B intoxication?
12. Important adverse effect of protamine is? A. Slow pulse
a. Hypotension B. Increased pulse
b. Bleeding C. Distended neck veins
c. Anaphylaxis Ans: A
Ans: A 23. Main reason for development of brain
13. A man has tremors whenever he performs abscesses:
a voluntary action. Site of lesion is? a. Hematogenous spread of infection
a. Cerebellum b. Neurosurgical intervention
b. Basal ganglia c. Penetrating wounds
c. Pons d. Frontal trauma
Ans: A e. Infections around cancer in brain
14. A surgeon is performing bronchoscopy on Ans: A
a patient which of the following will be seen 24. Virus causes cancer by alteration in
first by the surgeon? A. DNA
A. Right lower lobe B. RNA
B. Anterior basal C. proto oncogene
Ans: A D. Oncogene
15. Eversion and inversion occurs at which joint Ans: C
a. Subtalar joint 25. On ECG of a girl, you see saw tooth waves,
b. Ankle joint with P waves in a ratio of 3:1. Her pulse is
c. Tibiofibular regularly irregular. Which of the following
Ans: A has occurred?
16. Facial pain, hyperacusis. Ganglion involved a. Atrial flutter
was asked b. Atrial fibrillation
a. Trigeminal ganglion c. Ventricular tachycardia
b. Geniculate ganglion
d. Ventricular fibrillation
c. Ciliary ganglion
Ans: B e. Ventricular flutter
17. Which drug does not exert its affect by acting Ans: A
on receptors 26. Median umbilical ligament is a remnant of?
a. Aluminium hydroxide A. Urachus
b. Morphine B. Umbilical artery
Radiant Notes—FCPS PEARLS Golden File 4 By: Dr. Rafi Ullah
C. Umbilical vein 36. There is a mass in front of the neck which
Ans: A moves during swallowing. In which of the
27. A pregnant lady has presented to you with following layers is it enclosed?
DVT. Regarding warfarin which is true? A. Pre tracheal
A. Not to be given in pregnancy B. Pre vertebral
B. It is antagonized by Vitamin K Ans: A
C. It is rapidly antagonized by FFP's 37. Which is a slowly progressing tumor of
Ans. All of them correct. Must be something thyroid?
wrong with the stem or either with CPSP :P A. Epidermoid
28. A man is transfused 2 weeks old blood. Which B. Papillary
of the components will it mainly contain? C. Medullary
A. RBCs D. Follicular
B. Lymphocytes Ans: B
C. Platelets 38. A woman has presented with dysphagia which
D. Monocytes is worsening since 3 months. Now it is
E. Neutrophils difficult for her to swallow pudding. On
Ans: A examination of the neck surgeon notices
29. Most potent antioxidant among the following? enlarged lymph nodes on the side of her neck.
A. Vitamin A What is the cause?
B. Vitamin E A. Ca Esophagus
C. Vitamin C B. Ca trachea
Ans: B C. TB
30. Which of the following is related to cancer? Ans: A
A. Silicosis 39. A woman has received a loop diuretic.
B. Anthracosis Loop diuretic causes decreased
C. Asbestosis reabsorption of NaCl in the loop of Henle.
Ans: C Which of the following will occur?
31. A man working in a factory has A. Decreased osmolarity of medullary interestitium
developed decreased sensation in B. Negative water clearance
legs. Exposure to which of the C. Increased medullary interestitium osmolarity
following is responsible for this? Ans: A
A. Copper 40. Regarding sperm reaching epididymis?
B. Lead A. Circular movement
C. Mercury B. Movement becomes directional
D. Iron C. Acrosomal reaction takes place
Ans. B D. Capacitance occurs here
32. True regarding carcinoid heart disease is Ans: B
A. It occurs only in cancers 41. Regarding fibrinogen which is true=
B. Woman are affected more A. Effects ESR
C. Thickening of tricuspid valve B. Decreases in liver disease
Ans: C C. Takes part in clot formation
33. Regarding anal canal which is true = Ans: A
Ans: supplied by both superior & inferior rectal artery 42. Regarding Resistance in a blood vessel, it is
34. Primary control of secretion from Zona directly proportional to
Glomerulosa is by which hormone? A. Viscosity
A. Angiotensin 2 B. Fourth power of diameter of vessel
B. Renin C. Length of vessel
C. ACTH Ans: A
Ans: A 43. What will happen in acute tubular acidosis?
35. What is the most important diagnostic feature Ans: Hyperkalemia
of malignancy? 44. The main circulatory drive of blood flow is
A. Invasion a. Ventricular contraction
B. Pleomorphism b. MAP
C. N/C ratio c. Pulse pressure
D. Metastasis Ans: B
Ans: D 45. Supportive cell of par nervosa??
A. Astrocyte
B. Pituocyte
Radiant Notes—FCPS PEARLS Golden File 4 By: Dr. Rafi Ullah
Ans: B Ans: C
46.Regarding CSF what is correct= 54. A boy has developed bruise and swelling.
Ans: Contains proteins 20-40 mg/dl What caused this swelling?
47.Blood supply of head of humerus is? A. Carbs
A. Anterior circumflex B. Fats
B. Posterior circumflex C. Proteins
C. Arcuate artery D. Water n electrolytes
D. Axillary artery Ans: C
Ans: C 55. About N-REM what is true
48. Bell's phenomenon is A. Synchronization of Delta waves occurs
A. Bell's palsy B. Nightmares
B. Eye globe downwards and drooping of lid C. Variable blood pressure and muscle tone
C. Eye globe upwards, drooping of lid Ans: A
D. Eye globe moves upward when he tries to close 56. Which of the following releases a peptide
his eyes involved in bone growth?
Ans: D A. Liver(ans)>>>IGF 1
49. A woman works in a factory, has B. Brain
developed pulmonary fibrosis. What C. Kidney
changes will be seen in spirometry D. Lungs
A. Decreased diffusion of O2 Ans: A
B. Increased residual volume 57. Transient cell adhesion is done by=
Ans: A A. Integrins
50. A pharma representative has come to you B. Selectins
and is offering you a free 3 day trip to C. Proteoglycans
Thailand. He tells you that there will be an (E-cadherin was not in the options)
academic panel discussion in your hospital Ans: B
about a newly launched drug and wants you 58. Type of necrosis seen in Omentum?
to take part in it. A. Fat necrosis
A. Accept the offer B. Liquefactive necrosis
B. Politely decline the offer C. Coagulative necrosis
C. Do what your colleagues will do Ans: A
D. Ask your hospital administration what to do 59. An infection has spread to the brain. Which type
Ans: B of necrosis will be seen?
51. Fine touch, vibration, two point A. Liquefactive necrosis
discrimination of left leg are affected by B. Coagulative necrosis
lesion of? C. Fat necrosis
A. Nucleus gracilis D. Gangrenous
B. Nucleus cuneatus Ans: A
C. Posterior Spinocerebellar 60. A child is born with cataract. Which of the
D. Anterior white column following would have affected his mother
E. Lateral white column during pregnancy?
Ans: A A. H.influenza
52. A woman was cycling and reached a B. Rubella
height where atmospheric pressure was Ans: B
700mmhg. Regarding gases which of the 61. A pregnant woman has presented with German
following is correct? measles at 6th week
A. PaO2 will be 147 and Nitrogen will be 553mmhg of pregnancy. Which of the following is her
(ans) child most likely to develop at this stage?
B. PaO2 110, Nitrogen 583 A. Deafness
C. Pa02 92, Nitrogen 608 B. Cataract
Explanation=O2 percentage in air is 21, C. Septal heart defects
so 700*21/100=147 and Nitrogen D. Another option on heart defect
percentage is 79, so 700*79/100=553 Ans: B
Ans: A 62. A boy has systolic BP 132 mmHg and
53. Regarding primary motor cortex what is diastolic BP 66 mmHg. Calculate the
true= Mean arterial pressure.
A. Receives no sensory input A. 66
B. Pyramidal tract B. 88
C. Receives sensory input C. 111
Radiant Notes—FCPS PEARLS Golden File 4 By: Dr. Rafi Ullah
Explanation: C. PT
MAP=2/3rd diastolic+1/3rd systolic Ans: A
= (2/3x66) + (1/3x132) 73. Two questions on Niacin were
= 44+44 there which is (same question
MAP=88(ans) repeated twice) Pellagra caused
Ans: B by which vitamin?
63. Regarding lysosomes what is true? A. Thiamine
A. Contains hydrolytic enzymes B. Niacin
B. contains oxidases C. b12
C. Produced from SER D. Pantothenic acid
Ans: A Ans: B
64. A boy had trauma to pelvis and his 74. Phosphodiesterase inhibitor MOA:
acetabulum moved in postero- A. Inhibit conversion of cAMP to 5 AMP
superior direction. Which of the 2 B. Inhibits conversion of ATP to AMP
bones are fractured? C. Activates adenylate cyclase
A. Ischium and Ilium Ans: A
B. Pubis and ileum 75. Bone derived from 2nd pharyngeal arch
Ans: A A. Squamous part of temporal bone
65. Regarding exudate what is true? B. Styloid part of temporal bone
A. Specific gravity less than 1.012 C. Zygomatic bone
B. Has high protein content D. Maxilla
Ans: B E. Palatine
66. A man with COPD developed resp. acidosis. Ans: B
What occurs to potassium level? 76. Scenario of 22 years old girl having
A. Rise as a substitute for aciduria recurrent episode of Staph aureus
B. Fall as it has inverse relation with H excretion infections and 2 episode of
Ans: A aspergillosis. Diagnosis was asked.
67. Co2 in blood transported maximum in which A. C1 esterase deficiency
form? B. NADPH deficiency
A. Bicarbonate C. IgA deficiency
B. Carboxyhemoglobin Ans: B
C. Dissolved in plasma 77. Which of the following acts through cAMP:
Ans: A A. D1
68. A boy is suffering from vasculitis, B. H1
arthritis and malar rash. Which of C. 5HT3
the following will be seen in him? D. 5HT4
Ans: Complement deficiency Ans.A
69. Melatonin and ACTH are involved 78. MOA of ondansetron in vomiting?
in circadian rhythms. Which part A. Inhibits 5HT3 receptors
is responsible for regulating B. Inhibits release of dopamine in chemoreceptor
circadian rhythms? trigger zone
A. Supraoptic nucleus Ans: A
B. Pre optic nucleus 79. Regarding Tractus Solitarius which is true?
C. Suprachiasmatic nucleus A. 1st order neurons from taste pathway
Ans: C B. 2nd order neurons from taste pathway
70. Type 1 collagen is seen in- Ans: B
A. Skin 80. When AV conduction velocity is slowed down,
B. Epiglottis what will be seen?
Ans: A A. Increased heart rate
71. Dorsalis pedis artery is missing B. Increased PR interval
congenitally. Which artery will supply Ans: B
dorsum of the foot? 81. Cardiac tissue is most vulnerable to ventricular
A. Peroneal artery fibrillation at which point?
B. Median Plantar A. At the start of action potential
C. Lateral Plantar B. Just at the end of action potential
Ans: A C. When the action potential is about to end
72. In ITP which one of the following is deranged D. At peak of action potential
A. BT Ans. C
B. APTT
Radiant Notes—FCPS PEARLS Golden File 4 By: Dr. Rafi Ullah
82. Superior colliculus is compressed. It will affect Ans: A
eye muscles and cause what? 89. A man was hiking and noticed a rash
A. Nystagmus appeared on his arm which was not
B. Convergence covered by clothing. The rash grew in
C. Contralateral saccades size for 2 days and then subsided after
D. Ipsilateral saccades 2 weeks. Which of the following
Ans: C reaction took place in this person?
83. A 55 year old lady with swelling of both A. Type 1 hypersensitivity
legs and B/L pleural effusion presents to B. Type 2 hypersensitivity
you. Lab values= AST 263, ALT 223, LDH C. Type 3 hypersensitivity
760, CK 150. No fever or other symptoms. D. Type 4 hypersensitivity
Diagnosis? Ans. D
A. Reno vascular hypertension 90. Thermogenesis is carried out by?
B. Recurrent thromboembolism A. Posterior hypothalamus
C. Rheumatoid arthritis B. Anterior hypothalamus
Ans. B (Remember this BCQ is taken from Ans: A
USMLE review notes---Pulmonary hypertension 91. A woman had a Pap smear done. It
and subsequent right heart failure can occur in showed big cells (Characteristics of
the small number of cases in which recurrent dysplastic cells were mentioned).
thromboembolism takes place. The right heart What type of change has occurred?
failure that occurs then leads to hepatic passive A. Dysplasia
congestion with centrilobular necrosis that is the B. Metaplasia
cause for the increased transaminases and LDH C. Hyperplasia
(but normal CK, since the heart is enlarged, but D. Hypertrophy
not ischemic) Ans: A
84. Scenario on increased hepatic 92. Which of the following is an opportunistic
glucagon level, no change on bacteria?
giving oral glucose. Oral fructose A. Staph aureus
level was normal. Enzyme B. Strep pyogenes
deficient? C. Histoplasma
A. Hexokinase D. Corynebacterium jeikium
B. Fructokinase Ans. D
C. Glucokinase 93. Which cancer involves peripheral nerve
D. Glucose 6 phosphatase sheaths?
Ans. D A. Ex-pleomorphic adenoma
85. A man has presented to you in ER after 6 B. Adenoid cystic
hours of MI. Which enzyme is most likely Ans. A
to be raised in this pt 94. How can right kidney differentiated from left
A. LDH kidney
B. CPK Ans: Relations at hilum
C. Trop I
95. A surgeon is performing open heart surgery.
D. Trop T
Which vessel is running close to LAD and can be
Ans: B
damaged?
86. What is the fuel of liver in its post absorptive A. Great cardiac vein
form? B. Anterior cardiac vein
A. Glucose C. Middle cardiac vein
B. Fatty acids Ans: A
C. Lactate
96.Calcium requirement in pregnancy is=
Ans: B
A. 1200
87. On auscultation first heart sound is variable B. 400
in= C. 800
A. Atrial flutter D. 600
B. Atrial fibrillation Ans: A
C. Ventricular tachycardia
97. Most common cause of death in rheumatic fever
D. Ventricular flutter
A. Mitral stenosis
Ans: B
B. Endocarditis
88. Type of epithelium in intestine? C. Pericarditis
A. Simple columnar D. Myocarditis
B. Simple columnar ciliated Ans. D
Radiant Notes—FCPS PEARLS Golden File 4 By: Dr. Rafi Ullah
98. A person who returned from a trip in Cairo Ans.B (Ref: pharmacology pharmacy technique page 549
presented with painless hematuria. Which type chapter 33 which states that long acting OR
of cancer has he developed? intermediate acting insulin twice a day)
A. Squamous cell carcinoma 108. Maternal inheritance pattern seen in
B. Transitional cell carcinoma A. Mitochondrial DNA
Ans: A B. Mitochondrial RNA
99. Chromosome replication occurs in which stage? Ans: A
A. Interphase 109. Which structure damage during
B. Metaphase appendectomy, while giving incision
C. Anaphase on Mc’burney’s point?
Ans: A A. Deep circumflex iliac artery
100. A man was found to have history of B. Illiohypogastric nerve
smoking multiple packs per day, has C. Genitofemoral nerve
developed small cell carcinoma (oat Ans: B
cell). Which will be secreted by it? 110. Chorda tympani carries?
A. PTHrP A. Secretomotor fibers to parotid
B. ACTH B. Supplies posterior 1/3rd of tongue
Ans: B C. Joins lingual nerve
101. TCA overdose causes D. Supplies mucosa of middle ear cavity
A. Tonic clonic seizures Ans: C
B. Pinpointed pupils 111. A woman developed sciatic nerve
Ans: A compression at L5 what will happen?
102. Lympahatic drainage of lower lip A. Weakness of plantar flexors
A. Submental + submandibular B. Weakness of dorsiflexors
B. Submandibular + parotid C. Absent ankle reflex
C. Jugulodiagastric D. Brisk ankle reflex
Ans: A Ans: B
103. Which of the following about lymph 112. Regarding RBCs
flow is correct= A. Largest blood cells
A. Increases by massage B. Biconvex
B. Decreased by incompetent calf muscles C. They have glycolytic enzyme activity
Ans: A Ans: C
104. Lymph flow decreased by 113. Scenario of a woman with myxedema and
A. Hemorrhage macrocytic anemia. What can be detected
B. Increase venous return in her?
Ans: B A. Anti-parietal cell antibody
105. Which is the longest phase of HIV B. Antibody against intrinsic factor
without ANY symptoms? Ans: B
A. Incubation period 114. A patient is taking multiple drugs
B. Acute phase to treat his TB. Now he is unable
C. When overt AIDS converts in active to differentiate between red and
D. Latent phase green light. Drug responsible for
Ans: D (Ref: HIV AIDS Care and Counselling: A this is=
Multidisciplinary Approach by Alta C. Van Dyk) A. Rifampicin
B. Ethambutol
106. about osteogenesis… (Cannot recall other
C. Isoniazid
options.
Ans: B
A. One was non calcified bone is called osteitis
(something like that) 115. A painter has developed cough which
B. Intramembranous ossification is when bone subsides on Sundays. What has he
develops from cartilage developed?
Ans. (Poor recall) A. Allergic alveolitis
107. Girl with type 1 diabetes have mostly hyperglycemia B. Occupational asthma
in morning .which is appropriate option? Ans: A
a. regular insulin before meal short acting insulin 116. A patient has presented to you in ER
b. twice b4 meal long acting insulin twice a day with vasodilation and hypotension.
c. b4 meal long acting and intermediate insulin twice Which type of shock is causing this?
a day A. Septic shock
d. intermediate insulin twice a day B. Hypovolemic
C. Cardiogenic
Radiant Notes—FCPS PEARLS Golden File 4 By: Dr. Rafi Ullah
D. Vasovagal D. IgA deficiency
Ans: A Ans: A
117. Major driving force for bicarbonate 127. Ventricle maximally filled in
absorption= A. Rapid inflow
A. Na, H+ antiport at luminal surface B. Atrial systole
B. Increased bicarb secretion from cells to lumen C. Isovolumetric relaxation
Ans: A Ans: A
118. A women has undergone surgery for 128. Verapamil MOA on heart?
craniopharyngioma. Which is the best to A. SA node automaticity
give for ovulation? B. Increases intracellular calcium
A. Clomiphene citrate C. Increases AV nodal delay
B. First human menopausal gonadotropins D. Decreases AV nodal delay
followed by human chrionic gonadotropins Ans: C
C. Pulsatile gonadotropins releasing hormones 129. Superior radioulnar joint type
D. Continuous gonadotropins releasing hormones A. Pivot
Ans: B B. Hinge
119. Which cells are a part of innate C. Condyloid
immunity? D. ellipsoid
Ans: NK cells Ans: A
120. Which of the following transmitted 130. Functional residual capacity
through feco oral route? Ans: ERV+RV
A. Hep B 131. Diabetic pt with decreased sensation in foot
B. Hep C and ulcer on big toe
C. Hep D A. Neuropathy and Angiopathy
D. CMV B. Angiopathy
E. Hep E C. Neuropathy
Ans. E Ans: A
121. Hypoxemia in blood is sensed by 132. What should be checked to diagnose Hep B
A. Chemoreceptors disease?
B. Central receptors A. HbS antigen + anti-HBC
C. Carotid body B. HBsAg + HBe antigen
D. Carotid sinus C. Anti HbC
Ans: C Ans: B
122. Which is an immuno-stimulatory hormone? 133. A woman developed pneumonia and
A. GH (before puberty) wound infection during her stay in
B. Cortisol hospital. Gram positive rods are
C. Thyroxine seen forming green colonies.
Ans: A Organism responsible for this:
123.Aldosterone is produced by which layer? A. Pseudomonas
A. Zona Glomerulosa B. Staph aureus
B. Zona Fasciculata C. Klebsiella
C. Zona Reticularis Ans: A
Ans: A 134. Cause of false negative Mantoux test
124. About hyaline cartilage, which is true in T.B pt
A. Collagen fibers are visible Ans: Immunosuppression
B. Fibers are not visible 135. In Asthenuria what to be checked for
C. Both elastic and collagen fibers are present to see tubular concentrating function
Ans: B A. Urinary Na
125. Howship lacunae contains B. Hyperosmolar plasma
A. Osteoclasts Ans: A
B. Osteoblasts 136. In a population, a group of people with a
Ans: A particular feature are represented by
125. An 8 months old child with recurrent A. Prevalence
respiratory infections, sinusitis, multiple B. Incidence
episodes. B cells are reduced. All C. Distribution
immunoglobulins are low. Diagnosis asked D. Occurrence
A. X-linked Agammaglobunemia Ans. D
B. Combined immunodeficiency
C. SCID
Radiant Notes—FCPS PEARLS Golden File 4 By: Dr. Rafi Ullah
137. A smoker working in a tyre factory had was given in the scenario
hematuria and developed Ca. Cause was and mechanism of immune
asked? deposition was asked.
A. Smoking A. Forms antibodies against tissues
B. Nitrose amines B. Complexes form in blood and get deposited
C. Vinyl Ans: A
Ans: A 148. Innervation of adrenal medulla is by
138.A woman with hx of allergic respiratory A. Preganglionic great thoracic splanchnic nerve
symptoms but feels better when on B. Preganglionic lesser thoracic splanchnic nerve
vacations, what is important point in history Ans: A
pointing to dx? 149. Sodium retention is seen in
Ans: She keeps parrots in her basement A. Heart failure
B. Vasopressin
139.50 years lady received a renal transplant C. Addison’s disease
from a cadaver. One month later renal failure Ans: A
s/s were seem. some biopsy findings 150. Severe form of steatorrhea seen in
pointing to HLA1 association, which cells A. Pancreatic resection
are involved B. Gastric resection
A. CD4 Ans: A
B. CD8 151. Woman is on parenteral
C. Macrophages nutrition. She is most likely to
Ans: B develop which complication?
140. Cholangio hepatitis organism Ans: Hyperglycemia
Ans: Clonorchis Sinensis 152. Nonbacterial endocarditis is seen in
141 Primordial germ cells derived from A. Neoplasms
A. Endoderm B. SLE
B. Ectoderm C. Vegetations of mitral valves
C. Mesoderm D. Rheumatoid arthritis
D. Neural crest cells E. IHD
Ans: B Ans: A
142. Regarding pacinian corpuscles 153. Repeated scenario. A woman with 10
A. Detect high frequency vibration days hx of fever, continuous chest pain
B. Detect low frequency vibration radiating to back on auscultation rustling
Ans: A sound is heard. Diagnosis?
143. Steady pressure is detected by A. Myocarditis
A. Merkel disc B. Pericarditis
B. Ruffini’s Ans: B
C. Meissener’s 154. Infection anterior to pre tracheal fascia is
D. Pacinian most likely to spread to
Ans: B A. Anterior mediastinum
144. Regarding coronary blood flow B. Superior mediastinum
A. Increases with inc in heart rate C. Inferior mediastinum
B. Decreases with inc. heart rate D. Posterior mediastinum
C. Increases in systole Ans: A
Ans. B 155. S2, S3, S4 injury
145. Which drug is removed by active A. Rectal incontinence
tubular secretion? B. Painless labor
A. Streptomycin C. Anorgasmia
B. Mannitol Ans. A
C. Benzillin/ Benzathine penicillin 156. Blood flow to tissues is maintained by
D. Ethyl alcohol A. Ventricular contraction
Ans: A B. Pulse pressure
146. Which of the following exerts its effect on C. Systolic pressure
kidneys D. Mean arterial pressure
A. Clonidine Ans. D
B. Bumetenide 157. Best investigation for pneumonia?
Ans: B A. Sputum for AFB
147. Typical picture of B. Sputum culture
rheumatic heart disease C. Blood culture
Radiant Notes—FCPS PEARLS Golden File 4 By: Dr. Rafi Ullah
D. Spirometry C. Requires carrier proteins
Ans: B Ans: B
158. A patient has developed 168 A woman with jaundice for 1 year. Total
bitemporal hemianopia. bilirubin is 6. Conjugated Bilirubin is 1
Which structure is (No other values were given) She has splenomegaly.
compressing on optic What is the cause?
chiasma? A. Hemolytic anemia
A. Pituitary gland B. Hemochromatosis
B. Hypothalamus C. Cholestatic jaundice
Ans: A D. Hepatocellular jaundice
159. Scenario of a woman with stony dull E. Obstructive jaundice
sound on auscultation. Long statements Ans: A
in options were given about what is true. 169. A boy fell from a bike, and landed on
Ans: It occurs in pleural effusion his shoulder. He had his arm
160. Man on ventilator what will happen if you hanging by the side, and loss of
give positive expiratory force innervation of lateral forearm. Which
A. Alveolar damage nerve is likely to be damaged?
B. Gas redistribution A. Lateral pectoral nerve
C. Decrease FRC B. Medial pectoral nerve
Ans. C C. Axillary nerve
161. In the mechanism of D. Nerve roots C5, C6
thrombus formation most Ans. D
important mechanism is 170. A man was given TCA for trigeminal
A. Platelet aggregation neuralgia, when will the analgesic effect of
B. Platelet adhere to the site TCA start?
C. Prostacyclin’s against platelet aggregation A. After 6 weeks
Ans: C B. 3 months
162. Auriculotemporal nerve accompanies C. 3-4 weeks
which artery? Ans: C
Ans: Superficial temporal artery 171. TCA overdose causes=
163. Metabolic action of thyroid hormone A. Tonic clonic seizures
at normal physiological levels. B. Pinpointed pupils
A. Increases protein synthesis Ans: A
B. Decreases gluconeogenesis 172. Which of the following about lymph flow is
C. Decreased fatty acid synthesis correct=
Ans: A A. Increases by massage
164. What will happen in a man who B. Decreased by incompetent calf muscles
received 2L N/S infusion? Ans: A
A. Increased blood volume 173. Fibrin degradation product by its natural
B. Decrease in urine osmolarity anticoagulant??
Ans: A A. Fibrin polymerization of monomer
165. Regarding C2 vertebrae which is true B. Interfere with calcium
A. Contains odontoid process Ans. B (not sure, something missing)
B. Heart shaped body 174. A woman with pro myelocytic leukemia
C. Contains spine presented with (Picture of Councilman
Ans: A Bodies was given) Eosinophilic
166. Regarding adrenergic receptors cytoplasm etc. Mechanism involved was
A. Parasympathetic postganglionic asked
B. Sympathetic postganglionic to SA node A. Apoptosis
C. Postganglionic to sweat glands B. Karyolysis
D. Preganglionic to sweat glands Ans. A
E. Parasympathetic post ganglionic to skeletal 175. A boy feels pain from pleura on his
muscles shoulder. Nerve involved
Ans. B a. C2, 3, 4
167. Regarding movement of substance b. C3, 4, 5
c. C5,C6,C7
from high to low concentration Ans: B
movement?
176. In diabetic nephropathy, what will be
A. Requires ATP
indicate that nephropathy has occurred?
B. Does not require ATP
Radiant Notes—FCPS PEARLS Golden File 4 By: Dr. Rafi Ullah
Ans: Microalbuminurea C. O to AB
177. Best way to measure high protein level in D. A to B
plasma E. A to O
A. Urea Ans: D
B. Urea and creatinine 188. Characteristic of carcinoma
C. Urinary nitrogen content A. Keratin
Ans. C B. Vesmin
178. Regarding hyperplasia C. Desmin
A. Increase in cell no without increase in size Ans: A
B. Tissues which have potential of mitosis 189. Again a question on Niacin deficiency
Ans: B Ans: Pellagra
179. Nerve taking afferents from 190. Regarding Anti thrombin III
baroreceptors Ans: Heparin exerts its actions by anti-thrombin III
A. Glossopharyngeal
B. Facial Surgery and Allied 1st March 2017
C. Trigeminal
Ans: A Evening
180. Regarding exocrine pancreatic Q.1 micturition reflex
secretions a. self-regenerative
A. It contains more Cl than plasma b. supplied by sacral
B. Secretion increased by CCK Ans: B >A (Ref Jaypee pg. 343)
C. Secretion increased by gastrin Q.2 min pressure in aorta
Ans: B Ans. Iso vol contraction
181. Child is born with a renal Q.3 right bronchopulmonary segments
abnormality, his mother was Ans. 3, 2, 5
taking which drug during her Q...4 Pt presented with raised skin thickening on
pregnancy? cheek which was premalignant
Ans: Captopril a. Actinic keratosis
182. A man with COPD suddenly developed b. intradermal nevus
dyspnea and tachycardia. Which of the Ans: A
following has occurred? Q.5 Anemic hypoxia
A. Air embolism
a. Methemoglobinemia
B. Pulmonary embolism
Ans: B b. Cyanide poisoning
183. A scenario on respiratory alkalosis. Ans: A
Renal compensation was asked. Q.6 multiple chest abscesses which is most
A. Kidneys will secrete HCO3 common organism?
B. Kidneys will produce more NH3 a. Klebsiella
Ans: A b. Staph aureus
184. Which of the following is a chemical c. Streptococci
carcinogen Ans: B
A. Ethyl alcohol
Q.7 pt pain on walking in calf and relived by rest due
B. Benzidine
to
Ans: B
a. popliteal artery
185. Regarding max. Determinant of CSF
b. Post tibial
composition is
A. Choroid cells c. Ant tibial
B. Ependymal cells Ans: A
C. Arachnoid granulations Q.8 how to develop doctor patient mutual.
Ans: B Ans: Dignity maintain
186. MOA of Vancomycin Q.9 hyaline cartilage
A. Acts on human cells but not bacterial cells a. Collagen fibers invisible
B. Converts DHF to THF b. Visible
C. Causes change in bacterial component Ans: A
Ans: C
Q.10 Spinal cord ends at
187. Most severe blood reaction is seen in
Ans: L1
A. A to AB
B. B to AB Q.11 sever chest pain. Pt died. On autopsy
dissecting aorta. What is cause?
Radiant Notes—FCPS PEARLS Golden File 4 By: Dr. Rafi Ullah
a. medial necrosis Q.24 From foramen caecum to midline neck
b. arthrosclerosis diverticulum remnant of which forms thyroglossal
Ans: A cyst
a. thyroglossal diverticulum
Q.12 Muscle important in unlocking of knee joint
b. thyroglossal duct
a. Popliteus
Ans: A
b. Rectus femoris
Ans: A Q.25 Femoral artery
a. Medial to femoral nerve in femoral triangle
Q.13 interstitial fluid measured by
a. Mannitol D20 b. Leaves through adductor longus
c. lies posterior to midinguinal point
b. Evans blue inulin
Ans: A
c. Evan heavy water
d. Mannitol /inulin Q.26 Ulnar nerve supply:
Ans: Adductor pollicis muscle
Ans: B (interstitial fluid is measured as ECF minus
plasma volume, For ECF inulin while for plasma Q.27 Osmotic diuretics act on
a. PCT
Evans blue is used)
b. DCT
Q.14 carotid sinus receptors
c. Collecting duct
a. act as chemoreceptors
Ans: A
b. stimulated by change in BP
Ans: B Q.28 Projection fibers
a. Internal capsule
Q.15 Accessory rib
Ans: Compression of subclavian vessels n brachial b. Corpus callosum
plexus c. Fornix
Q.16 Erythrocytosis secondary to erythropoietin... Ans: A
a. Renal adenoma Q.29 Cushing syndrome
b. Transitional a. Leukocytosis
Ans: A b. Eosinopenia
Q.17 Most common fracture of clavicle c. Neutropenia
a. lateral 1/3 Ans: B
b. medial 2/3 Q.30 .Tactile signals from finger tips
Ans: A a. Merkel
Q.18 Tumor invading to major vessels b. Meissner
Ans: clear cell carcinoma c. Ruffini
Q.19 Splenic vessel in which Ligament d. Free nerve ending
Ans: Lienorenal Ans: B
Q.20 sphincter urethrae is supplied by Q.31. walking barefoot. Feet on pointed thing.
a. Ilioinguinal Pointed object touches the foot. Reflex is initiated by
b. Pelvis splanc nerve a. pacinian
c. Sacral splanchnic nerves b. Meissner
d. Autonomic nerves c. ruffini
e. Internal pudendal d. free nerve ending
Ans: E Ans: D
Q.21 a mass in mid line, biopsy done, normal thyroid Q.32. A child with exchange transfusion After 10
tissue, what will be seen on histology. days develop palmar erythema spreads to rest of the
Ans: Cuboidal follicular body ALT and bilirubin raised and diarrhea
Q.22. appendicular artery is branch of a. Graft versus host
a. ileocolic b. Delayed transfusion reaction
b. post cecal Ans: A
Ans: A Q.33 Excessive ADH produced by some tumor
Q.23 Massive hyperkalemia a. Hypo osmotic over hydration
a. heavy exercise b. Hyperosmotic over hydration
b. Chronic diarrhea Ans: A
Ans: A Q.34 Pain and warmth by
a. C-fibers
Radiant Notes—FCPS PEARLS Golden File 4 By: Dr. Rafi Ullah
b. Delta-A Ans: osteoporosis and prone to bone fracture.
c. beta-fibers Q.47 So there was this question about chances of
Ans: A what is more 5 years after transplant.
Ans: Lymphoproliferative malignancy
Q.35 Collagen fibers regularly arranged in
There was no skin option
a. Dermis
Q.48 prosthetic valve. Discharge on medication.
b. Epidermis
Come in ER with severe epistaxis Hb 6 TLC.....
c. Reticular layer Platelet 75000 cause of bleeding trauma
Ans: C a. thrombocytopenia
Q.36 Sodium b. drug induced
a. Regulated by ADH Ans: B
b. Primary active transport in PCT Q.49 turner karyotyping
c. Primary active transport in loop of Henle a. XO
Ans: A b. XX
Q.37 Anterior cardiac vein drains into c. XY
a. Coronary sinus Ans: A
b. Right atrium Q.50 most aggressive tumor
c. Great cardiac vein Ans: melanoma
d. Small cardiac vein Q.51 breath sounds on post thorax sitting position
Ans: B till
Q.38 at apex, in sulcus, which vein starts a. 8rib
a. Great cardiac b. 10th
b. Small cardiac c. 12
c. Coronary sinus Ans: A
Ans: A Q. 52 source of carbohydrate for diabetic pt.?????
Q.39 Ambiguous genitalia having 46xx.. Diagnosis? a. whole wheat
a. Congenital adrenal hyperplasia b. bread
b. Mixed gonadal c. Rice
Ans: A Ans: A
Q.40. Blood flow is inversely related to Q.53 Vit D richest source?
Ans: Resistance a. butter
Q.41 Energy expenditure in inspiration b. milk
a. 5% c. fish
b. 10% d. Cod liver oil
c. 15% Ans: D
Ans: A Q.54 Pt deranged aPTT normal
Q.42 chronic smoker. Suffered from emphysema a. factor 7
a. inc alveolar space inc arterial O2 b. factor 12
b. Dec alveolar space inc arterial O2 c. antithrombin
c. Dec alveolar space Dec O2 Ans: A
d. Space increase O2 decrease Q.55 pain epigastrium and back. And on Rt shoulder
Ans: D on eating fatty food. What is origin of pain?
Q.43 pH 7.52 Hco3 30 pco2 40 Ans: gall bladder
Ans. Uncompensated metabolic alkalosis Q.56 Edema caused by
Q.44 gastric lymphoma associated with a. lymphadenitis
Ans: h. pylori b. Dec interstitial osmotic
Q.45 human placenta has covered with amnion on Ans: A
its fetal side
a. Chorioallantois Placenta Q.57. IVC blocked blood above the origin of
b. Get Separated Along The Stratum Spongiosum azygous will be shunted to
c. Get Separated When There Is Rupture Of a. Portal vein
Many uterine arteries b. Left gastric
Ans: C Ans: B
Q.46 prolong use of steroid
Radiant Notes—FCPS PEARLS Golden File 4 By: Dr. Rafi Ullah
Q.58 At what plasma glucose conc. Glucose appears Ans: A
in urine Q.73 Lactotrophs inhibited by
a. 180 a. Dopamine
b. 200 b. GnRH
c. 250 Ans: A
Ans: C Q.74 Gallbladder differentiated from intestine
Ans: Muscularis mucosa
Q.59. Malignancy features
Q.75 Part of bone formed from secondary
a. Invasion
ossification
b. Pleomorphism
c. Inc N/c ratio a. Epiphysis
Ans: A b. Diaphysis
Q.60 Clonorchis Sinensis causes c. Epiphyseal plate
Ans: Cholangio carcinoma d. Metaphysis
Q.61. for Syphilis sample taken from Ans: A
Ans: genital sores Q.76 Hartman pouch
Q.62 Edema, protein 6 g. Defect in a. Part of cystic duct....
Ans: basement membrane b. Only present in pathological condition....
Q.63 Councilman Bodies c. Common site of obstruction of gall bladder....
Ans: yellow fever (found in the liver of individuals d. formed in calot triangle
suffering from viral hepatitis (acute), yellow fever, or Ans: C
other viral syndromes. It represents a hepatocyte that is Q.77 which one is not involved in normal regulation
undergoing necrosis/apoptosis) of body temp?
Q.65 High grade fever chills For 4 days black color a. Hyperventilation
urine on anti-malarial b. Sweating
a. G6pd Ans: A
b. Falciparum Q.78. 6 hrs after MI. pt developd cyanosis bcz of
Ans: A poor ventilation improper
Q.66 Taenia solium a. V/Q
Ans: Uncooked meat b. right to left shunting
Q.67 Middle meningeal in the c. slow cardiac
Ans: Foramen Spinosum Ans: A
Q.68 Structure post to ureter 79. Guitarist with loss of sensation in lateral palmer
a. genitofemoral aspect of palm
Ans: Median
b. Iliac vessels 80. Child with coronary artery aneurysm Ans:
Ans: B Kawasaki
Q.69 Ataxia, diplopia and nystagmus, which artery 81...Body 1st line of defense against microbes
block Ans: Skin
a. PCA 82. First line of defense in tissues
b. Basilar a. Macrophages
c. Vertebral b. Neutrophils
Ans: B Ans: B (although few friends saying A giving
Q.70 Primary Center of Ossification of Long bone at explanation as 1st response in tissue is macrophage
Birth? while in blood its neutrophils, i haven’t found any
a. Lower end of Femur authentic ref for it, neither they have. Just blindly
following group, for me whether its cellular or tissue
b. Lower End of Humerus
level 1st response is neutrophils)
c. Upper End of Fibula
83. Floor of sub occipital triangle???
d. Upper End of Tibia
Ans: A
a. post auricular artery
Q.71 Winged scapula b. occipital artery
Ans: Long thoracic nerve c. vertebral artery
Q.72 Attachment of radius with Ans: C (if the question changed its stems, remember
it contains Third part of vertebral artery, suboccipital
a. Lunate nerve and Suboccipital venous plexus)
b. Trapezium 84. Monitoring of oral anticoagulant by?
c. Hamate a. Pt
Radiant Notes—FCPS PEARLS Golden File 4 By: Dr. Rafi Ullah
b. Aptt 94. Female with breast Ca and axillary nodes
Ans: A enlargmnt. On histopathology of node. no
85. Factor 8 produced by metastasis. Cause of lymph node enlargement?
a. Hepatocytes a. Sinus histocytosis
b. Endothelial cells b. granulomatous change
Ans: B c. lymph obstruction
86. Brodie abscess d. Infectious lymphadenitis
a. Pyogenic osteomyelitis e. para cortical hyperplasia
b. Tb osteomyelitis Ans: A
Ans: A 95. Rt testicular tumor lymph drains into
87. Child dyspnea, pseudo membrane cause is Ans: paraortic
a. Exotoxin of diphtheria 96. Rt 12th nerve damage causes
b. Fibrinous exudate a. Rt half paralysis with atrophy of Rt side
c. Endotoxin. Of diphtheria b. Rt half paralysis without atrophy
Ans: A c. Lt side paralysis with atrophy
88. Hormone sensitive lipase inhibited by d. Lt side paralysis without atrophy
a. Cortisol Ans: A
b. Thyroxine 97. Tidal vol 350 ml dead space 100 mil RR 18
c. Insulin calculate alveolar ventilation rate
Ans: C a. 4 L
89. Liver resected in a 45 yrs. female. For regrowth b. 4.5 L
of remaining hepatocytes into full sized liver what is c. 4.8 L
required Ans: B
a. TNF 98. Aqueous diffusion
b. EGF a. Phenobarbitone
c. PGE b. Diazepam
d. steroid Ans: B (ref: pharmacology and pharmacokinetics by
e. hepatocyte growth factor mark tomlin--page 14--depends on log P value ,
Ans: E greater the value greater the absorption – P value for
90. Asthmatic patient, post-operative pain killer diazepam is 2.7 while for Phenobarbitone is 2.0)
a. Pethidine 99. Scenario of pan eating
b. Diclofenac Ans. Submucosal fibrosis
c. Ketorolac 100. Disease vs. non- disease
Ans: C Ans. case control
91. First response against tumor 101. 2x2
a. Apoptosis Ans. Chi square
b. NK cells 102. How to convince society
c. Macrophages Ans: Education of basic health
Ans: B 103. Axillary sheath formed
92. Transplantation kidney 6 months renal functions Ans. Prevertebral
deteriorated. But after immunosuppression kidney 104. Bundle of His
function become normal. What is cause of a. RCA
deterioration?
b. LAD
a. acute cell mediated Ans: A
b. acute humoral rejection 105. At birth Umbilical cord contains :
c. chronic rejection a. 2 Umbilical arteries
d. toxicity of Cyclosporin b. 2 umbilical veins
Ans: A Ans: A
93. Forearm supination lost 106. which artery connects Basilar with ICA ?
a. Radial and musculoskeletal a. post communicating
b. Radial and ulnar b. ant communicating
c. Ulnar and median Ans: A
d. Median and ant introcus
107. Potent chemotactic
e. Radial and ant intrrocus
Ans: C5a
Ans: E (post compartment of the forearm is
responsible for supination supplied by radial nerve) 108. Alcohol detoxified by
a. Peroxisome
Radiant Notes—FCPS PEARLS Golden File 4 By: Dr. Rafi Ullah
b. SER Ans: C (Direction of the sound is detected by the
c. RER superior Olivary nucleus, the auditory cortex is
Ans: A essential for perception of direction of sound)
109. Farmer round mass 123. Endometrial hyperplasia cause was asked
Ans: aspergillosis Ans. Estrogen prolonged use
110. Dye normal on one side just little on other side 124. Bulbar urethra rupture urine will go (sup
near bladder perineal not in options)
a. pelvic kidney a. scrotum
b. Unilateral agenesis b. thigh
Ans: A c. Ischiorectal fossa
111 Scenario joint abdomen pain anemia d. Anal triangle
Ans. HB-SS e. Deep pouch
112 A new born with blood Group B+ve is Ans: A
having Erythroblastosis Fetalis which is most 125. A person with occipital severe headache came
Appropriate line of treatment? to u . . . Kafi lamba scenario tha then asked u
a. Exchange transfusion with B-ive blood prescribe some drug now he came again with
b. Transfusion with type O blood dyspnea. Drug was
Ans: A a. Propanalol
113. A woman required blood transfusion her red b. methyl Dopa
cells are agglutinated by antisera anti-A and anti-D Ans: A
while her serum is agglutinating B cells suitable 126. tumor Reversible change in size shape
blood donor? dyspolarity?
a. A negative a. Dysplasia
b. A positive b. metaplasia
c. AB positive c. Anaplasia
d. O negative Ans: A
e. O positive 127. Cervix with squamous epithelium in Endocervix
Ans: B it is?
114. Heterochromatin a. CIN
a. inactive b. Dysplasia
b. Synthesize rRNA c. Metaplasia
Ans: A d. Hyperplasia
115 VIT A deficiency e. Invasive carcinoma
Ans: Night blindness Ans: C
117.Regarding Rectum 128. Following injury, Loss of opposition of thumb.
Ans: Posteriorly related to 3, 4, 5 sacral vertebrae Which nerve involved?
118.Rectum also supplied by Ans: Median nerve
Ans: Median Sacral 129. Long scenario about patient can’t hold bottles
118. Pulse pressure increased by mixed sensation at thumb... Loss of flexion at distal
a. maximum resistance in arteries then aorta phalanx of thumb
b. Inc compliance of veins a. Median nerve
Ans: A b. Radial
c. Ulnar
119. 2 year old boy Macrocytic
Ans: A
a. Vit b 12
b. Intrinsic factor antibodies 130. A young male has petechial pallor lethargy.
Ans: B Definitive diagnostic test
a. CBC
119.Thyroid enclosed in
b. Serology
Ans: pretracheal
c. Molecular studies
121. Tall man normal ejaculation but azospermia d. Bone marrow aspiration
cause lie in Ans: D
Ans: Sertoli cells 131. Anaerobic abscess after Abd surgery by
122. Which of following detect Direction of sound Ans: Bacteroides fragilis
a. Auditory cortex 132. Mainly in abdominal and Gynae surgeries
b. Cochlea. a.Bacteroides
c. Sup Olivary
b. E.coli
d. medial geniculate
Ans: A
133. Medullary thyroid
Radiant Notes—FCPS PEARLS Golden File 4 By: Dr. Rafi Ullah
a. Inc calcitonin b. venular capillary
b. Inc Ca inc Phosphate Ans: A
Ans: A 149. Sensitive for SLE
134. Medial longitudinal arch pillar by? a. ANA
a. Talus b. Anti-Ds Dna
b. Calcaneus Ans: A
c. Navicular 150. Streptococcal pneumonia
d. Cuneiform Ans: Benzathine penicillin
e. Cuboid 151... Diabetic 50 plus comes in coma Rx is
Ans: A Ans. Reg insulin
135. After marginal branch Blocked affected will be 152.5th month of pregnancy with thyrotoxicosis
Ans. AV node drug of choice.
136. Parasympathetic a. PTU
Ans. causes Dec diameter of pupil b. Methimazole
137. Adrenergic Ans: B
Ans. Inc Glycogenolysis 153 During quiet inspiration ant. Post and
138. T8 proprioception loss T9 temp pain loss. (No transverse diameter Inc mainly by
option of T8 Hemisection) a. Diaphragm
Ans. T11 (all options were wrong 100%) b. Ext intercostal
139. Smooth muscle Ans: A
a. Cardiac is specialized smooth muscle 154. 59 year known Hypertensive on medication
b. Slow spikes came in comatose condition cause is
c. Inner circular outer longitudinal a. Acute renal failure
Ans: C (Ref: BRS Smooth muscle fibers in the b. Chronic renal failure
intestine are arranged in two layers; Inner circular c. Diuretics
layer and Outer longitudinal layer) Ans: A
140. PNS is 157. Patient on warfarin Monitored by Ans. PT
Ans. Spinal nerves plus autonomic ganglia 155. Pudendal nerve
141. 1st meiotic completes? Ans. On lateral side of pudendal canal
a. just before puberty 156. Post triangle injury of neck damages which
b. At 3rd month structure
Ans: A (if before ovulation in options is present Ans: Spinal accessory
prefer that) 157. SA node
142. Body motor image at Ans: generate impulses spontaneously
a. Premotor 158. Pseudomonas long scenario cause Ans:
b. Primary motor endotoxin
Ans: A 159. Hemolytic anemia associated with
143. Respiratory alkalosis a. unconjugated
a. Salicylate poisoning b. kernicterus
b. High altitude Ans: A
Ans: A (salicylates causes’ respiratory alkalosis and 160. gall bladder differentiated from colon
metabolic acidosis) Ans: muscularis mucosa
144. Ventricles are completely depolarized at 161. Women, hemorrhage, which part affected
a. QRS complex Ans: PCT
b. St segment 162. GFR best measured
Ans: B Ans: inulin
145. During shock most important 163. Sucralfate with cimetidine
a. Adrenosympth system a. Dec metabolism
b. CNS ischemic response b. Suralfate decreases cemitidine absorbtion, give
Ans: B it after two hrs
145. Child got anemia and other brother has Ans: B
history of transfusion 164. A new born with jaundice have to study upon
Ans. Thalassemia his liver...dye should b given through his
147. Anemia ALT AST were given almost normal ALP Ans: ligamentum teres
slightly raised 165. Myxoid degeneration is mostly associated with
Ans: Stones in gall bladder a. carcinoid valve disease
148. Pressure maximally dissipated at b. infection endocarditis
a. Arterial capillary c. Libman endocarditis
Radiant Notes—FCPS PEARLS Golden File 4 By: Dr. Rafi Ullah
d. marantic endocarditis B) Drained by independent vein
e. mitral valve prolapsed C) Supplied by independent somatic nerve.
Ans: E Ans: A
167. Siblings have also bleeding problem bleeding 6) PT unable to extend hic MCP joint, abducts &
from umbilical stump best test extends his thumb but with intact
a. PT sensations, Nerve damaged is ,
b. VWD A) Ulnar nerve at wrist
c. APTT B) Ulnar nerve at elbow
Ans: C C) Post interosseous nerve
168. 1st pass metabolism maximum D) Radial nerve above elbow
Ans: oral route E) Median nerve
169. Intrinsic factor released from Ans: C
Ans: fundus of stomach 7) PT lost eversion of foot, muscle
paralyzed???
170. A boy has small upper and lower limb, normal A) Tibialis posterior
trunk pattern is B) Tibialis anterior & posterior
Ans: autosomal dominant C) Peroneus longus
171. Medullary thyroid Ca? D) Peroneus Tertius
a. inc level of calcitonin Ans: C
b. 24,25 OH –Cholecalciferol 8) Joint between intervertebral disc
Ans: A A) Syndesmosis
172. ADH acts on?????? B) Fibrous joint
a. collecting duct C) Symphysis
b. cortical collecting duct D) Gomphosis
c. DCT E) Synchondrosis
Ans: A Ans: C
Surgery and Allied 2nd March 2017 9) Basilar artery divides into
A) Two middle cerebral artery
Morning B) Two posterior cerebral artery
1) Which of the following is Mediator of C) Anterior inferior cerebral artery
systemic inflammation. D) Posterior community artery
A) IL1 E) Anterior communicating artery
B) IL2 Ans: B
C) IFN Gamma 10) Types of Hypersensitivity in myasthenia
Ans: A gravis
2) Which of the following would be common A) Type 1 HS
cause of Megaloblastic Anemia in 35 years B) Type 2 HS
old male. C) Type 3 HS
A) Dietary Insufficiency D) Type 4 HS
B) Intrinsic factor antibody Ans: B
C) Deficiency of Transcobalamin 2. 11) Organism most commonly involved in
Ans: B gynecological procedures
3) During development, T lymphocyte are A) E coli
protected from autoimmune destruction by. B) S aureus
A) Blood thymus barrier C) Bacteroids
B) Protected by hassals corpuscles. D) Pseudomonas
Ans: A Ans: C
4) What is clearance of substance when its 12) Drug of choice for gas gangrene
concentration in plasma is 10 mg/dl, its A) Ceftriaxone
concentration in urine is 100 mg/dl, and urine B) Ciprofloxacin
flow is 2 ml/min? C) Penicillin G
A) 2 ml/min D) Vancomycin
B) 20 ml/min Ans: C
C) 100 ml/min 13) PT known HIV is suffering from severe
D) 200 ml/min headache , photophobia and signs of
Ans: B meningeal irritation, on microscopy
5) Regarding Broncho pulmonary segment true organism has halo around it , the organism
is is
A) Supplied by tertiary Bronchiole A) Toxoplasmosis
Radiant Notes—FCPS PEARLS Golden File 4 By: Dr. Rafi Ullah
B) Histoplasmosis Ans: C (This is called obligatory water loss must be
C) Cryptococcus between minimum 400-600ml/day)
D) Candida Ref: Concepts in Medical Physiology by Julian,
E) Cytomegalovirus page: 362
Ans: C 22) A PT has fracture clavicle of junction of outer
14) A man from Sibi , had fever with chills for & inner 2/3rd. which of following muscle is
last 6 month , he suddenly died on autopsy responsible for elevating medial end of
blood clots were present in peritoneum, clavicle.
diagnosis is A) Trapezius
A) Malaria B) Deltoid
B) Kala Azar C) Sternocleidomastoid
C) G6PD def D) Subclavius
D) Sickle cell anemia Ans: C
Ans: B 23) A PT known case of tabes dorsalis having
15) Patent ductus arteriosus is derivative of some urinary bladder problem. Which of
A) 4th aortic arch following pathology of bladder occurs in
B) Left 6th aortic arch tabes dorsalis.
C) 5th artic arch A) Atonic bladder
Ans: B B) Spastic bladder
16) Lymph drainage of Ca cervix C) Automatic bladder
A) Internal iliac nodes Ans: A
B) Inguinal lymph nodes 24) A PT having enlarged Jugulodiagastric
C) External iliac nodes nodes. Which of following is draining.
D) Para aortic nodes A) Pharynx
Ans: A B) Post 1/3rd of tongue
17) True about mucus secreting cells C) Palatine tonsils
A) Have nucleus located at base D) Larynx
B) Have abundant matrix Ans: C
C) Cells are columnar in shape 25) Lymph from medial and inner aspect of
Can’t recall other options mammary gland into all of following except.
Ans: A A) Pectoral group of modes
18) Mechanism of fever in infection B) Internal thoracic nodes
A) Direct action of cytochromes on C) Communicates e that of opposite breast
hypothalamus D) Inferior phrenic nodes
B) Through activation of complement system Ans: A
Can’t recall other options  Lymph drainage from lateral quadrant:
Ans: B o The majority of lymph (>75%), particularly from
19) A 5 years old child presented with unilateral the lateral quadrants, drains to the axillary
cryptorchidism and testis is present outside (pectoral) lymph nodes.
scrotum within inguinal canal. Which of the  Lymph drainage from Medial quadrant:
following is more likely to occur in this child
o Parasternal nodes/internal thoracic nodes
A) Infection
B) Malignancy o Lymph from medial quadrant may also drains
C) Infertility into the opposite breast.
D) Trauma  Lymph drainage from inferior Quadrant
Ans: B o Inferior phrenic lymph nodes.
20) A pt having nasal polyps , which of following 26) Relation of otic ganglion
is most likely A) Lies above foramen ovale
A) Eosinophilia B) Lies below foramen ovale
B) Ciliated columnar epithelium C) Lies with in coverings of brain
C) Lymphocytes Ans: B
D) Squamous metaplasia 27) A pt having injury to spinal cord above
Ans: D sacral segment. Which pathology will be
21) Minimal amount of urine required to remove present in bladder
metabolic wastes from body is A) Spastic bladder
A) 200—400 ml B) Atonic bladder
B) 100—200ml C) Neurogenic bladder
C) 500—600ml Ans: A
D) 1000—1200ml
Radiant Notes—FCPS PEARLS Golden File 4 By: Dr. Rafi Ullah
28) If mass movements of intestine is affected, A) T1 cells
pathology most likely in B) T2 cells
A) Myenteric C) Macrophages
B) Auerbach D) Neutrophils
Ans: B Ans: A
29) Tickle sensation are carried by 38) IL2 is produced by which of following.
A) A delta fibers A) T1 cells
B) A beta fibers B) T2 cells
C) C fibers C) Macrophages
D) B fibers D) Neutrophils
Ans: C (ref: P. P. Newman neurophysiology pg: 114) Ans: A
30) Common b/w facilitated diffusion & co- 39) Sensation ( sterogonosis) from lips carried
transport by
A) Both use ATP A) A beta
B) Both occur downhill B) A delta
C) Both uses carriers proteins C) C fibers
D) Both are for inorganic transport D) B fibers
Ans: C Ans: A
31) For iron def anemia , best test is 40) Which of following increase gastric secretion
A) Serum iron A) Ingestion of protein
B) Serum iron + ↑ TIBC B) Increase in gastric acidity
C) Serum ferritin C) Secretin secretion
D) Serum ferritin + ↑ TIBC Ans: A
Ans: D 41) Regarding pacinian corpuscles which of
32) A pt having normocytic, normochromic following true
anemia. which of following is most likely A) Tonic
A) Vit B12 def B) Phasic
B) Iron def C) Un-encapsulated
C) Acute blood loss Ans: B
D) Chronic blood loss 42) Regarding pituitary gland true is
Ans: C A) Lies above diaphragm sellae
33) Hyaline cartilage is present in B) Lies outside dura matter
A) Epiglottis C) Lies medial to cavernous sinus
B) Ear pinna Ans: C
C) Larynx 43) Cells with large granules and larvicidal
D) Temporomandibular joint A) Basophils
Ans: C B) Eosinophils
34) A newborn ë recurrent chest infections. C) Monocytes/ Macrophages
Which of following is most likely deficient. D) Neutrophils
A) Ig A Ans: B
B) Ig D 44) Anti-oxidant function of vitamin A.
C) Ig E A) Protection of germinal epithelium
D) Ig G B) Protection of endothelium of skin
Ans: A C) Protection of blood vessels
35) Regarding Fat embolism which is most likely Ans: A
A) Occur usually 72-96 hours often trauma 45) Which of following has longest incubation
B) Usually reversible with meticulous period
medical therapy A) Measles
C) Occurs usually due to fracture of long B) Chicken pox
bones C) Small pox
D) Cannot occurs enzymatically D) Rubella
Ans: C E) Influenza
36) Radiation usually causes malignancy after Ans: D
A) 1 year 46) Which of following is thermo genic hormone
B) 5 years A) Growth hormone
C) 10 years B) Estrogen
D) 20 years C) Progesterone
Ans: C D) Cortisol
37) TNF ᾀ is produced by which of following. Ans: C
Radiant Notes—FCPS PEARLS Golden File 4 By: Dr. Rafi Ullah
47) Which of following factor deficiency is 55) To assess nutrition deficiency what will you
associated with clotting than bleeding check?
A) Factor V A) Protein content
B) Factor ẊII B) Carbohydrates
C) Factor Ẋ C) Lipids
D) Factor VII D) Essential minerals
Ans: A E) Vitamins
48) A mechanic during work crushed his little Ans: A
finger at MCP joint which of following muscle 56) Which Hb chain is mainly absent in fetal life.
is not severed A) Alpha chain
A) Palmar interossei B) Beta chain
B) Dorsal interossei C) Gamma chain
C) Lumbricals D) Delta chain
D) Flexor digitorum superficialis Ans: B
E) Flexor digitorum profundus 57) Maternal inheritance occurs.
Ans: D A) X-linked dominant
49) Which of following muscle or muscles B) Mitochondrial DNA
causes adduction at wrist C) X-linked recessive
A) Flexor carpi ulnaris D) Autosomal dominant
B) Flexor & extensor carpi ulnaris E) Autosomal recessive
C) Extensor carpi ulnaris Ans: B
D) Extensor digital minimi
Ans: A 58) A student was studying cell in which he
50) Deficiency of fibrinogen what to give observed an organelle in dividing phase
A) Albumin inside cell. Which of following is most likely.
B) FFP A) Mitochondria
C) Cryoprecipitate B) Centrioles
Ans: C C) Golgi bodies
51) 3rd and 4th ventricle true is. D) RER
A) Flow of CSF is from 4th to 3rd ventricle E) SER
B) Communicates through cerebral Ans: A
aqueduct 59) Loop diuretics acts on.
C) Communicates through foramen of A) Ascending limb of loop of Henle
Monro B) Descending limb
Ans: B C) DCT
52) Mitral valve stenosis. Where can be mitral D) CT
valve best heard? Ans: A
A) Above mitral valve 60) Rt bronchus as compare to left
B) Lt second intercostal space A) More obliquely placed
C) Rt second intercostal space B) Is wider
D) Apex of heart C) Commonly obstructed
Ans: D Ans: C
53) True about external oblique 61) Anterior pituitary tumor will compress.
A) Inc vertical diameter A) Occulomotor nerve
B) Inc AP diameter B) Abducent nerve
C) Lowers rib during expiration C) Optic nerve
Ans: C D) Trochlear nerve
 Remember diaphragm inc vertical diameter E) Trigeminal nerve
 while intercostal (especial external intercostal) inc AP Ans: C
diameter 62) Nucleus ambiguous arises from
A) Pons
Here in the question, external oblique is mentioned not B) Medulla
external intercostal. Which lowers rib during forceful C) Mid brain
expiration. Ans: B
54) True about external oblique 63) Cerebellar lesion characteristic
A) Fibers crosses ribs A) Resting tremor
B) Fibers passes downward & forward B) Dysmetria
C) Fibers passes downward & backward Ans: B
Ans: B 64) Cerebellar lesion occurs all except.
Radiant Notes—FCPS PEARLS Golden File 4 By: Dr. Rafi Ullah
A) Intention tremor 74) A pt is unable to turn his RT eye laterally
B) Dysdiadokokinesia which of following is most likely
C) Swaying to same side of lesion A) Infection in Rt cavernous sinus
D) Paralysis of skeletal muscle B) Infection in Lt cavernous sinus
Ans: D C) Damage to superior oblique
65) A pt having resting tremor, mask face Ans: A
(scenario of Parkinsonism). Where is lesion 75) Pterygopalatine raphe is attached to
A) Caudate nucleus A) buccinators
B) Substantia nigra B) inferior constrictor
C) Lentiform C) middle constrictor
D) Globus pallidus D) masseter
Ans: B Ans: A
66) Relation of uterine artery & ureter. 76) A women of 45 years age presented with
A) Ureter crossed above uterine artery fracture neck of femur. Serum Ca is 12.1 –
B) Uterine artery crosses above ureter (N8-11) what is most likely cause.
Ans: B A) Osteoporosis
67) Regarding drug-drug interaction , correct is B) Vit D deficiency
A) Pharmacokinetics only C) Hyperparathyroidism
B) Pharmacodynamics only D) Pseudo hyperparathyroidism
C) Both pharmacokinetics & Ans: C
pharmacodynamics 77) Pt has MI involving posterior 1/3rd of I/V
Ans: C septum artery involved
68) In case of liver failure which of following A) LCA
enzyme is decreased. B) RCA
A) Acetylcholine C) LCX
B) L-dopa D) LAD
C) Psuedocholine esterase Ans: B
Ans: C 78) There was different scenario but same post
69) Which of following potent anti- oxidant? 1/3rd of IV septum involved& answers was
A) Vit A
B) Vit E Ans: RCA
C) Glutathione 79) A pt presented with pure water loss which of
D) Vit C following fluid suitable for replacement
Ans: C A) 0.9 % N/s
70) Regarding cimetidine which is common B) 5 % D/W
adverse effect which causes it to be stopped C) 10 % D/W
A) Gynecomastia D) Hemaccel
B) CNS side effects Ans: B
C) Inhibition of hepatic enzymes 80) Proto-Oncogen converted to oncogene
Ans: A through
71) Which of following Ca never metastasizes A) Point –mutation
A) Squamous cell Ca B) Gene amplification
B) Malignant melanoma Ans: A
C) Basal cell carcinoma 81) Dynamic support of uterus.
D) Adenocarcinoma A) Pelvic diaphragm
Ans: C B) Broad ligament
72) Which of following cells has activity against C) Urogenital diaphragm
cancer cells & prevents carcinogenesis. D) Transverse cervical ligament
A) T cells ( Helper) Ans: A
B) B cells 82) A pt living river side from 2 years complains
C) NK cells of loss of sight. Which organism is likely to
D) Monocytes / macrophages be involved
E) Cytotoxic T cells A) Trichuris trichuri
Ans: C B) Onchocercosis
73) Supply of blood to parathyroid gland. C) Filariasis
A) Superior thyroid artery D) Ancylostoma duodenale
B) Inferior thyroid artery Ans: B
C) Superior & inferior thyroid artery 83) Vagus nerve action on heart.
Ans: C A) Dec contractility of heart
Radiant Notes—FCPS PEARLS Golden File 4 By: Dr. Rafi Ullah
B) Dec heart rate Ans: A
C) Inc heart rate 94) Confidentially of pt can be breached
Ans: B A) Patient is terminally ill
84) Morphine acts on which receptors B) Patient allows you
A) Strong Meu receptors C) Insurance claims
B) Weak kappa receptors Ans: B
C) Weak Meu receptors 95) First meiotic division in female occurs
Ans: A A) Before ovulation
85) keratin filaments are seen in B) Before fertilization
A) Carcinoma C) 3rd intrauterine month
B) Sarcoma D) 6th intrauterine month
C) Seminoma Ans: A
Ans: A 96) Source of creatinine is
86) True regarding axillary artery. A) Skeletal muscle
A) Begins as continuation of brachial artery B) Heart
B) Begins at lateral border of first rib C) Fats ( adipose tissue)
C) Divides in radical & brachial artery Ans: A
D) Lies in front of pectoralis minor 97) During last trimester separation of mother &
Ans: B fetus
87) Femoral pulse , true is A) Cytotrophoblast & synctio-trophoblast
A) Can be palpated at mid inguinal B) Fetal endothelium & cytotrophoblast
ligament C) Fetal endothelium &synctio-trophoblast
B) Can be palpated at mid inguinal point Ans: C
C) Can be palpated medial to femoral vein 98) If parasympathetic cut which of following will
Ans: B be affected mostly
88) Following is caused by DNA virus A) Sweat glands
A) Mumps B) Salivary glands
B) Measles C) Muscle of GIT
C) Rubella D) Skeletal muscles
D) Infection mononucleosis Ans: C
E) HIV 99) Systemic antifungal not given in
Ans: D A) Fungal meningitis
89) During thyroid surgery nerves injured related B) Widespread fungal infection
to superior thyroid artery C) Fungal sinusitis
A) Recurrent laryngeal nerve D) Oral candidiasis ( thrush)
B) External laryngeal nerve Ans: D
Ans: B 100) Antigen presenting cells in skin &
90) Characteristics of C7 vertebrae thus protecting from pathogens are
A) C7 has no spine A) Melanocytes
B) C7 has longest spine B) Dendrites
C) C7 is a typical vertebrae C) Reticulocytes
Ans: B Ans: B
91) Soldier coming down from height having 101) Rt & Lt kidney differentiated from
peripheral cyanosis & breathlessness. Cause each other
is A) Rt larger than Lt
A) Secondary polycythemia B) Rt smaller than Lt
B) Polycythemia Rubra vera C) Rt higher than Lt
C) Methemoglobulinemia D) Relation of structures at hila
D) Heart failure Ans: D
Ans: A 102) A pt chronic smoker has Hx of
92) Increase intrathoracic pressure causes. coughing now present with altered mental
A) Dec volume of pulse pressure status & difficulty in talking. MRI brain shows
B) Dec negative intrapleural pressure lesion which of following will biopsy of brain
Ans: B lesion show.
93) Proximal part of bile duct supplied by A) Glioblastoma multiforme
A) Cystic artery B) Metastatic carcinoma
B) Hepatic artery C) Meningioma
C) gastroduodenal artery Ans: B
D) Celiac trunk 103) MOA of propylthiouracil
Radiant Notes—FCPS PEARLS Golden File 4 By: Dr. Rafi Ullah
A) Inhibit release of iodine E) Macrocytic normochromic anemia
B) Inhibit action of T3 & T4 peripherally Ans: C
C) Inhibit synthesis of iodine 112) Regarding pheochromocytoma which
Ans: A of following correct.
104) Which of following associated with A) Associated with men I syndrome
lymph node B) Usually intra-abdominal in 95% of
A) Cystic hygroma cases.
105) A pt having pain RIF. appendectomy C) Usually smaller than 5 mm in diameter
done which blood picture most likely Ans: B
A) Eosinophilia 113) Reverse T3 is present in abundance
B) Basophilia in body
C) Neutrophilic Leukocytosis A) Fetus
D) Lymphocytosis B) Pregnancy
E) Neutropenia C) Adult
Ans: C D) Puberty
106) Tumor associated with gene Ans: A
amplification 114) Ciliary movements are associated
A) Pheochromocytoma with
B) Neuroblastoma A) Microtubules and dynein
C) Retinoblastoma B) Intermediate filaments and kinesin
D) Medulloblastoma C) Actin/myosin
Ans: B Ans: A
107) A pt after walking 100 feet has pain in 115) Metabolic acidosis e normal anion
calf muscles which is relieved by resting. gap.
Artery most likely block. A) Neuropsychosis
A) Femoral artery B) Starvation
B) External iliac artery C) DKA
C) Popliteal artery D) CRF
D) Tibial artery E) Carbonic anhydrase inhibitors
Ans: C Ans: E
108) Halothane should not be given in 116) During exercise blood flow to lung
hypotension because increase to which zone
A) It is potent vasodilator A) Zone 1
B) Causes hypersensitivity B) Zone 2
C) Raises ICP C) Zone 3
Ans: A D) Zone 1 & 2
109) Pt delivered baby through epidural E) Zone 2 & 3
anesthesia now c/o loss of sensation in Ans: D
thumb. Nerve segment most likely involved  During exercise the blood flow to lung is increased in all
A) C5 parts of lung
B) C6  During exercise pulmonary vascular pressure rises
C) C7 enough to convert lung apex that is zone 1 and 2 into
D) T1 zone 3 pattern. The zone 3 already had maximum blood
Ans: B
flow
110) 18 years old girl with lump in her RT
 Here is the tricky point, the zone 3 already had maximum
breast which is non-tender and no enlarged
lymph Node noted, what is diagnosis??? flow, so the flow will be increased to zone 1 and zone 2
A) Carcinoma in situ which is option D
B) Invasive ductal carcinoma
C) Fibroadenoma 117) Which of following is commonly
D) Traumatic injury caused by asbestos.
E) Mastitis A) Meningioma
Ans: C B) Mesothelioma
111) A lady after surgery with acute blood C) Bronchogenic carcinoma
loss. Which type of anemia will be present .
A) Microcytic normochromic anemia Ans: C
B) Microcytic hypochromic anemia 118) Third heart sound
C) Normocytic normochromic anemia A) Produced during early systole
D) Megaloblastic anemia B) Also known as atrial gallop
Radiant Notes—FCPS PEARLS Golden File 4 By: Dr. Rafi Ullah
C) Produced during late diastole Ans: B
D) Don’t remember other options 129) Hyperosmolar over hydration is
Ans: C associated with?
119) Point mutation occurs in A) SIADH
A) Sickle cell anemia B) Inc water intake
B) Carcinoma of breast C) Cushing syndrome
C) Thalassemia major D) Bronchogenic Ca
Ans: A Ans: C
120) Arbor vitae true is Remember
A) Trapezoid body o Hyperosmolar over hydration = Cushing
B) Branching of cerebellar hemispheres syndrome
Ans: B o HypoOsmolar over hydration =SIADH
121) Regarding axonal flare
Ans: Triple response 130) Regarding relation of hepatic flexure.
Some other options can’t remember A) Lies above third part of duodenum
122) Deep cervical artery is a branch of B) Lies in front of RT kidney
A) Costocervical trunk Poor Stem can’t recall other options
B) Thyrocervical trunk Ans: A
C) Maxillary artery 131) Boundaries of Morrison’s pouch
D) Posterior auricular artery formed by all except.
Ans: A A) Anterior abdominal wall
123) Relation of superior thyroid artery B) Falciform ligament
A) Lies above hyoid bone C) Coronary ligament
B) Lies in front of isthmus of thyroid D) Inferior surface of liver
Some other options was very tough one Ans: B
132) Which of following is most potent for
Ans: B microbial killing.
124) Which of following deficiency is A) H2O2
present in hemophilia A. B) Superoxide
A) Anti-hemophilia globulin Ans: A
B) Hageman factor 133) Which enzyme helps in angiogenesis
C) Tissue factor in wound healing
D) Plasma thromboplastin A) Alpha 1 antitrypsin
E) Fibrin stabilizing factor B) Caspases
Ans: A C) Metalloproteinases
125) Which growth factor is associated Ans: C
with pathogenesis in wound rather than 134) If specific gravity of urine is
physiological process? increased. which function of kidney are we
A) Endodermal derived growth factor measuring ( something like this)
B) Platelet derived growth factor A) Filtration
C) Transforming growth factor B) Concentration
D) Fibroblast growth factor C) Reabsorption
Ans: C D) Secretion
126) If a pt had VT 500 ml and lung Ans:B
capacity increased (something like this) so 135) Onchocercosis causes which of
that VT becomes 1000 ml. what are we following
measuring in this PT.? A) Interstitial nodules
A) Compliance B) Corneal hemorrhages
B) Resistance C) Eye and skin nodules
C) Total lung capacity D) Lymph nodes problem
Don’t remember other options Ans: C
Ans: A 136) True about graves ophthalmology
127) Regarding relation of splenic flexure A) Usually bilateral
No options recalled, read the topic B) Optic nerve is effected
128) Which growth factor most important C) Most commonly involves intraocular
for wound healing. muscles
A) EDGF Ans: A
B) PDGF 137) Most common site of gastric ulcer
C) Thromboxane A2
Radiant Notes—FCPS PEARLS Golden File 4 By: Dr. Rafi Ullah
A) Body Ref: Lippincott's Illustrated Q&A Review of Anatomy and
B) Antrum Embryology page 64, chapter: 4
C) Anterior surface 144) First to occur in atherosclerosis
D) Pylorus A) Endothelial injury
E) Fundus B) Fatly streak
Ans: B Ans: B
138) Growth hormones 145) In a patient with increase water
A) Decrease protein entry into cell content. Which of following most likely.
B) Decrease levels of free fatty acids in A) Cortisol excess
blood B) Growth hormone excess
C) Does not cause hyperglycemia C) Nephrogenic DI
Ans: A D) Diabetes insipidus
139) During exercise increase in Ans: C
ventilation & respiratory rate is due to. 146) A pt (long scenario given but with
A) Stretch receptors of skeletal muscle loss of ankle jerk) lesion is present in which
B) Increase in arterial co2 nerve root.
C) Increase in venous co2 A) L4
D) Decrease in arterial 02 B) L3
E) Direct stimulation by medulla C) S1
Ans: C D) S2
140) Basal ganglia damage which of Ans: C
following still intact 147) Another one about loss of ankle jerk
A) Voluntary movements & root value was asked and ans was S1.
B) Withdrawl of foot upon pinching of big
toe. 148) In standard deviation
Ans: B A) Mean, Mode, Median Coincide
141) Regarding hip joint true is
A) Capsule covers anterior surface of It 149) Variance is related to
B) Supplied by inferior gluteal artery A) Square root of standard derivation
C) Supplied by femoral nerve B) Variability
(There was no option of its relation to C) Range
obturator externus muscle) Ans: B
Ans: B
150) Regarding supply of urinary bladder.
142) A pt transfused with blood which was A) Sympathetic supply is from S2 , 3 , 4
followed by intravascular hemolysis. Which B) Parasympathetic supply from L1, 2.
of following will be most likely. C) Supplied by pelvic splanchnic nerve &
A) Hemoglobulinemia inferior hypogastric plexus.
B) Hemoglobinuria Ans: C
C) Hyperbilirubinuria
151) True about stomach.
D) Hyperbillirubinemia
A) Antrum lies b/w incisura & pylorus
E) Severe cyanosis
B) Fundus lies above cardiac orifice
Ans: B
C) Capacity is 100 ml in adults
143) Lesser sac opening relations. D) Is not most distensible part of stomach
A) Related posteriorly to aorta Ans: A
B) Related superiority to quadrate lobe of
152) Function of Ca++ in synapse.
liner
A) Increase depolarization of membrane
C) Related anteriority to inferior vena cava
B) Blocks receptors on post synaptic
D) Bile duct lies on Lt side of hepatic artery
membrane
Ans: A
C) Causes release of Ach in NMJ
 The lesser sac lies anterior to the pancreas and posterior Ans: C
to the stomach. 153) Scenario of thalassemia major
o Anteriorly: Right free margin of the lesser
omentum 154) A pt having fever some treatment
o Posteriorly: The inferior vena cava, Aorta was given now presented with jaundice,
o Superiorly: Caudate process of the liver. painful leg ulcers (scenario of sickle cell but
o Inferiorly: First part of the duodenum was confused with G6PD def). What is pt
now suffering from?
A) Hemolytic crises
Radiant Notes—FCPS PEARLS Golden File 4 By: Dr. Rafi Ullah
B) Auto immune crisis D) ↑ bleeding time
C) Painful crisis Ans: A
D) Sequestration crisis 164) A child has undergone splenectomy,
E) Aplastic crisis now present with epistaxis and multiple
Ans: A petechiae over body which of following is
155) True about blood supply of bones. most likely.
A) Epiphyseal arteries supplies end of long A) ↑ PT , ↑ APTT , ↑ BT , ↓ D-DIMERS
bones B) ↑ PT , ↑ APTT , ↑ BT , ↑ D-DIMERS &
B) Nutrient arteries supplies flat bones. C) NORMAL PT , NORMAL APTT , ↑ BT ,
Ans: A ↓ PLATELET
156) Sign of water intoxication I marked B due to DIC because nothing
A) Slow pulse else was making sense.
B) Irritability Ans: B
C) Raised JVP 165) MAP is regulated by
Ans: A A) Venous return
157) Max pressure in aorta during B) Cardiac output
A) Reduced ejection C) Blood volume
B) Isovolumetric contraction Ans: B
C) Rapid ejection 166) Infundibulum present in (which part
Ans: A of) heart.
158) Intercostal notches are present A) Right atrium
(scenario of Coarctation of aorta). B) Right auricle
A) Pre- ductal Coarctation of aorta C) Right ventricle
B) Post –ductal Coarctation of aorta D) Left atrium
Ans: B E) Left atrium
159) Pt with anemia, fatigue weight loss. Ans: C
Which of following responsible. 167) True about Hartman’s pouch.
A) Tape worm A) Present in callot triangle
B) Ancylostoma duodenale B) Common site for gall stones
C) Onchocercosis C) Only present in pathological conditions
D) Enterobius vermicularis of gall bladder
E) Trichuris trichuri Ans: B
Ans: A (Tapeworms (e.g. Diphyllobothrium latum 168) A pt presented with urine osmolarity
causes megaloblastic anemia) of 1200 serum osmolarity of 350. What is
160) A pt with cirrhosis of liver. Which cause
vein dilated? A) SIADH
A) Hepatic vein B) Diabetes insipidus
B) Inferior vena cava C) Water deprivation
C) Portal vein Ans: A (in SIADH there is Hyponatremia. & Urine
Ans: C osmolality > serum osmolality)
161) A new born baby with extravasation 169) Study conducted about
of urine & malformed pubic Atherosclerosis and was found that
A) Exstrophy of bladder pathogenesis started in child hood. First to
B) Persistent allantois appear is
C) Absent mesenchyme A) Fatty streak
Ans: A B) Platelet deposition
162) A pt with ↑BT, ↑ APTT & normal PT C) Subendothelial monocytes
with platelet count of 180000 which of D) Fibrous plaques
following most likely. Ans: A
A) Hemophilia 170) A pt with cervical lymph node
B) Platelet functional disorder enlargement. which of following has best
C) Von Willebrand disease prognosis
D) Vit. k deficiency A) NHL
Ans: A B) Hodgkin lymphocyte predominant
163) A pt diagnosed as DIC , which of Ans: B
following is most useful for diagnosing DIC 171) Regarding thoracic duct true is
A) Presence of fibrin degradation fibers A) Drains into junction of subclavian &
B) ↑PT & APTT internal jugular vein.
C) ↑ platelet count
Radiant Notes—FCPS PEARLS Golden File 4 By: Dr. Rafi Ullah
B) Begins in abdomen as cisterna chyli on 181) A scenario about inheritance pattern
LT side of aorta was given in which both gene are expressed,
C) Transmitted through esophageal and co-dominance was answer.
opening of diaphragm
Ans: A 182) A scenario was given with lens
172) True about hydrocele dislocation was present.
A) Persistent processus vaginalis A) Marfan syndrome
B) Peritoneal fluid accumulation B) Ehler Danlos syndrome
C) Testes inflammation Ans: A
Ans: A 183) Related to X link
173) Safety margin of drug A) Duchene is linked recessive was ans.
A) Therapeutic index 184) A scenario of ITP was given
B) Efficacy A) Ans was inc megakaryocyte in bone marrow
C) Efficiency 185) First step in inflammation by WBC’S.
Ans: A A) Margination was answer
174) Medical ethics. 186) Regarding Micturition Reflex.
A) Required for PMDC A) Lumbosacral segment of spinal cord
B) Working of doctor in professional life B) Sacral segment of spinal cord S1 & S2
C) Moral code of conduct of doctor C) Controlled by Mid Brain
Ans: C D) Controlled by Cerebrum
175) For sterilization of instruments. True Ans: A and C both correct, i prefer C here
is 187) Neutrophils studied in blood and are
A) All spores can be destroyed by dry found in periphery with O2 consumption.
heating upto 130 centigrade This process is necessary for.
B) Autoclaving is heating objects at 121 C A) Phagocytosis
and (some Pascal’s like that) B) Adhesion
C) Heating will destroy polyvinyl chloride C) Microbial killing
Ans: B Ans: A
176) Which of following is associated with 188) Conversion of Testosterone to
Paraneoplastic syndrome. Estradiol by.
A) Squamous cell Ca lung A) FSH
B) Small cell Ca lung B) LH
C) Renal cell carcinoma Ans: A
D) Invasive ductal carcinoma 189) A PT undergone cholecystectomy
Ans: B and is taking diet deficient in fresh fruit and
177) Spontaneous pneumothorax vegetables now presented with difficulty in
associated with. wound healing which of the following will be
A) Prolong oxygen therapy seen.
B) Shift of trachea to contralateral side A) Deficiency of collagen
Ans: B B) Defective collagen
178) True about beta receptors Ans: B (Dec tensile strength of collagen if present in
A) Causes vasoconstriction options click that)
B) Causes bronchoconstriction
C) Causes cardio acceleration
D) Decreases heart rate
4th May Morning 2017
Ans: C SURGERY
179) True about gastric cells. 1. Rapidly adapting receptor
A) Copious eosinophilic cytoplasm with abundant a) pacinian
nucleus b) ruffini
c) Messiner
180) A pt presented with difficulty in d) merkel
closing eyes with drooling of salvia from Ans: A
angle of mouth. Which of following is 2. hypertensive and chronic renal
responsible failure, what is unlikely
A) Corynebacterium diphtheria a) anemia
Some other options were present of different organism b) hypophosphatemia
c) hyperkalemia
d) hypocalcemia
Radiant Notes—FCPS PEARLS Golden File 4 By: Dr. Rafi Ullah
Ans: B 12. A child of presented with complain of
per rectal bleeding on examination there
3. A patient came with complain of severe is rectal prolapse, causative organism?
vomiting with following labs PH=7.33(7.34-7.44) Ans: Trichuris trichuri
PCO2 35mmhg (35-40) HCO3 22(normal 23- 13) A child with history habit of eating mud. He
26) what’s the interpretation. also complain of worms of 6 cm coming out
a) Metabolic acidosis respiratory alkalosis a) Ascaris Lumbricoides
b) Respiratory alkalosis, partially compensated. b) Enterobius wormicularis
c) Respiratory acidosis with metabolic alkalosis c) Taenia Solium
Ans: A d) Tania Saginata
4. Pulsating mass in abdomen of an 80-year- Ans: A
old lady. Most common location? 14. Response of posterior hypothalamus in
a) T12-L2 temperature regulation of body?
b) T9-T11 a) Peripheral vasodilation
c) L2-L5 b) Shivering
d) S2-S4 c) Sweating
Ans: A Ans: B
5. 2 year old child, born premature. Most 15. Relation of internal jugular with carotid artery in
common cardiac defect? Neck
a) foramen ovale failure to close a) Post
b) PDA b) Medial
c) Failure of obliteration of umbilical artery c) Lateral
Ans: B Ans: C
6. Ligamentum arteriosus mass, which 16. Retromandibular joins which vein to form
structure will be damaged. external jugular?
Ans: left recurrent laryngeal nerve a) Posterior auricular
7. What causes constipation? b) Facial
a) colchicine Ans: A
b) diamorphine 17. Sympathetic fibers in?
c) ampicillin a) Each spinal nerve
Ans: B b) thoracic spinal nerve
8. Aspirin contraindicated in? c) lumbar spinal nerve
Ans: peptic ulcer d) cervical spinal nerve
e) sacral spinal nerves
9. Alfa fetoprotein Ans: A
Ans: Hepato carcinoma 18. Surgical neck of humerus fractured. Which
10. lichen planus rate of conversion to cancer artery?
a) 10-15% a) Axillary
b) 1-10 b) posterior Humeral
c) 15-20 c) Brachial
d) 20-25 Ans: B
Ans: B 19. Pterygopalatine raphe provide
10. Most common premalignant in or attachment to?
cavity?(don’t remember exact stem, may be it was a) Buccinator
about most premalignant in oral cavity) a) superior constrictor
a) lichen planus b) middle constrictor
b) leukoplakia Ans: A
c) Erythroplakia 20. A 78 years old male with presented with
Ans: B burning micturition, urgency, dysuria and back
11. A 25 year old male has developed cluster of pain. There is also history of left upper limb
vesicles on lateral side of lips. The causative pain and fracture of long bone. On labs there is
organism resides in? increase alkaline phosphatase. Most appropriate
A) Trigeminal Ganglion Investigation?
B) Pterygoid ganglion a) Serum PSA
C) Otic ganglion b) Serum Calcium
Ans: A c) PTH level
Ans: A
21. Blood supply of skin of scrotum.
Radiant Notes—FCPS PEARLS Golden File 4 By: Dr. Rafi Ullah
a) External pudendal artery 32. A person has difficulty in vision at night,
b) Testicular artery this due to deficiency of?
c) inferior epigastric artery a) Retinol
d) inferior vesicle artery b) Retinoic Acid
Ans: A c) Retinaldehyde
22. Supra renal gland Ans: A
a. on each side IVC 33. A patient with acute pancreatitis, drug
b. Rt supra renal gland lying in tendinous appropriate for pain relief?
ligament a) Pethidine
c. Right suprarenal gland lie in central tendon of b) diamorphine
diaphragm c) morphine
d. Left one is related anteriorly to pancreas Ans: A
e) Left suprarenal gland lie in central tendon of 34. After infusion of Propofol the hypotension
diaphragm occurs due?
Ans:A a) its direct effect on vasomotor center
23. Water intoxication? b) It causes peripheral vasodilation
Ans: slow pulse c)its direct effect on metabolites
24. Infection in pretracheal fascia goes to Ans: B
Ans: ant mediastinum 35. 3 year old boy with flank pain, on
25. Micturition reflex? ultrasound there is hydronephrosis of left kidney.
a) is integrated sacral spinal segment What is the most likely cause of unilateral
b) once initiated its self-regenerating hydronephrosis?
Ans: A a. atony of bladder
26. Nerve supply of rectus abdominus? b. external urethral meatus stenosis
a) last five intercostal and subcostal never c. constriction of posterior urethral valves
b) subcostal only d. constriction at pelviuretic junction
c) last five intercostal , subcostal, ilioinguinal and Ans: D
iliohypogastric
d) Last five intercostal, subcostal and ilioinguinal 36. Anterior inferior iliac spine fracture, pelvic
only. avulsion is done by which muscle?
Ans: A a. rectus femoris
27. Heparin needs which anticoagulant factor to be b. Sartorius
present for its action? c. quadriceps
a) antithrombin III d. hamstrings
b) factor XII e. obturator internus
c) inhibitor antithrombin III Ans: A
Ans: A 37. In dehydration with ADH effect,
28. Mast cells release which anticoagulant which part of nephron will be hypotonic
substance? fluid
a) Heparin a. PCT
b) Histamine b. DCT
c) Bradykinin c. Collecting duct
Ans: A d. Ascending loop
e. Descending loop
29. Which nerve lies in the cavity of
Ans: C
cavernous sinus?
Ans: abducent 38. In Tabes Dorsalis?
a) there damage to posterior column
30. Vit k dependent factor?
b)there is atony of bladder
a) Prothrombin
c)is due to loss of parasympathetic
b) Stable factor
efferent nerve
c) Stuart Factor
Ans: B
Ans: A
39. A pathologist done a study on risk factors of
31. Richest source of Vit k?
congenital urinary defect. Which of following is
a) tubers
associated with increased risk of urothelium origin
b) cereals
carcinoma?
c) green veg
a) Exstrophy of Urinary bladder
d) pulses
b) Duplication of ureter
Ans: C
c) Medullary spongy kidney
d) unilateral renal agenesis
Radiant Notes—FCPS PEARLS Golden File 4 By: Dr. Rafi Ullah
e) Horse-shoe shape kidney 51. Papillary cancer in female patient is due
Ans: A to?
40. A patient has swelling of parotid gland a) she had received irradiation
and posterior auricular pain, which is b) exposure to arsenic
increased by mastication, is due to the Ans: A
involvement of which nerve? 52. Body of hyoid bone lies at which level
Ans: Auriculotemporal nerve in erect position?
41. Trachea is supplied by which nerve? a) C3
a) recurrent laryngeal nerve b) C4
b) glossopharyngeal nerve c) C6
c) intercostal nerve Ans: A
Ans: A 53. Ureter at pelvic brim damage during
42. There is a patient with Jugulodiagastric ligation of which vessel?
lymph node enlargement it is due the a) Iliolumbar vein
a) pharyngeal tonsil b) superior gluteal artery
b) palatine tonsils c) ovarian artery
c) parotid Ans.C
d) mandibular 54. Thorn prick abscess causative organism?
e) Sublingual Ans: staph aureus
Ans: B
43. Growth on ductus arteriosus, which 55. A child with swollen, tender hot joint,
structure compresses movement at joints are limited, joint
a) left recurrent laryngeal nerve aspiration is done, it will yield which
b) right recurrent organism?
c) right phrenic Ans: staph aureus
d) left phrenic 56. Infection in pretracheal fascia goes to
Ans: A Ans: ant mediastinum
44. Musculophrenic artery branch of? 57.Tabes Dorsalis incontinence affect
a) internal thoracic Ans: posterior column
b) Pericardiophrenic 58. Cohort study?
Ans: A a. Exposure vs non exposure
45. Branch of lateral cord of brachial b. Disease vs non disease
plexus? Ans: A
a) Musculocutaneous
a) long thoracic
59.multiple lung abscess with yellow granules
b) Thoracodorsal
Ans: Actinomyces
Ans: A
46. During supra pubic catheterization a
60.preterminal bronchiole epithelium
transverse incision is given, which most
Ans: Simple columnar without goblet cell
likely to be damaged here?
a) inferior epigastric artery
b) superior epigastric artery 61. Derivative of 2nd pharyngeal arch Ans:
c) deep circumflex artery Stylohyoid ligament
Ans: A
47. Epicardium is supplied by? 62.largest epidural space is at
Ans: Pericardiophrenic artery Ans: L2
48. Dorsiflexion is done by which muscle? 63 80 year old man high altitude swelling in legs...
a) Tibialis anterior a) Co poisoning
b) Peroneus longus b) venous thrombosis
Ans: A c) cardiorespiratory failure
Ans: C
49. In flexor sheath in flexor retinaculum
contains? 64. Esophagus-gastric junction at level of?
a) Flexor pollicis brevis a) T7
b) flexor pollicis longus b) t11
c) Extensor pollicis longus c) L2
Ans: B Ans: B
50. Brodie’s abscess cause? 65. OCP increase risk of?
Ans: pyogenic osteomyelitis Ans: thromboembolism
66. Increase BP but decrease heart rate?
Radiant Notes—FCPS PEARLS Golden File 4 By: Dr. Rafi Ullah
a) exercise Ans: A
b) increase cranial pressure 78. Clostridia causes?
c) sympathetic stimulation a) gas gangrene
Ans: B b) Upper respiratory infections
67. Blood flow velocity increase? c) necrotizing fasciitis
a. svc Ans: A
b. right atrium 79. Which drug cause constipation...
c. cerebral capillaries a) colchicine
d. pulmonary artery| b) digoxin
e. right ventricle c) quinine
Ans: D Ans: A (if morphine inoptions prefer that)
68. ADH increase when 80. Child PT', APPT normal prolonged BT?
a) pressure decrease in right ventricle a) Factor VIII def
b) pressure decrease in right atrium b) Christmas disease
Ans: B c) idiopathic thrombocytopenic purpura
69.Ascitic tap most common organism Ans: C
Ans: E coli
70) Patient of pancreatic Ca right sided leg pain, 80. In L.P structure pierced?
his pain was relief by cutting down? a) Dura
a)left lateral spinothalamic b) supraspinatus ligaments
b)left ventral c) spinal cord
Ans: A d) pia matter
71. Problem in doctor patient relationship Ans: A
misconception? 81. IgE binds with?
Ans: Hurried and casual approach a) Platelets
b) Mast cells
72. A person can’t scratch on his back c) Lymphocytes
rolling back his arm due to damage to? Ans: B
a) Latissimus dorsi 82. A hypertensive patient has headache then
b) Serratus ant he suddenly got unconscious. LP was done
c) teres major and it showed RBC in CSF.
d) supraspinatus Ans: SAH
e) infraspinatous 83. Bradykinin is formed?
Ans: B a) Platelets
73. Distention of rectum is caused by? b) by action killikein
a) superior rectal nerve Ans: B
b) inf. Rectal nerve 84. Which is non-malignant?
c) sympathetic innervation a) Hamartoma
d) parasympathetic b) Burns Scar
Ans: D Ans: A
74. Regarding Skull? 85. Central value of serial data?
Ans: pterion is the area where branches of middle a) Mean
meningeal lies b) Median
75. Atherosclerosis commonly occur in? c) Average
a) Aorta Ans: B
b) Penetrating myocardial arteries coronary 86. Natural defense against neoplasia?
Ans: A Ans: Apoptosis
76. Helminth disease caused by mosquito? 87. Organelle with double membrane
a) Filariasis a) The Nucleus
b) onchocercosis b) Golgi
a) Schistosomiasis c) Centrioles
Ans: A Ans: A
77. A person who does heavy exercises 88. Carcinoma is caused by which
regularly, His resting HR will be inorganic compound?
a) less than 60 a) asbestoses
b) 80 b) Silica
c) 80-100 c) Dust
d) 100-150 Ans: A
Radiant Notes—FCPS PEARLS Golden File 4 By: Dr. Rafi Ullah
89. When more energy is needed to cell what b) pelvic
happens to mitochondria? Ans: A
a) mitochondria divide 100. Inguinal hernia most common structure
b) undergo replication using its own DNA encountered while doing routine surgery for
Ans: B inguinal hernia?
90. Regarding Dead cell diagnostic on Ans: pampiniform plexus
microscopy? 101. Action of epinephrine on alpha receptors?
a) karyorehexis a)pupillary radial muscle constriction
b) karyolysis b)bronchodilation
c) cell shrinkage c)Increase heart rate
d) Hydropic degeneration d)vasodilation
Ans: B Ans: C
Pyknosis, karyorrhexis and karyolysis, all 102. Angle of axis of head of femur with long axis
are the cardinal morphological features of of shaft of femur?25
cell death----- a. 25
typical pattern is pyknosis (nuclear b. 125
condensation) followed by karyorhexis c. 67
(nuclear fragmentation) followed by Ans: B
karyolysis (complete dissolution of the 103. Regarding Aspirin?
nuclear fragments), Ans: thromboxane synthase inhibition
Pyknosis is common to all cell death
104. A baby with mental retardation, upward
processes. It’s the next step that differs.
slanting epicanthal folds?
Apoptosis will show karyorhexis, Oncosis
a) trisomy 13
will show karyolysis.
b) Trisomy 18
91. Which lung spirometry parameter is helpful c) trisomy 21
in COPD? Ans: C
a) v/q ratio =1
104. Deglutination phases?
b) v/q ratio=0.1
a) bolus moves down to throat by voluntary
c) v/q ratio = infinity
movement of tongue
d) physiological dead space
b) upward movement of larynx during pharyngeal
e) shunting
phase prevent food aspiration
Ans:B
c) primary peristalsis is initiated by pharyngeal
 A normal V/Q ratio is around 0.80
phase of deglutition
 In copd vent dec bt perfusion remains same so
Ans: A
ratio dec
105. Focal glomerulosis?
92.Atrophy
a) deposition under epithelium
a) is increase in number of cell
b) can reoccur in transplant kidney
b) increase in cell size
Ans: B
c) Decrease in cell size
Ans: C 106. A baby is suffering from severe diarrhea,
develop severe dehydration, IV hydration is
93. Hep B spreads
required for treatment, to make an IV line, a
a) Saliva
saphenous Venesection is done , which structure
b) lacrimation
can be damage by doing this procedure?
c) Mosquito bite
a) Saphenous nerve
d) Sexual contact
b) peroneal nerve
Ans: D
Ans: A
94. Winging of scapula
107. Which does not change during
Ans: Serratus Anterior
pregnancy?
95. Carpal tunnel a) Cortisol
Ans: Flattening of thenar eminence b) PTH
96. IV drug user? c) GH
Ans: infective endocarditis d) FSH
97. MS murmur best heard at? Ans: C
Ans: at apex of heart 108. Increase alpha feto protein is associated with?
98. Bile salt absorption occurs in? Ans: hepatocellular carcinoma
a) ileum 109. True hermaphrodite?
99. Most common position of Appendix? a) XXY
a) Retrocecal b) XYY
Radiant Notes—FCPS PEARLS Golden File 4 By: Dr. Rafi Ullah
Ans: A a. more readily bonds with basic drugs
110. Basic mechanism involved in aqueous b. more readily binds with acidic drugs :
humor production: c. All drug are transported binding to plasma
a) Active sodium secretion protein.
b) Passive Chloride absorption Ans: B
c) Glucose diffusion 121. Regarding pleuroperitoneal membrane?
d) Ultrafiltration Ans: separates peritoneal cavity from thoracic
e) Water secretion by simple diffusion cavity
Ans: A 122. Decrease FSH in male result in
111. A feature of atrial fibrillation? decrease?
Ans: pulses deficit a) testosterone
112. Regarding PTH? b) sperm count
a. decrease absorption of calcium Ans: B
b. Decrease conversion of 25-cholicalciferol to
1,25 di Cholecalciferol
Surgery and Allied 4th May 2017
Ans: B (it requires PTH to be present) Evening
Q1. In comparison to ICF ECF has?
113. Most common congenital anomaly of head Ans: greater CL ion
and neck? Q2. Main determinant of diastolic pressure is?
a) cleft lip and palate Ans: venous return
b) deformities of pinna Q3.about baroreceptor?
Ans: A Ans: present in adventitia of large arteries
114. Hypertrophy? Q4. Scenario of hypothyroidism was asked what will
a. increase in size of cell increase in hypothyroidism
b. increase in number of cell Ans: blood cholesterol.
Ans: A Q5.myelination of peripheral nerve is through?
Ans: Schwann cell
115. Ventricle systole corresponds on ECG
Q6.in scenario of hemolysis?
with?
And: decreased haptoglobin
a. PR interval
b. ST Q7.in primary intention healing what will be the
c. QRS ‘ contributing factor?
d. QT Ans: closely stitched wound (approximation of edges)
Ans: C
116. Which organ utilizes 70% of O2 of its arterial Q8. About milk production during pregnancy?
supply at rest? Ans: inhibited by estrogen and progesterone
a. liver Q9.a 5 years old boy who had generalized edema
b. brain periorbital edema what will the cause?
c. heart ‘ Ans: albuminuria +sodium retention
d. kidney Q10. One question was about ATP and its uses in
e. skeletal muscles the body?
Ans: C Ans. Energy release when phosphate bonds of ATP
117. Ca of breast of outer upper quadrant will drain breaks and produce energy
into? Q11.obstructive and restrictive disease of lung
a. Pectoral nodes differentiated by which test?
b. internal mammary nodes Ans. FEV1/FVC
c. supra clavicular nodes Q12.aspirin decrease fever by which mechanism?
Ans: A Ans. Decrease prostaglandin.
118. Acute inflammation? Q13. Peroxisome causes?
a. systemic response of living tissue to infection Ans. Oxidation of very long chain fatty acid.
b. Local response of living tissue to tissue insult.’ Q14. About plasma osmolality most correct
Ans: B statement is?
119. Prophylactic antibiotic? Ans. Similar to 0.9 % normal saline.
a. should be given at the time of induction of Q15.necotinic receptor blocked by?
anesthesia Ans. Curare drugs
b. should be given post operatively Q16.during repolarization which ion goes out of
Ans: A cell?
120. Regarding plasma albumin? Ans. K+
Radiant Notes—FCPS PEARLS Golden File 4 By: Dr. Rafi Ullah
Q17. About axon? Ans. Falciparum malaria
Ans. Carry impulses towards cell body Q31.which one act through signal on receptor?
Q18. About ESR which one is most appropriate? A. Gq protein
Ans. Increased in infection B. Adenyl cyclase
Q19.T cells need to bind with what to access C. Phosphorylation
antigen? Ans: A
Ans. With MHC Q32.costocervical artery is branch of?
Q20. Pt presented with chest pain what single Ans. Second part of subclavian artery
question help you most to reach the diagnosis? Q33.damaged RBCs filters and destroys in which
Ans. Put your finger and point out where you feel the structure of spleen?
pain. Ans. Red sinus with red pulp
Q21. A pregnant lady with obstructive jaundice Q34.which one is antitumor cell?
which test should be done? Ans. CD8 T lymphocyte
Ans. GGT Q35. Most common site of thrombi of pulmonary
Q22.about T lymphocyte? embolism is?
Ans. Mature in thymus after birth Ans. Deep vein of calf muscle (vein of soleus muscle)
Q23.clinically palpable lymph node but not mattaled Q36. Lower part of floor of acetabulum is fractured
and the likelihood of metastasis in malignancy what which bones are involved
will be the stage of this cancer? Ans. Pubic bone and ischium
Options Q37. Which artery passes anterior to 3rd part of
A. T1N1M1 duodenum?
B. T1N1M0 Ans. superior mesenteric artery
c. T2N1M1 Q38. Main pancreatic duct opens in which part of
d. T1N0M0 duodenum?
Ans: C Ans. Second part of duodenum
Q24. Pt has seizure (increased neuronal discharge) Q39. Process vaginalis is derived from?
what inhibitory neurotransmitter is decreased? Ans. Peritoneal diverticulum
Ans. GABA Q40. Vol. of air maximally expired after deep
Q25.in a study carotid chemoreceptor denervated in inspiration?
animals so which one is least effected. Ans. Vital capacity
a. A.PO2 arterial Q41. What is true about FRC?
b. B.PCO2 arterial Ans. Amount of air remained in lung after expiration of
c. C.PO2 venous tidal vol.
d. D.H+ ion
Q42. Infection from anterior part of neck can spread
e. HCO3 level
to?
Ans: E
Ans. Anterior mediastinum
Q26.a lady who’s sister is died few week ago
Q43. Which structure pass most posteriorly in
presented to you for consultation but when come in
diaphragm?
your room started crying could not say words due to
Ans. azygous vein, thoracic duct and aorta
crying now what should you say to her?
Ans. Please take your time Q44.in spinal tab which CSF was blood stained what
will be the diagnosis?
Q27. Gastrin secretion not affected by?
Ans. Subarchanoid hemorrhage
a. Antrum distension
b. Caffeine Q45. In parasympathetic which one is least effected?
c. Alcohol Ans. Pupillary dilator muscle.
d. Protein in stomach Q46.loss of sensation on medial side of hand and
Ans: C claw of two medial fingers in distal interphalangeal
Beverages with a higher ethanol content (whisky, gin, cognac) joint which nerve is damaged?
do not stimulate gastric acid secretion or release of gastrin Ans. Ulnar nerve
Q28. Which tissue is mostly susceptible to ischemic Q47. Sub mucus Brunner’s gland present in which
injury? part of GIT?
Ans. Neuron Ans. Duodenum
Q29. A women did difficult delivery in a rural area Q48. Which one is primary cartilaginous joint?
with history of fever, bleeding, bruises and Ans. Costochondral joint
increased PT and APTT what will be the diagnosis? Q49. One question was about scalp infection?
Ans. DIC Ans. I did sebaceous cyst
Q30. Periodic high grade fever in a man who recently Q50. Opening of mouth through which muscle?
traveled to Pakistan and urine turned black? Ans. Lateral pterygoid muscle
Radiant Notes—FCPS PEARLS Golden File 4 By: Dr. Rafi Ullah
Q51. Spared wrinkle over forehead but deviation of Ans. Most abundant in connective tissue
corner of mouth on right side where is the lesion? Q69. What is bells phenomenon?
Ans. Left upper motor neuron of facial nerve? Ans. Upward and outward movement of eye when pt
Q52. A scenario of brown Séquard syndrome which want to close the eye
one is most appropriate? Q70. Nearest structure to crus cerebri is?
Ans. Loss of vibration, fine touch, pressure, loss of Ans. Substantia nigra
motor on same side and pain temperature sensation on Q71. A male working in an industry exposes to
contralateral side of Hemisection. hydrocarbons developed fever and weight loss what
probably this pt will develops?
Q53. Nucleus pulposus is remnant of? Ans. CA bladder
Ans. Notochord Q72. Parathyroid hormone acts by which mechanism
Q54. About autonomic system ? to increase blood Ca level?
Ans. Preganglionic autonomic nervous system always Ans. Increase Reabsorption of Ca from distal convoluted
works by acetylcholine tubules and decrease reabsorption of phosphate from
Q55. Melanocyte derived from? proximal convoluted tubule.
Ans. Neural crest cell. Q73.which hormone of anterior pituitary gland is
under direct inhibitory control of hypothalamus?
Q56.mechanism of action of loop diuretic? Ans. Prolactin
Ans. Inhibit co transport system (Na/K/2Cl) of thick Q74. A 14 years old boy with trauma to anterior
ascending limb of loop of Henle aspect of thigh after one month developed pain , pus
Q57.most common mode of inheritance? formation and hip extension restriction and
Ans. Multifactorial decreased size of muscle what will be the cause ?
Q58. Most common cells in tail of pancreas of islet A. Contracture.
cell of Langerhans? B. Septic shock
Ans. Beta cell Ans: A
Q59. About sternal angle of Louis Q75. Cause of pyogenic peritonitis?
Ans situated b/w t4-t5 vertebrae Ans. Bacteroids.
Q60 what is most common cause of prolonged Q76. A scenario of TB what should be done for
bleeding? confirmation of TB in histological study?
a. Aspirin toxicity Ans. Caseous necrosis.
b. Von Willebrand disease Q77.a 70 years old man experiences repeated UTI
c. Thrombocytopenia what will be the cause?
Ans: A Ans. Obstruction
Q61. A 9 month old boy with petechiae all over body Q78. What feature is most appropriate about
since 2 days history of vaccination normal two malignancy?
weeks ago with normal Hb. There was decreased Ans.
platelet and WBC? A. Increased N/C ratio
Ans ITP B. Polymorphism
Q62 acrosome of sperm contains? Ans: A
Ans. Lysosomal enzyme Q79. In HIV pt. definite diagnostic infection is?
Q63. Vit C helps in which process? Ans. Mucor mycosis
Ans. wound healing Q80. Paramesonephric duct give rise to ?
Q64. In hemophilia A which fluid should be Ans. Uterine tube + uterus + cervix + upper part of
transfused? vagina
Ans. Cryoprecipitate Q81. In right shift oxygen Hb. Dissociation curve
Q65.type of study and relation b/w duration and atmospheric pressure will be?
body temperature shows by which of the following? a. 26mmhg
Options: b. 35mmHg
A. T test Ans: B
B. Regression test Q82.pleurocentesis is done where?
Ans: A Ans. Lower part of 5th and 6th intercostal space anterior
Q66.after abdominal surgery pt developed Abd. axillary line.
Distention hypotension and tachycardia what is the
Q83. A baby with papillary carcinoma of thyroid with
cause?
no family hx Of cancer what would be the risk?
Ans. Hypovolemic shock
Ans. Fetal period radiation exposure.
Q67. Most potent biomarker for MI is?
Ans. Trop T
Q68. What is the correct statement about collagen?
Radiant Notes—FCPS PEARLS Golden File 4 By: Dr. Rafi Ullah
Q84. A 6 years old boy with pneumonia, respiratory Q101. A women with weakness dyspnea, HB 8mg/dl
problem, And Fever, most probably organism will big platelet mcv 110 mch 43 and macrocytosis?
be? Ans. Megaloblastic anemia
Ans. Strep. Pneumonia Q102. Creatinine is used for clinical measurement of
Q85. A scenario of suspected case of beta hemolytic GFR over urea because?
infection confirmed by? Ans. Not reabsorbed in blood
Ans. Blood culture Q103. Urea formed in liver to excrete?
Q86.green discharge from site of infected wound Ans. Ammonia
blood test showed gram negative rod a non-lactose Q104. Calcitonin related to?
fermenting organism which organism is culprit for Ans. Thyroid
infection? Q105. Chronic alcoholic, USG abdomen showed
Ans. Pseudomonas cirrhosis of liver on palpation liver not enlarged but
its hard pt. is on group therapy with other chronic
Q87. A 9 years old boy after knee operation user ,left drinking and continued abstinence, but still
developed fever tenderness swollen knee with pus there is chance of which problem.
filled blood test showed beta hemolytic catalase Ans. Developing Hepatic CA
positive coagulase positive which organism is Q106. Cause of Cholangiocarcinoma?
culprit for this case? Ans. Clonorchis Sinensis
Ans. Staph. Aureus Q107. Aflatoxin B marker of?
Q88. Most common hospital acquired wound Ans. Hepatic CA
infection is caused by which organism? Q108. Cause of malignancy in lung?
Ans. Staph. Aureus Ans. Asbestosis (cause mesothelioma)
Q89. Common carotid artery divided at which level? Q109. Most diluted urine found in?
Ans. Upper border of thyroid cartilage Ans. DCT
Q90. Amoeba cause lesion most commonly in which Q110. Carbonic anhydrase (acetazolamide) act anti-
part of GIT? hypertensive effect by?
Ans. Cecum Ans. Cause self-limited NAHCO3 diuresis and decrease
Q91.human being is intermediate host of which total body HCO3 stores.
infection? Q111. Acetylcholine cant breakdown due to
Ans. Hydatid cyst deficiency of?
Q92.pt. with granulomatous disease but no caseous Ans. Psuedocholinesterases
necrosis present can be all the following except? Q112. Rigidity and mask face spastic limbs cause
Ans. TB is?
Q93. About olfactory nerve tract? Ans. substantia nigra damage (Parkinson disease)
Ans. From olfactory receptor to olfactory bulb then direct Q113. Sympathetic activation cause?
to olfactory cortex without relay to thalamus. Ans. Bronchodilation
Q94.premalignant lesion? Q114.accomation for vision is caused by?
Ans: barret esophagus. Ans. Ciliary muscle contraction.
Q95. About atrophy? Q115.which of the following is first line defense
Ans. Decrease cell size against bacteria?
Q96.a women with vaginal bleeding on examination A. Complement
shows cervical polyp in Endocervix lined by B. Leukotrienes
stratified squamous epithelium which is correct C. Opsonin
about epithelium? D. Skin
Ans. Metaplasia Ans: D
Q97. Nasopharyngeal carcinoma cause by? Q116. Which drug is secreted in saliva?
Ans. EBV A. Erythromycin
Q98. Burkitt lymphoma caused by? B. Phenytoin
Ans. EBV C. Streptomycin
Q99. Which tumor is most sensitive to radiotherapy? D. Digoxin
A. Neuroglia glioma E. Phenobarbitone
B. Craniopharyngioma Ans: B
Ans: A  Drugs that appear in saliva includes phenytoin, lithium,
Q100. The off spring of affected parents have 50 % digoxin and salicylates
chance of developing disease and there is no  Excretion of drugs into saliva depends on lipid solubility,
carrier? PKa and plasma protein binding.
Ans. Autosomal dominant trait
Radiant Notes—FCPS PEARLS Golden File 4 By: Dr. Rafi Ullah
 Lipid solubility for drugs are phenytoin > digoxin > Ans. Puborectalis muscle.
Phenobarbitone Q132.left recurrent laryngeal nerve is related to
Q117.which drug diffuse through aqueous humor? which structure?
A. Penicillin Ans. Arch of aorta
B. Cefoxitim Q133. Superior thyroid artery ligated there is chance
C. Erythromycin of damage of which nerve?
Ans: C Ans. External laryngeal nerve
Q118Which local anesthetic mostly used in Q134. A women give birth to an O -ve baby and she
children? herself was Rh-ve, due to bleeding blood transfused,
a. Lidocaine after that she developed transfusion reaction which
b. Bupvicaine will be most appropriate regarding to that?
Ans: A Options:
A. Hemolytic reaction may developed in next pregnancy
Q119. Pt on ATT developed red colored urine drug
B. immediately give anti D antibody
responsible?
C.no reaction will occur
Ans. Rifampicin
D. No need giving anti D antibody
Q120. Pt on ATT developed vision problem which Ans: D
drug is responsible?
Q135. Most common feature of esophageal atresia
Ans. Ethambutol.
and tracheoesophageal fistula is?
Q121. Which analgesic drug cause tachycardia on Ans. Respiratory distress after feeding.
therapeutic dose?
Q136.dialysis fluid has more which substances than
Ans. Pethidine
plasma?
Q122. Which structure connects the two epithelial Ans. Glucose
cells?
Q137.local cause of edema?
A. integrin
Ans. Allergy
B. gap junction
c. Tight junction Q138. One liter blood lost and one liter fluid
D. Desmosome transfused which element return last to normal
E. hemidesmosome value?
Ans: C Ans. RBCs.
Q123.cemitidine cause increase INR and cause Q139.in a person two liter of fluid lost due to
bleeding on pt taking warfarin because? sweating and replaced with two liter pure water what
Ans. Inhibits hepatic metabolism of warfarin on liver. will occur?
Ans. Increase intracellular fluid volume.
Q124.ophthalmopathy develops due to deficiency of
which Vit? Q140. Which one is tumor suppressor gene?
a. Thiamine (B1) Ans. P53
b. VIT D Q141. Which agent cause mutation in
Ans: B Protooncogens to produce its effect??
Vitamin D deficiency is significantly associated with A. Radaiation
ophthalmopathy in patients with Graves' hyperthyroidism B. virus
Ans: A
Q125.a scenario of cheilosis, scaly dermatitis and
glossitis develops due to deficiency of which Vit? Q142.a pt with right hypochondrium pain liver
Ans. Riboflavin (B2) biopsy showed large Epitheloid cells with
granulomatous disease in center one giant cell with
Q126. A diabetic pt with peripheral neuropathy
multiple nucleoli in periphery what is it indicating?
develops gangrene what is the cause?
Ans. caseous necrosis
a. Thrombus in artery
b. Atherosclerosis Q143.adult polycystic kidney disease is?
Ans: B Ans. Autosomal dominant
Q127. In renal disease pheochromocytoma, conns Q144. Most appropriate about multifactorial
syndrome, acromegaly what is common? chromosomal abnormality?
Ans. Hypertension Ans. Trisomy 21
Q128.About thyroid gland most appropriate? Q145.about Klinefelter syndrome most appropriate
Ans. Enclosed in pretracheal fascia statement is?
Ans. Gynecomastia is always present.
Q129.about parathyroid glands most appropriate is?
Ans. Supplied by superior and inferior thyroid artery Q146.a scenario of gynecomastia tall height
chromosomal study shows xxy?
Q130. In first week test for typhoid is?
Ans. Klinefelter syndrome
Ans. Blood culture
Q147. Spinal cord in children ends at which level?
Q131. Which structure is part of levator ani?
Ans. L3
Radiant Notes—FCPS PEARLS Golden File 4 By: Dr. Rafi Ullah
Q148.hemorrhage due to perforation of posterior Q168. Nodule present on lateral side of tongue
part of first part of duodenum which artery is anteriorly diagnosed malignancy first lymph node
involved? involved in drainage is ?
Ans. Gastroduodenal artery Ans. Submandibular lymph node
Q149. Acid secretion of stomach decrease by? Q169.taste sensation lost from anterior 2/3 of tongue
Ans. Gastric acid in antrum of stomach. (Negative reflex) which nerve is damaged?
Q150.most common cause of congenital Ans. Facial nerve
diaphragmatic hernia?? Q170. Pt. brought in ER with muscle contraction,
Ans. Defect in pleuroperitoneal peritoneum. excitement and nervous Ca level is 6, BSR is
Q151. Lymph drainage of gall bladder is to? 120mg/dl what is the cause?
Ans. Lymph node in porta hepatis Ans. Hypocalcemia
Q152.left renal vein pass anterior to ? Q171. Drug of choice in MRSA is?
Ans. Aorta Ans.
Q153 insertion of transvers abdominal muscle is? A. Fusidic acid
Ans. Xiphoid process + linea Alba + symphysis pubis. B. Aztreonam
Q154. Which of the following is closely related to C. Co amoxiclav
femoral ring? D. Metronidazole
Ans. Inguinal ligament E. Penicillin
Ans: A
Q155. Inferior epigastric artery located laterally to
 Aztreonam has only role against gram negative, and no
hernia sac of?
Ans. Direct inguinal hernia role against MRSA
Q156.lymph drainage of glans and corpora of penis  For MRSA: Vancomycin (Drug of choice), daptomycin,
is to? linezolid, tigecycline, ceftaroline, doxycycline
Ans. Deep inguinal lymph node.
Ref: First aid page 194.
Q157. Lymph drainage of cervix?
MRSA remain sensitive to Fusidic acid (Ref: Wikipedia and
Ans. External and internal iliac lymph nodes
NCBI)
Q158.which type of joint is between carpometacarpal
joint of thumb? Q172. During surgery given one liter of blood, pt.
Ans. Saddle joint after recovery in ward developed high grade fever
and tachycardia what is the cause?
Q159. Which type of joint is intervertebral joint?
A. bacterial contaminated blood transfused
Ans. Symphysis
B. febrile non hemolytic transfusion reaction
Q160. About elbow joint most appropriate is? C. transfusion reaction
Ans. Its hinge joint. D. septic shock
Q161.most appropriate about upper part of anal Ans: B
canal? Q173.cause of nonbacterial Libman sack thrombotic
Ans. Sensitive to stretch. endocarditis is?
Q162.sever pain felt at epigastric area this pain Ans. SLE
transmitted through? Q174. Cerebral flow upper and lower limit is?
Ans. Greater splanchnic nerve Ans. 50- 150 ml/min
Q163. Superior rectal artery is a direct branch of? Q175. Most common congenital defect of great
Ans. inferior mesenteric artery. vessel is?
Q164. Which artery arise from terminal part of aorta Ans. Patent ductus arteriosus
at midline posteriorly? Q176. In cardiac cycle most appropriate statement
Ans. Median sacral artery is?
Q165. Arteries and nerves enter base of bladder Ans. Aortic valve closes during Isovolumic relaxation
through? Q177. Pt. of MI admitted in ICU which was good,
Ans. Dorsal lateral ligament suddenly developed dyspnea what is the cause?
Q166. What is landmark for giving pudendal nerve Ans. Ventilation/perfusion defect
block? Q178. Speed of current is highest in which
Ans. Ischial spine conductive system of cardiac tissue?
Q167. A scenario of pt. previously confused and Ans. Purkinje fibers
diplopia suddenly developed unconsciousness what Q179. Most common presentation in Horner
will be the cause? syndrome?
A. Basilar artery thrombosis Ans. Dropped upper eye lid
B. Pontine hemorrhage
Q180. Leg area in representational area of cerebral
Ans: A
cortex supplied by?
Ans. Anterior cerebral artery
Radiant Notes—FCPS PEARLS Golden File 4 By: Dr. Rafi Ullah
Q181. Right bundle branch of heart supplied by? Q198. About anatomical surface of thorax which one
Ans. LCA is most appropriate?
Q182. What is coxa vara? Ans. 12th rib used for purpose of counting of ribs upward
Ans. Decrease in shaft –neck angle of femur Note: there were others options which my mind mostly
Q183. Injury to which nerve cause foot drop? went on this option. But I don’t remember other options
Ans. Common peroneal nerve. Q199.sternohyoid muscle is supplied by which
Q184. Which muscle do not have relation with wrist nerve?
joint movement? Ans. Ansa cervicalis.
Ans. Brachioradialis muscle. Q200. A scenario of viral hepatitis closely related
Q185. Which vein is located in posterior with vasculitis, autoimmune relation and also there
interventricular grove of heart? was given one autoantibody which hepatitis virus
Options: will be that?
A. Great cardiac vein Ans. Hepatitis B virus.
B. Middle cardiac vein (ANS) GYNAE/OBS 2nd march 2017 Morning
C. Small cardiac vein 1. Basilar artery gives off
D. Anterior cardiac vein a. Vertebral arteries
Q186. Anterior surface of heart mainly formed by ? b. Posterior cerebral arteries
Ans. Right ventricle c. Anterior cerebral arteries
Q187. Latissimus dorsi muscle supplied by which Ans. B
nerve? 2. When the vagal supply is cut. Which group of
Ans. Thoracodorsal nerve muscles most affected
Q188. Pt cannot rotate scapula forward which Ans: GIT muscles
muscle is damaged? 3. Regarding megaloblastic anemia
Ans. Serratus anterior Ans: Oval macrocytosis
Q189. Which nerve is a direct branch of trunk of 4. Regarding pernicious anemia
brachial plexus? a. Anti-parietal cell antibodies
Ans. Suprascapular nerve b. Megaloblastic anemia
Q190. Pectoralis minor damaged due to ? c. Iron deficiency anemia
Ans. Fracture of coracoid process Ans. A
Q191.about Clavipectoral fascia which one is most 5. hCG produced in late pregnancy by
appropriate? Ans: Sytotrophoblast and fetal endothelium
Ans. Enclose pectoralis minor muscle. 6. Cells with anti-tumor activity
Q192. About dorsalis pedis artery which one is Ans. Natural killer cells
correct? 7. Ciliary movements
Ans. Situated b/w first and second metatarsal bones. a. Actin/ Myosin
Q193. By damage of which muscle one cannot flex b. Intermediate filaments/Desmin
thigh and medially rotate it? c. Microtubules/Denein arm
A. Sartorius Ans. C
B. Semitendinous 8. Confidentiality can be breached when
C. Piriformis Ans: Patient authorizes you
D. Vastus intermedius 8. Coefficient of variance indicating prefect positive
E. Rectus femoris correlation
Ans: B a. +0. 5
Q194.oclumotor nerve related to which sinus? b. +1
Ans. Cavernous sinus c. -1
Q195. Middle meningeal artery is branch of which d. -0. 5
artery? e. 0
Ans. Maxillary artery. Ans. B
Q196. About posterior triangle most appropriate is? Explanation: Perfect negative correlation r=-1,
Options: perfect positive correlation r=+1, no correlation r=0
A. Roof is formed by paravertebral layer (ANS) 9. Medical ethics
B. Enclosed by deep cervical fascia a. Required for Pakistan medical and dental
C. Lined by sternocleidomastoid muscle council
D. Posterior rami of C2,C3,C4 present in it b. Code of conduct of doctor
Q197.foramen rotundum damaged which structure c. Study of moral aspects of doctor's life
will be severed? Ans. C
Ans. Maxillary branch of trigeminal nerve. 10. Aponeurosis of transversus muscle
a. Forms inguinal ligament
Radiant Notes—FCPS PEARLS Golden File 4 By: Dr. Rafi Ullah
b. Conjoint tendon c. Heart rate increases
c. Lacunar ligament d. Venous return increases
d. Superficial inguinal ring Ans. D
Ans. B 19. Regarding testosterone
11. Regarding pituitary gland a. Produced by Leydig cells
a. Lateral to cavernous sinus b. Intradermal route less effective
b. Derived from endoderm c. Something about oral route
c. Lies in diaphragm sellae Ans. A
d. Pituitary tumor causes Junctional scatoma 20. Which of the following will prevent involution of
Ans. D corpus luteum.
12. Regarding bronchopulmonary segments a. Endogenous chorionic gonadotropin
a. Conical in shape b. Exogenous chorionic gonadotropin
b. Has an independent vein c. Progesterone
c. Has an independent somatic nerve Ans. A
d. Supplied by tertiary bronchioles 21. Safety margin
e. Supplied by terminal bronchioles a. Therapeutic index
Ans. D b. High with digoxin
13. Diphosphoglycerate Ans. A
a. Product of glycolysis 22. A lady has 28th day cycle. She has increased
b. Increased in stored blood estrogen level but no Increase in progesterone for
c. Increases the affinity of hemoglobin for oxygen the last three days.
d. Concentration increases with increase in a. Cessation of menstruation
temperature b. Start of menstruation
e. Shifts oxygen-hemoglobin dissociation curve c. Secretary phase
to the left d. Ovulation
Ans. A e. Regression of corpus Luteum
(There are two types of diphosphoglycerate: Ans. B
-1, 3 diphosphoglycerate and 2, 3 diphosphoglycerate. 23. Regarding levator ani
-1, 3 diphosphoglycerate is an intermediate in glycolysis. a. Contains ischiocavernosis, corpus spongiosum
-2, 3 diphosphoglycerate is decreased is stored blood and Bulbospongiosus
and decreases the affinity of hemoglobin to oxygen. It b. Tripartite muscle pierced by urethra, vagina
shifts oxygen-hemoglobin dissociation curve to the right.) and anal canal
14. A patient has pain on nose and the area between Ans B
mouth and orbit and temporal region. Which of the 24. Injury to spinal cord above sacral segments
following nerve involved a. Spastic bladder
a. Cervical branch of facial nerve b. Atonic bladder
b. Maxillary branch of trigeminal nerve c. Neurogenic bladder
c. Mandibular branch of trigeminal nerve d. Automatic bladder
Ans. B Ans. A
15. Which of the following metabolite of vitamin D 25. Non-Contraceptive benefit of contraceptive pills
can be used to assess the level of vitamin D a. Prevention of ovarian cancer
a. 25 hydroxyvitamin D b. Prevention of breast cancer
b. Cholecalciferol c. Prevention of thromboembolism
c. 1, 25 dihydroxyvitamin D Ans. A
Ans. A 26. Inherited form of endometrial carcinoma is
16. Radiation induced carcinoma after associated with
a. 8 years a. BRCA
b. 2 years b. Breast cancer
c. 10 years c. HNPPC
d. 1 year Ans. C
e. 6 years 27. Mean systemic filling pressure determines
Ans. C a. Venous return
17. Neck of bladder and urethra nerve supply b. Right atrial pressure
a. Sympathetic L1- L2 c. Cardiac output
b. Pelvic splanchnic nerves S2-S4 d. Decreases pulmonary resistance
Ans. A Ans. A
18. during inspiration 27. Regarding micturition reflex
a. Venous return decreases a. Sacral segments of spinal cord S1 and S2
b. Alveolar pressure increases b. Sacral segments of spinal cord S2 and S3
Radiant Notes—FCPS PEARLS Golden File 4 By: Dr. Rafi Ullah
c. Lumbosacral segments of spinal b. Membranous nephropathy
d. Controlled by midbrain c. Membranoproliferative GN
e. controlled by cerebrum d. Interstitial nephritis
Ans. B Ans. C
28. Systemic antifungal therapy is 37. Proto-oncogenes converted to oncogene
CONTRAINDICATED in a. Point mutation
a. Positive blood culture by IV catheter b. Amplification
b. Candida isolated from two sources in body Ans. A
c. Oral thrush and oral candidiasis 38. A man has sweating on a hot day. He has?
d. When only Candida is isolated from fluid a. Positive free water clearance B. Negative free
e. Aspergillus infection water clearance
Ans .A b. ECF hypo-osmolarity
29. Strong VAGAL stimulation c. Urine hypo osmolarity
a. Decrease heart rate Ans. B
b. Increase heart rate 38. Duct of Sylvius between
c. Decrease myocardial contractility Ans: Third and fourth ventricle
Ans. A 39. TNF production by
30. Common in facilitated and co-transport. a. Helper T cells
a. Both for inorganic ions b. Monocytes/macrophages
b. Uphill c. Cytotoxic T cells
c. Require carriers for transport Ans. B
Ans. C 40. Tickle sensations
31. Minimum amount of urine produced in a. A alpha fibers
dehydration to remove waste products from body. b. A delta fibers
a. 500-600 ml c. A beta fibers
b. 200-400 ml d. C-fiber
c. 1000 ml Ans. D (Ref: P. P. Newman neurophysiology pg: 114)
d. 800 ml 41. about right principal bronchus
e. 2000ml a. More wider than left
Ans. A (This is called obligatory water loss must be b. Runs obliquely
between minimum 400-600ml/day) c. More liable to obstruction
32. Hemolytic anemia occurs after a miss-match d. Longer than left
blood transfusion. It can be diagnosed clinically Ans. A
a. Bilirubinuria 41. Pacinian
b. Hemoglobinuria a. Unmyelinated fibers
c. Severe cyanosis b. Unencapsulated
Ans. B c. Detect low frequency vibrations
33. Moderate hemorrhage can be diagnosed d. Phasic
clinically as Ans. D
a. Postural dizziness 42. Most likely about tuberculosis
b. Supine hypotension a. Foul smelling sputum
c. Postural hypotension in 35 years old b. Wheezing never occurs
d. Postural hypotension in 65 years old c. Never involve pleura
e. Supine tachycardia d. Involve posterior segment of upper lobe
Ans. A Ans. D
34. Corticospinal tract 43. Metabolic acidosis with normal anion gap
a. Contains fine touch a. Starvation
b. Decussate in midbrain b. Diabetic ketoacidosis
c. Carry only motor fibers c. Chronic renal failure
d. Unmyelinated d. Carbonic anhydrase inhibitors
Ans. A Ans. D
35. Hyperosmotic over hydration 44. While doing research, statistical tests are done
a. Excess ADH for
b. Bronchogenic tumor a. The results are true
c. Excessive use of diuretics b. The results are clinically significant
d. Cushing syndrome c. To accept the placebo
Ans. D d. Minimize random or experimental error
36. HIV associated nephropathy Ans.B
a. FSGN
Radiant Notes—FCPS PEARLS Golden File 4 By: Dr. Rafi Ullah
45. Most likely about iron absorption b. Secondary
a. Ferric form readily absorbed than ferrous c. Tertiary
b. Most absorption occurs in distal ileum Ans. B
c. Heme is readily absorbed 55. The undifferentiated germ cells in seminiferous
d. Dietary iron is readily absorbed tubules
Ans. C a. Spermatids
Dietry iron is mostly in ferric form which is least absorbable. b. Type A Spermatogonia
Heme contains ferrous form and readily absorbed c. Type B Spermatogonia
46. Left shift of oxygen-hemoglobin dissociation (A spermatogonium (plural: Spermatogonia) is an
curve undifferentiated male germ cell. Spermatogonia undergo
a. Anemia spermatogenesis to form mature spermatozoa in the
b. Hypothermia seminiferous tubules of the testis.
c. Increased temperature There are three subtypes of Spermatogonia in humans:
d. Acidosis Type A (dark) cells, with dark nuclei. These cells are
e. Ageing reserve spermatogonial stem cells which do not usually
Ans. B undergo active mitosis.
47. When each person has equal chance of being Type A (pale) cells, with pale nuclei. These are the
selected in a sample. This sampling is called? spermatogonial stem cells that undergo active mitosis.
a. Stratified sampling These cells divide to produce Type B cells.
b. Random selection Type B cells, which divide to give rise to primary
Ans. A spermatocytes.
48. Cimetidine Ans. B
Ans: Hepatic enzyme inhibitor
49. Inherited by mother only 56. A baby with hypoplastic nails, microcephaly,
a. X genes cleft lip, mother during her pregnancy was exposed
b. Mitochondrial DNA to?
c. Ribosomes a. Fetal-alcohol syndrome
d. Endoplasmic reticulum b. Phenytoin
e. Golgi complex c. Warfarin
Ans. B d. Hypervitaminosis A
Ans. B
50. Thalassemia
a. Abnormal globin chain production 57. Regarding blood supply of bones
b. Unequal chain ratio a. End of Long bones supplied by epiphyseal
Ans. A artery
b. Flat bone supplied by Nutritional artery
51. For the nutritional assessment of a patient, we
Ans. A
assess nutrition from his weight loss and test for
a. Minerals 58. Function of Vitamin A?
b. Vitamins a. Forms germinal epithelium
c. Trace elements b. Maintains Integrity of skin epithelium
d. Albumin and TIBC Ans. B
e. Lipids 59. the most potent anti-oxidant
Ans. D a. Glutathione
52. Which globulin absent in intrauterine life? b. Vitamin E
a. Alpha (α) c. Vitamin A
b. Beta (β) Ans. A
c. Gamma (γ) 60. Murmur of mitral stenosis is best heard at
d. Delta (δ) a. Left second intercostal space
e. Epsilon (ε) b. At apex
Ans. B c. Right 2nd intercostal space
53. The primary oocytes complete their first meiotic d. Left 5th intercostal space
division Ans. B
a. Before ovulation 61. Shortest incubation period
b. 3rd month intrauterine life a. Measles
c. 6th month intrauterine life b. Rubella
d. After fertilization c. Chicken pox
e. Before fertilization d. Mumps
Ans. A e. H. Influenza
54. Pap- smear prevention level Ans. E
a. Primary 62. In liver diseases which enzyme is decreased
Radiant Notes—FCPS PEARLS Golden File 4 By: Dr. Rafi Ullah
a. 5 nucleotides 73. Maternal mortality in Pakistan compared to
b. AST developed countries ?
c. ALT a. 10 times more
d. ALP b. 100 times more
e. Psuedocholinesterases c. 200 times more
Ans. E d. 1000 times more
63. A girl during her menstruation developed high Ans. B
grade fever because she was using tampons. 74. Epidural space
Microorganism responsible for her symptoms is? a. From foramen magnum to L5
a. E.coli b. From foramen magnum to s2
b. Staphylococcus aureus c. From foramen magnum to L1
Ans. B Ans. B
64. The most acute effect of radiation 75. Iron deficiency anemia diagnosis
a. Endarteritis obliterans a. Ferritin + TIBC
b. Skin malignancy b. CBC
c. Lymphoproliferative cancers c. TIBC
d. Desquamation d. Ferritin
Ans. D e. Bone marrow biopsy
65. Water intoxication clinical sign? Ans. A
a. Slow pulse 76. During exercise the blood flow in all zones of
b. Raised JVP lung is like that of
Ans. A a. Zone 3
66. A woman feels lethargic and has dyspnea on b. Zone 2
exertion, tingling sensation in feet and difficulty c. Combination of zone 2 and zone 3
walking. Her Hb is 5 6, WBC 3500, platelets d. Zone 1
120,000.Diagnosis? e. Combination of zone 1 and zone 3
a. Iron deficiency anemia Ans. A
b. Aplastic anemia 77. Regarding pulmonary parameters, what happen
c. Vitamin B12 deficiency to them in pregnancy?
Ans. C a. Vital capacity decreases
 B12 deficiency leads to neurological manifestation and b. FEV1 /FVC ratio decreases
pancytopenia c. Low P02
d. Low PC02
67. A pregnant female needs a lead shield to Ans. D
undergo 78. during ovulation which is not present?
Ans: x-ray chest a. temperature increase
68. A female has undergone hysterectomy, now she b. Vacuolization
has pain in flanks and lower abdomen, oliguria and Ans. B
difficult micturition. Structure damaged? 79. Posterior 1/3 of interventricular septum is
Ans: Ureter involved in MI. Which artery involved
69. Fat embolism a. Right coronary
a. Fracture of long bones b. Left coronary
b. Symptoms appear after 72 to 96 hours c. Marginal
Ans. A d. LCX
e. Left descending artery
70. After surgery, a patient’s wound is not healing.
Ans. A
She is not taking fruits and vegetables. What is the
cause? 80. Glucose transported through placenta
a. Collagen tensile strength is decreased a. Facilitated diffusion
b. Type 3 collagen is deficient b. Passive diffusion
Ans: A c. Simple diffusion
Ans. A
71. Hepatitis- B positive when
Ans: HBsAg and HBcAg positive 81. Increased Specific gravity of urine,
derangements in which function of Kidney has
72. Hepatitis –B is best monitored by
occurred?
a. ALP
a. Concentration
b. AST
b. Filtration
c. SGPT
c. Reabsorption.
d. LDH
d. Secretion
Ans. C
Ans. A
Radiant Notes—FCPS PEARLS Golden File 4 By: Dr. Rafi Ullah
82. Mechanism of action of local anesthesia b. Down syndrome
a. Blockade of sodium channels c. XXY
b. Inhibit resting membrane potential d. Normal female
c. Move K outside cell e. normal male
Ans. A Ans. D
83. In cerebellar lesion, which one among the 91. Strong microbicidal kill
following is present? a. Hydrogen per oxide
a. Dysmetria b. Superoxide
b. Resting tremors Ans. A
c. Muscle paralysis 92. the most striking histopathological feature of
d. Hypotonia nasal polyp
Ans. A a. Squamous metaplasia
84. Saw toothed appearance of glands, vacuolization b. Ciliated columnar epithelium
and edematous stroma? c. Lymphocytes
a. Secretory phase d. Eosinophils
b. Proliferative phase Ans. A
c. Menstrual phase 93. Undescended testis removed because risk of?
d. Ovulation a. Carcinoma transformation
Ans. A b. Infertility
85. Secondary amenorhea history, best Ans. A
investigation? 94. Urothelium in following condition will have great
a. Mid-luteal phase progesterone chance to convert to carcinoma?
b. Estrogen a. A Horseshoe shaped kidney
c. FSH, LH, Prolactin b. Exstrophy of bladder
Ans. C c. Duplicate ureter
86. First stage during onset of puberty? d. Medullary sponge kidney
a. Menstruation Ans. B
b. Breast bud appears 95. Serous cells
c. Pubic hair a. Contain mucin
Ans. B b. Basophilic base
87. The most important function of progesterone is c. Nucleus flat
a. Secretory changes of endometrium d. Eosinophilic
b. Development of alveoli of breasts e. Nucleus at periphery
c. Menstruation Ans. B
Ans. A 96. Mass felt on pubis and pubic bones malformed.
(Progesterone converts the endometrium to its secretory Child getting wet easily
stage to prepare the uterus for implantation.) a. Exstrophy of bladder
88. Breast innermost is involved in carcinoma. b. Absent mesenchyme
LEAST likely drainage into Ans. A
a. Anterior axillary nodes 97. A long scenario for Gram-negative rods on urine
b. Internal thoracic nodes culture
c. Inferior phrenic nodes a. E-coli
d. Posterior intercostal nodes b. Staph-aureus
e. Supraclavicular nodes Ans. A
Ans. C 98. Sexually transmitted disease most commonly by
89. Patient with Adrenogenital syndrome a. Chlamydia
a. High 17-Ketosteroid, high cortisol and low b. Neisseria gonorrhoeae
pregnenolone Ans. A
b. High 17- Ketosteroids, low cortisol and high 99. Cervix lymph drainage
pregnenolone Ans: Internal iliac nodes
c. Low 17-Ketosteroid, high cortisol and high 100. A boy having high grade fever with
pregnenolone Jugulodiagastric lymph nodes enlarged?
d. Low 17-ketosteroid, high cortisol and low a. Parotid gland
pregnenolone b. Palatine tonsils
e. High 17-Ketosteroid, high cortisol and low c. Sub-mandibular
pregnenolone d. Sublingual
Ans. B Ans. B
90. One Bar body present in nuclei/cell of 101. Variance definition
a. Turner syndrome
Radiant Notes—FCPS PEARLS Golden File 4 By: Dr. Rafi Ullah
a. Square of standard deviation c. Adhesion
b. Square root of standard deviation Ans. B
Ans. B 113. Which of the following factors are mostly
102. Genital triangle Nerve supply lost nerve associated with a pathological process rather than a
involved? physiologic process?
a. Genitofemoral nerve a. Epidermal growth factor
b. Ilioinguinal nerve b. Transforming growth factor.
c. Hypogastric nerve c. Fibroblast growth factor
d. Pudendal nerve d. Platelet growth factor Endothelial growth factor
Ans. D e. insulin like growth factors
103. in normal distribution Ans. B
Ans. Mean, mode and median coincide 114. Exomphalos
104. New born baby with recurrent infections? a. Defect in anterior abdominal wall
Ans: IgA deficiency b. Omphalocele
105. In ARDS, edema is due to? c. Gastroschisis
a. Increased capillary permeability d. Defect of umbilicus
b. Increased hydrostatic pressure Ans. A
Ans. A 115. Left renal vein lies posterior to
106.Parturition is initiated by a. Aorta
a. Maternal cortisol b. Renal artery
b. Fetal cortisol c. Third part of duodenum
Ans. B d. Head of pancreas
107. Angiogenesis in wound healing by? Ans. D
a. Anti-trypsin 116. A pregnant female not taking fruits and
b. Caspases 9 vegetables
c. Metalloproteinases a. Anencephaly
Ans. C b. Omphalocele
Ans. A
108. Parathyroid gland supplied by
117. Cerebellar lesion will not lead to
a. Thyroid ima and inferior thyroid artery
a. Muscle paralysis
b. Thyroid ima and superior thyroid artery
b. Speech loss
c. Superior and inferior thyroid artery
d. Superior thyroid artery Ans. B
e. Inferior thyroid artery 118. Myasthenia gravis hypersensitivity
Ans. C a. Type 1
109. Cytokines cause fever by. b. Type 3
c. Type 4
a. Direct action on anterior hypothalamus
d. Type 2
b. C-reactive protein
Ans. D
c. By prostaglandin
d. Thermogenesis 119. Joints between vertebral bodies
e. Neutrophil recruitment a. Symphysis
Ans. C b. Synchondrosis
110. Pars distalis c. Syndesmosis
a. Derived from neuroectoderm Ans. A
b. supplied by superior Hypophysial artery 120. Vitamin B12 carried in blood by
c. Hypothalamic-portal tract a. A .intrinsic factor
Ans. B b. Transcobalamin 1
111. A cells with large granules and larvicidal c. Transcobalamin 2
polypeptides Ans. C
a. Eosinophils 121. Most important mediator of systematic
b. Neutrophils symptoms
c. Monocytes a. IL 1
d. Basophils b. TNF
Ans. A c. Interferon
112. Neutrophils are studied in blood and are found d. IL 6
in periphery with oxygen consumption. This process Ans. A
is necessary for. 122. The pain of diseased ovary is felt on medial
a. Phagocytosis thigh because of involvement of
b. Microbicidal killing a. Femoral nerve.
b. Obturator nerve
Radiant Notes—FCPS PEARLS Golden File 4 By: Dr. Rafi Ullah
c. Pudendal nerve d. Normal saline
d. Perineal nerve e. 5% dextrose
Ans. B Ans. E
123. A person got a stab wound 3 cm vertical on 132. Factor common in facilitated and co-transport.
lateral right side of linea alba. Initially he was fine a. Both for inorganic ions
but then develops hypotension and shock. Which b. Uphill
vessel damaged? c. Require carriers for transport
a. Superior mesenteric artery Ans. C
b. IVC 133. A female with bleeding and coagulation defects.
c. Inferior mesenteric artery Intrauterine death has occurred. Management?
d. Abdominal aorta a. Platelet transfusion
e. Ileal branch of superior mesenteric artery b. Delivery
Ans. B c. Whole blood transfusion
124. Which of the following is from posterior division d. Fluid replacement
of sacral plexus Ans. B
a. Obturator internus and nerve to gamelius 134. Free radicles produced in reperfusion injury
superior after MI by
b. Common peroneal nerve a. Monocytes/macrophages
Ans. B b. Neutrophils
125. A lady taking Glibenclamide for diabetes Ans. B
develops hypotension and her blood glucose level 135. Nucleus ambiguous arises from
was 40. She had acute asthma for which she took a. Pons
some drug. Which drug interacted with b. Mid-brain
Glibenclamide? c. Medulla
a. Salbutamol d. Cerebral cortex
b. Beta blocker Ans. C
c. Steroids 136. A person having anti-sera A and D with
Ans. C agglutinins B. Blood group?
126. Axon reflex a. B-positive
Ans: Triple response b. A-positive
126. Last stage of puberty in girls c. O-positive
a. Puberche d. AB-positive
b. Telarche e. B-negative
c. Menarche Ans.B
Ans. C
127. A lady having symptoms of menopause Gynae Obs 2nd March Evening 2017
Ans. Increased FSH and LH 1. Medial relation of floor of lateral ventricle
128. Pacinian corpuscles a. Hippocampus
a. Unmyelinated fibers b. Caudate nucleus
b. Unencapsulated Ans. A
c. Detect low frequency vibrations 2. Swelling on front of neck move with
d. Phasic swallowing.
Ans. D Ans: pretracheal fascia
129. about right principal bronchus 3. Endotoxin released due to
a. More wider than left a. when bacteria are lysed ans
b. Runs obliquely b. antigen
c. More prone to obstruction Ans. A
d. Longer than left
4. Whip like movement
Ans. A
Ans: uterine tube
130. Tickle sensations
5. Stress related increase glycogenolysis
a. A alpha fibers
a. cortisol
b. A delta fibers
b. glucagon
c. A beta fibers
c. epinephrine
d. C-fibers
Ans. A
Ans. D (ref: P. P. Newman neurophysiology pg: 114)
6. Semen analysis abnormal thing
131. Pure water loss replaced by
Ans: sperm count less than 20 million
a. Isotonic saline
7, newborn with meningitis
b. Hypotonic saline
c. Ringer lactate a. Streptococcus
Radiant Notes—FCPS PEARLS Golden File 4 By: Dr. Rafi Ullah
b. Agalactiae Ans. B
Ans: A 23. sq. root of variance is
8. Longest incubation period a. variation
a. HEP B b. standard deviation
b. HIV c. median
Ans: B d. accuracy
9. 90% of anthrax have Ans. B
Ans: skin disease 24. The best approach to patient is
10. Study done on male patient where STD with a. start with open ended question
penile ulcer compared to non-penile ulcer on b. start with close ended question
Investigation gonococci was negative. What’s the c. start by asking presenting complains
cause of STD? d. Start by asking his name age
Ans: Chlamydia Ans. D
11. Risk of malignancy 25. During sexual excitement in female fluids are
Ans: more than 10 years secretedby vaginal walls mainly due to
12. Main uterine support. a. adrenaline
Ans: transverse cardinal ligament. b. bombesin
13. Case of burns in which skin graft not needed c. GnRh
decided on d. relaxin
a. skin appendages e. parasympathetic stimulation answer
b. granulation tissue Ans: E
c. connective tissue 26. Second messenger cAMP
Ans: A a. ADH II
14. Regarding bronchiole b. Cholecystokinin.
a. absent cartilage c. Calcitriol.
b. bronchiole completely develop d. Gastrin
prenatally Ans: A
Ans. A 27. Part of adrenal gland responsible for
15. Long scenario of laparotomy in which regulating eosinophils and lymphocytes
surgeon see chocolate cyst treatment of a. zona fasciculate
choice. b. zona Glomerulosa
Ans: Danazole c. adrenal medulla
Ans. A
16. During parturition enhance myometrial
activity is due to increase level of 28. Cystic hygroma related to
a. prostaglandin Ans: disease of lymphatic vessels
b. oxytocin 29. Patient of IHD and essential HTN having
Ans: B pedeal edema reason
17. about breast a. inc hydrostatic pressure
Ans: divided into 15-20 lactiferous duct b. Dec oncotic pressure
Ans. A
18. Maximal increase in extracellular volume or
full body volume or full body 30. platelet function
Ans: hypertonic saline a. clot formation
b. clot retraction
19. A boy who drink 2 liter distilled water will
c. clot organization
have
a. Inc ECF Ans: B
b. Dec osmolarity 31. Female with fever since 10 days got chest
Ans: A pain on auscultation some rustling sound
20. Old scenario of earth quake audible (point Of confusion by cpsp)
Ans: situational crises a. Myocardium ans
b. Pericardium
21. Thirst will be stimulated by
Ans. A
a. angiotensin 2
b. aldosterone 32. Highest protein content
Ans. A a. HDL
b. LDL
22. Standard deviation
c. Chylomicrons
a. is sq. of variance
d. VLDL
b. is sq. root of variance
Ans. A
c. mean is not considered
d. median is the central part 33. Best function to check renal function.
Radiant Notes—FCPS PEARLS Golden File 4 By: Dr. Rafi Ullah
a. serum urea b. HIV
b. creatinine clearance c. HSV
c. serum creatinine Ans. A
Ans: B 48. Cell which present antigen
34. mid trimester erythropoisis a. cytotoxic CD8 cells
a. liver b. macrophage
b. yolk sac Ans: A
c. spleen 49. Primitive cell for killing tumor cells
Ans: A a. NK cells
35. Most common big vessel anamoly b. CD8
a. PDA Ans. A
b. situs inversus 50. Thromboxane present in
c. ASD Ans: platelet
Ans: A 51. HIV positive but AIDS not proven CD4 cell
36. Vitelline duct remnant 800 got pyogenic SOM the causative
Ans: meckel diverticulum organism
37. Lymphatic flow decrease in a. pneumonitis juruveci
a. inc in venous pressure b. streptococcus pneumonia
b. inc arterial pressure c. staph aureus
c. inc protein leakage Ans. C (CD4 count is the trick)
Ans: A 52. EBV is diagnosed in which of the following
39. Which will increae GFR disease
a. moderate constriction of efferent arterioles Ans: burkits lymphoma
b. severe constriction of efferent arteriole 53. Amyloid is deposited mainly
Ans: A Ans: kidney
40. Similarity b/w nitroglycerine histamine and 54. Most commonly genetic anamoly
isoproterenol Ans: multifactorial
a. tachycardia 55. Lowest limit of spinal cord in children
b. postural hypotension Ans: Lowe border of L 3
Ans: A 56. Person had swelling on upper arm due to
41. Marked Inc in Cardiac Output some injury swelling remained 3 months but
a. pregnancy resolve In 1 year what was cause of cause of
b. exercise swelling
c. anxiety a. Hypertrophy
d. Extremes of temp b. Hyperplasia
Ans: B c. Neoplasia
42. Esmolol d. Dysplasia
a. Dec elimination in renal failure Ans. A
b. Dec elimination in liver failure 57. Blood supply of bundle of HIS
Ans. A Ans: RCA
43. During laparotomy near ovarian fossa which 58. S1 ans S2 are different what is specific in
structure is prone to be injured? former
Ans: internal iliac artery c. increase duration ans
44. Which cell is present on leucocyte? d. increase in frequency
a. LFA 1 e. increase in pitch
b. VCAM Ans. A
Ans: A 59. Alpha methyl Dopa MOA
45. Reversible change in which there is change a. inhibits angiotensinogen to angiotensin to
in size and polarity angiotensin
Ans: dysplasia b. it inhibits norepinephrine for post ganglionic
46. 4 yrs. old child having per rectal bleeding neuron to act
with mucus biopsy shows some Ans: B
inflammation 60. Maximum anti cholinesterase inhibitor which
a. hemartomatous can travel brain
b. hyperplastic a. physostigmine
Ans: A b. neostigmine
47. Most common cause of Ca cervix c. pyridostigmine
a. HPV Ans. A
Radiant Notes—FCPS PEARLS Golden File 4 By: Dr. Rafi Ullah
61. Case of pulmonary T.b with cavitation shows 75. Ca breast diagnosed surgical removal done a
calcification. Reason? small mass removed treatment after After
a. dystrophic calcification surgery
b. metastatic calcification Ans: Tamoxifen
Ans. A 76. Cervical mass 3.5 cm lymph node unilateral
62. Chances of neoplasia palpable and metastasis
a. protooncogenes a. T2 N1 M1
b. apoptosis b. T1 N2 M1
Ans. A Ans. B
63. Ectopic thyroid harmone produced by 77. 25 yrs. old mother of two children now
a. sq cell Ca of larynx having sec amenorrhea she noticed milk
b. small cell Ca discharge from ophthalmic changes also
Ans. A present. Diagnosis?
64. Infant diagnosed to have worm infestation Ans: Hyperprolactinoma
physician did CBC which conformed 78. Two tailed test???
diagnosis by Presence of Ans: No options recalled here is explanation for two
a. Eosinophilia tail test
b. IDA A two-tailed test allows you to determine if two
Ans: A means are different from one another.
66. Scenario of pregnant lady which develop In other words, a two-tailed test will taken into
some skin change just like her uncle who is account the possibility of both a positive and a
HCV +And cirrhosis … which are those negative effect.
changes? 79. Swelling on neck which moves on
a. Spider telengectasia and palmer erythema respiration. Infection is in??
b. Spider telengectiasisand hyperpigmentation Ans: pretracheal infection
Ans: A 80. Stratified cubital epithelium
67. Subacute endocarditis Ans: large glands of salivary glands
Ans: streotococus viridian 81. Increase in age will decrease
68. Ejection fraction calculation .. CO 5.5 litre HR a. HR
70 b. MAP
EDV 160 c. RV
a. O.5 d. TVC
b. 0.55 Ans: A
Ans. A 82. Deep inspiration causes?
69. C wave shows Ans: inc compliance in pulmonary vasculature
a. closure of semilunar valve 83. Gamma globulin form in
b. opening of semilunar valve Ans: plasma cells
c. closure of tricuspid mitral valve 84. There was short case of acid base
d. opening of tricuspid and mitral valve imbalance ph 6.3 CO2 50 HCO3 30.
Ans. C Diagnosis?
70 . Fetal Hb Ans: uncompensated respiratory acidosis
a. Alpha 2 beta 2 85. Tractus solitarious
b. Higher affinity or content of o2 as compared to Ans: taste pathway
adult 86. Fast adaptation
Ans: B a. golgi tendon
71. Scenario in which ther was RBC slide in b. merkels
which they show thin streak of iron c. ruffinis
Ans: hypochromic and microcytic d. hair containing parts of body
72. Band syndrome. Cause? e. Krause cprposcles
Ans: oligohydromnios Ans: E
73. Increase homocystinuria 87. Pars distalis contain
Ans: deficiency of Vit b 12 Ans: pituicytes
74. Most imp investigation for IDA 88. Product of GFR and plasma conc of
a. serum ferritin substance
b. serum iron and TIBC a. FF
Ans: A b. filtration co efficient
c. filtration load
Ans. C
Radiant Notes—FCPS PEARLS Golden File 4 By: Dr. Rafi Ullah
89. Hering buere reflex will lead to inhibitor of Ans.B
a. Dorsal respiratory group 104. Death due to hanging due to
b. apneustic centre Ans: odontoid process of C2
c. pneumatic centre 105. A boy standing in hot sunny day heat loss is
Ans. A by
90. Central receptor respond to a. activation of post hypothalamus
a. inc H+ conc in arterial blood b. piloerecti
b. inc CO2 in arterial blood c. Cutaneous hyperemia
c. inc H+ in cerebral interstitial fluid Ans. C
Ans. B 106. Apex of lung reach
91. Vit D 3 is called a. 2.5 cm above sternoclavicular joint ans
a. plant origin b. 4cm above sternoclavicular joint
b. animal origin c. 2.5 cm above midclavicular joint
c. 1,25 (OH)2 Ans. A
Ans. C 107 blood supply of bundle of HIS
92. Posterior surface of bladder Ans: RCA
Ans: denonvillers fascia 108. Insensible water loss
93. in partial thyridectomy posterior Ans: 200-400 ml
thyroidectomy done carefully because?? 109. How good a test at correcting excluding
a. parathyroid gland people without the condition is feature?
b. recurrent laryngeal nerve a. specificity ans
c. superior thyroid artery b. sensitivity
Ans. A c. negative predictive value
94. Medial relation of inferior horn of lateral d. positive predictive value
ventricle Ans. A
a. caudate nucleus 110. about primary active transport
b. hippocampus a. uphill movement
Ans. B b. downhill movement
95. over extension of neck lead to Ans. A
a. UMN lesion in upper limb 111. Most common organ involved in amyloidosis
b. LMN lesion in upper limb Ans: kidney
c. UMN in loWer limb 112. Parent bring their baby with difficulty in
d. LMN in lower limb taking milk and nasal regurgitation
Ans. B Ans: palantine shelves
96. Response of tapping of patellar tendon by 113. Thyroid hormones increase in pregnancy
tunning fork will lead to due to
Ans: contraction of quadriceps femoris muscle a. TBG
98. Bias study b. inc harmone synthesis
Ans: sample error c. fetal thyroxine level
99. Deep inspiration is associated with increases Ans. A
in spinal anesthesia. Reason? 114. The vitelline ducts form
Ans: due to cardiac depression a. ductus venosus
100. Rete ridges waala repeated question b. duodenum
a. Verrocus carcinoma c. meckel diverticulum
b. Squamous cell carcinoma d. portal vein
Ans: A (Ref: Philippine journal of pathologyDr. Manas e. urachus
Bajpai)
Ans. C
101. 18 yrs. old girl height 150 cm wt 70 kg hair
115. Iodination of tyrosine takes place in
distribution on biody FSH 52 and LH 40 inc
a. principle cell ans
testostero inc estrogen diagnosis ?
b. parafollicular cells
Ans: PCOD
Ans. A
102. 25 years old lady with sec amenorrhea with
116. Inc collagen synthesis which vitamin is
raised FSH and LH. what is cause?
required
a. autoimmune disease
Ans: Vit c
b. PCOS
Ans: B 117. Graded action potential
a. all or none
103. OCP discontinued due to presence of
b. incremental
a. migraine
c. non decremental
b. cervical neoplasia
Radiant Notes—FCPS PEARLS Golden File 4 By: Dr. Rafi Ullah
Ans. B Ans. B
118. Decrease lymphatic drainage 133, Aorta
a. inc venous obstruction Ans: Origin of renal artery from L2
b. heamorrhage 134. Sinusoid of liver
Ans. A Options not remembered
120.longest incubation period 135. Lateral boundry of deep inguinal ring
a. HIV a. conjoint tendon
b. HEP B b. transversalis fascia
Ans: A Ans. B
120. In prolong exercise will get glucose from 136. Regular insulin is
a. fatty acid from adipose tissue a. short acting enzymes
b. glycogenolysis in liver b. short acting chain mechanism
Ans: B Ans: B
121. Scenario of pregnancy in which Hb in 1st 137. Anterior frontanelle
trimester was 11.5 but at term it become 9.5 a. formed by fusion of 3 bones
what is cause? b. closed by 3rd month
Ans: physiological change c. vault of skull is more then size of face of body
121.central tendency 16, 18 20, 22, 24 median in this Ans: C
data is 138. Patient with fever with chills and rigors
Ans: 20 passing black urine. Some team came and
123. Autosomal dominant took Test and conform
a. only females are effective Ans: Malaria
b. complete penetrance 139. prostaglandin E2 F2
c. heterozygous Ans: vasodilation
Ans. B 140. Diaphragmatic hernia
124. Hereditary spherocytosis Ans: incomplete pleuroperitoneal memb
Ans: defect in memb cytoskeleton 141. Biopsy of gland with serrous acini
125. Fibroblast a. parotid gland
Ans: amorphous substance b. mandibular gland
126. best investigation to check renal function c. sublingual gland
a. serum creatinine Ans. A
b. serum urea 142. how you would identify ketone bodies
c. creatinine clearance a. acetoacetate
Ans. C b. acetyl acetate
127. Mid trimester erythropois is Ans. A
Ans: liver 143. Role of Vit D
128. post adrenergic / sympathetic a. reabsorption in kidney
Ans: miosis b. iron absorption
129. Lady with severe Abd pain diagnosed as c. germinal epithelium
mass in lt fallopian tube WBC Inc on biopsy Ans. A
a. monocyte/macrophage 144. A lady went to jeddah got oral mucosal
b. fibroblast vesicae
Ans. A Ans: HPV 1
130. Very thin wall tube with endothelium and 145. Maximum Inc in volume
smooth muscle 3-4 layers which one is from Ans: hypertonic saline
the Following. 146. Muscle of pelvic diaphragm
a. Venules Ans: puborectalis muscle
b. Arteriole 146. pelvic diaphragm
c. Medium size arteriole a. superficial muscle
Ans: C b. through which urethra vagina and rectu m
131. Increase causes of artherosclerosis passess
a. type b disorder Ans A (other option were not correct at all)
b. cholesteriama 148. Tall man with normal seminal fluid secretion
Ans. B but azoospermia defect is in
132. Conscious desire of thirst Ans: sertoli cells
a. gastric distention 149. Sertoli cells produce
b. hypovolemia Ans: MIF
Radiant Notes—FCPS PEARLS Golden File 4 By: Dr. Rafi Ullah
150. Parents were worried for thir son about he b. HIV.
has difficulty in passing urine? c. HSV.
Ans: ventral shaft of penis Ans: A
151. Cervical Ca lymph nodes involve 6) Infant diagnosed to have worm infestation
Ans: int and external iliac lymph nodes physician did CBC which confirmed diagnosis by
152. Nerve involved in cavwernous sinus Presence of:
Ans: abducent nerve a. Eosinophilia.
153. Disease in progress to terminate in Pakistan b. IDA.
a. polio Ans: A
b. T.b 7) Viruses produce Neoplasia by alteration in:
Ans. A a. Protooncogenes.
154. Reaabsorption in kidney takes place in which b. Apoptosis.
segment c. DNA
a. major calyses Ans: A
b. straight convoluted tubules 8) Fetal Hb:
Ans: B a. Alpha 2 + beta 2 chains.
155. Epidural block by reaching ischial spine, b. Higher affinity or content of o2 as compared to
which nerve is blocked adult
Ans: pudendal nerve c. Forms a major fraction of total HB upto 5 years
156’ maximum constriction of ureter is at of age.
Ans: Vesicoureteric constriction d. Has twice concentration of iron than adult.
e. Only type of HB that can be identified in fetal
157. during surgery urinary incontinence due to
blood.
paralysis of some muscle in which region
Ans: B
Ans: urogenital diaphragm
9) The Vitelline Ducts form:
158. Placenta attach but not invade myometrium
a. Ductus venosus.
a. placenta accretta
b. Duodenum.
b. placenta incretta
c. Meckel Diverticulum.
c. placenta pancretta
d. Portal vein
Ans. A
e. Urachus.
Gynae Obs 3rd May 2017Evening Ans: C
Wednesday 10. Coefficient of variance indicating Prefect Positive
1) Person had swelling on upper arm due to some correlation:
injury swelling remained 3 months but resolve In 1 a. +0. 5
year what was cause of cause of swelling: b. +1
a. Hypertrophy. c. -1
b. Hyperplasia. d. -0. 5
c. Neoplasia. Ans. B
d. Dysplasia Explanation:
e. Metaplasia.  Perfect Negative Correlation R=-1
Ans: A  Perfect Positive Correlation R=+1
2) Fast Pain fibers  No Correletion R=0
a. A delta 11 . Radiation induced carcinoma after:
b. A beta a. 8 years .
Ans: A b. 5 years
3) About breast: c. 10 years.
a. Divided into 15-20 lactiferous duct d. 1 year.
b. Extend from 2nd to sixth rib e. 6 years.
Ans: A Ans. C (5 to 10)
4) Hormone that acts via Steroid Pathway 12). Safety margin:
a. ADH (V 2 RECEPTOR) a. Therapeutic index.
b. Cholecystokinin. b. High with digoxin.
c. Calcitriol. Ans. A
d. Gastrin. 13). Stretch reflex is mediated by
e. Thyroid Ans: Gamma motor neurons
Ans: E 14). Shortest Incubation period :
5) Most common cause of carcinoma cervix: a. Measles.
a. HPV (16 AND 18) b. Rubella.
Radiant Notes—FCPS PEARLS Golden File 4 By: Dr. Rafi Ullah
c. Chicken pox. a. Aorta.
d. Mumps. b. SMA.
e. H. Influenza c. Third part of duodenum
Ans. E d. Head of pancreas.
(Explanation; Influenza.....1-4 days. Ans. A
Chicken pox......14-16 days 23). A pregnant female not taking fruits and
Rubella............12-23 days vegetables:
Measles........10 days a. Anencephaly.
Mumps.........14-25days.) b. Omphalocele.
15). A girl during her menstruation developed high Ans. A (Decrease green vegetables....low folic
grade fever because she was using Tampons... acid...neural tube defects --anencephaly.)
Microorganism responsible for her symptoms is? 24).Normocytic Normochromic Anemia is seen in:
a. E.coli a. Iron Deficiency Anemia.
b. Staphylococcus Aureus. b. Acute Blood Loss.
c. Streptococcus Epidermidis c. Chronic Blood Loss.
d. Streptococcus Pyogenes d. Thalassemia.
e. Neisseria Gonorrhea e. Aplastic Anemia.
Ans. B Ans.B Explanation :( in acute blood loss...its
16). 45 years old G3, P2 underwent cholecystectomy, normocytic normochromic, in chronic blood loss
after surgery her wound is not healing. She is not .............initially normocytic normochromic, later on
taking fruits and vegetables. What is the cause? iron deficiency anemia: pathoma)
a. Collagen tensile strength is decreased. 25). Self replicative organelle which is in dividing
b. Type 3 collagen is deficient. stage:
Ans. A (Vitamin C Deficiency.....Decrease a. Mitochondria.
Hydroxylation of Collagen...Decrease Tensile b. Centrioles.
Strength) c. Mitochondria.
17). Maternal Mortality in Pakistan compared to d. Ribosome.
developed countries? Ans: A
a. 10 times more. 26. IgM
b. 100 times more. a. Pentamer with 10 oxygen binding sites
c. 200 times more. b. present in seromucinous secretion
d. 1000 times more. c. can cross placenta oftenly
Ans. B Ans: A
18). Fetal Hemoglobin 27. Glucose transport to intestine
a. Alpha Beta a. Primary Active Transport
b. Alpha Delta b. simple diffusion
c. Alpha Gamma c. Facilitated Diffusion
d. D Beta Delta d. Cotransport
e. Alpha Zeeta e. Counter transport
Ans: C Ans: D (Secondary active Co transport)
19). One Bar body present in nuclei/cell of?? 28. Bladder derived from
a. Turner syndrome. a. Mesoderm
b. Down syndrome. b. Ectoderm
c. XXY. c. Endoderm
d. Normal female. d. Neural Crest
e. Normal male e. Paraxial Mesoderm
Ans. D Ans: C
20). Urogenital triangle Nerve supply lost nerve 29. Transitional zone Bladder derived from
involved? a. Mesoderm
a. Genitofemoral nerve. b. Ectoderm
b. Ilioinguinal nerve. c. Endoderm
c. Hypogastric nerve. d. Neural Crest
d. Pudendal nerve. e. Paraxial Mesoderm
Ans. D Ans: C
21).Parturition is initiated by: 30. Lesion of lateral Hypoyhalamus will lead to
a. Maternal cortisol. a. Inhibition of feeding
b. Fetal cortisol. b. Inhibition of Satiety Centre
Ans. B c. Increase Feeding
22). Left renal vein anterior to: d. Shivering
Radiant Notes—FCPS PEARLS Golden File 4 By: Dr. Rafi Ullah
e. Memory Loss b. Longitudinal Muscle layer in its outer part
Ans: B c. Covered with peritoneal infront of rectouterine
31. Antidote of Paracetamol pouch
a. Acetylcysteine d. supplied by pudendal + Vaginal
b. Nalaxone Ans: B
c. Morphine 39. Female lady came to Gynae OPD and telling you
d. Cobalt she has Facial growth on Chin and Face
e. Lithium a. Hirsuitim
Ans: A b. Polycystic Ovarian disease
32. Nerve placed in Sodium Free water medium. b. Testicular Feminizing Syndrome
Threshold stimulus was given. What will happen c. Turner Syndrome
next? d. Klinefelter Syndrome
a. No Action Potential Ans: A
b. Graded Action Potential 40. Pudendal Nerve root value
c. Depolarization a. S2-S4
d. Repolarization b. S3-S5
e. Decremental c. L3-L5
Ans: A d. S1-S4
33. Spinal Nerves e. S3,S4
a. Have long dorsal Rami & short venytral rami Ans: A
b. Formed in intervertebral foramina 41. Regarding spermiogenesis
c. Dorsal root ganglions are 2nd order neurons Ans: Spermatids to Spermatozoa
Ans: C 42. In Mediolateral Episiotomy muscle to be cut is
34. Ventricular filling is highest during a. Bulbospongiosis
a. First 3rd of Diastole b. Perineal body
b. Middle 3rd of Diastole c. Sphincter Urethra
c. Last 3rd of Diastole d. Ureter
d. Variable, dependent upon Atrial contraction e. Deep transverse Perineal Muscles
e. Constant Ans: A
Ans: A (80% filling) 43. Superior surface of ischio rectal fossa is formed
35. Increased in arterial Hydrogen ion concentration by
will stimulate respiratory centre through a. Levator Ani
a. Central chemoreceptors b. Obturator externus
b. Peripheral chemoreceptors c. Obturator Nerve
c. In aortic and carotid bodies d. Sphincter Ani Externus muscle
Ans: A e. Gluteus Maximus
36. Tricuspid auscultation best heard at Ans: A
a. Apex 44. Difference between Cow milk and Mother’s milk
b. Right lower body of sternum Ans: Mother milk has more Lactose than cow’s milk
c. Axilla 45. pH 7.23, pCO2 36, HCO3 18
d. Left lower body of sternum a. Compensated Metabolic acidosis
e. 3rd Ics b. Metabolic alkalosis
Ans: B c. Metabolic Acidosis Uncompensated
36. Corona Radiata arise from d. Respiratory alkalosis
a. granulosa Cells e. Respiratory acidosis
b. Theca Interna Ans: C
c. Theca Externa 46. Ideal time of Feeding a new born after delivery
d. Secondary Follicle a. After 1 Hour
e. Theca Intermedia b. After half an hour
Ans: A c. after 2hours
37. Relation of Lateral Fornix of Vagina e. After 45 minutes
a. Ureter f. After 2.5 hours
b. Urethra Ans: B
c. Rectum 47. Prophylactic antibodies in operation Theatre
d. Cervix should be given
e. Bladder a. 6 hours before surgery
Ans: A b. 12 hours before surgery
38. What is true about vagina? c. At the time of induction of Anesthesia
a. Simple squamous epithelium d. Just after Surgery
Radiant Notes—FCPS PEARLS Golden File 4 By: Dr. Rafi Ullah
e. Intraoperatively 55. Regarding Lactose
Ans: C a. Glucosyl Glucosidase
48. Most common long bone fracture b. yield glucose and fructose
a. Fibula c. Glucosyl Galactosidase
b. Tibia Ans: B
c. Femur 56. A young married epileptic lady who is being
d. Humerus treated with Phenytoin is interested in having a
e. Scapula baby. She wants to know if any change is required in
Ans: B her Antiepileptic drug used before and during
49. Which among the following intracellular helps in pregnancy. You should advice her to
wound healing. a. Change over to Phenobarbitone
a. Collagen b. Change over to carbamazepine
b. Tyrosine Kinase c. Change over to Valproic acid
c. Dermatan d. Change over to Lorazepam
d. Hyaluronic acid e. Increase dose of phenytoin
Ans: A Ans: A
50. 60year old woman having HRT for 10 years, her 57. Amniotic Fluid Increase with Increase In alpha
sister died due to Breast Carcinoma. Now she is at feto protein
risk of following. a. Aencephaly
a. Ovarian Ca b. Spina Bifida Occulta
b. Endometrial Ca Ans: A
c. Breast Ca 58. How to prevent life threatening complication of
d. Cervical Ca Warfarin
e. Bladder Ca a. Vitamin K
Ans: C b. FFP
51. What is unlikely in Atonic Bladder? c. Vitamin K + FFP
a. Overflow incontinence d. Iv Heparin
b. Neurosyphilis e. Cryoprecipitate
c. Motor supply damage Ans: C (word Immediate was not mentioned in exam.
d. Sensory supply damage If immediate was mentioned Answer would be B)
e. Atony of Bladder walls 59. Regarding Collagen
Ans: D a. Fibers Not visible
52. Myelination of Peripheral nerves of fetus b. Fibers are visble
a. during 10th week c. Fibers are yellow in color
b. End of 11th week d. Ligamentum Flavum
c. Near term Ans: A
d. 6th month 60. Salivary gland receptors
e. 4th month a. Cholinergic
Ans: E b. Muscarinic
53. 05 year old boy visited his grandmother who died c. Cholinergic Muscrinic
of respiratory tract disease 02 weeks before. From 5 d. Adrenergic
days he is having fever and cough. Xray shows Ans: C
Solitary lesion measuring 2mm in right lung middle 61. Hypertensive patient intraoperatively given
lobe and hilar lymphadenopathy. What boy is MgSo4, after some time she had hypotension,
suffering from? tachydycardia, and dyspnea. What to be given
a. Aspergillus a. calcium Chloride
b. T.B b. epinephrine
c. Pneumonia c. Magnessium
d. Small cell carcinoma of lung Ans: A (in exam calcium gluconate was not in option
e. Adenocarcinoma lund rater it was calcium chloride)
Ans: A 62. Regarding gangrene
54. 60 year female had abnormal feeding behavior, in a. Infected by Gram Positive bacteria
MRI lesion was in b. involve lower limb
a. Anterior Hypothalamus c. putrefaction
b. Basal Ganglia d. Coagulative necrosis
c. Lateral Hypothalamus e. Desication
d. ventral Hypothalamus Ans: D
e. Posterior Hypothalamus 63. Insulin secretion is inhibited by:
Ans: C a. Secretin
Radiant Notes—FCPS PEARLS Golden File 4 By: Dr. Rafi Ullah
b. Theophyline a. Hyperkalemia
c. Beta Blocker b. Hypokalemia
d. Glucagon c. Hypernatremia
e. Epinephrine d. Hyponatremia
Ans: C Ans: B
64. Howship Lacunae 72. Iron Storage in Parenchyma of Organ in the form
a. Osteoclasts of
b. osteoblasts a. Hemosedrin
c. Osteoprogenitors b. Ferritin
d. Paget disease c. Transferrin
Ans: A d. Hemochromatosis
65. Regarding Propylthiouracil e. Heme
a. Inhibit Thyroid Hormone Ans: B
b. Inhibitb peroxides 73. Lateral part of Breast drained via.
c. Inhibit Peroxisomes a. Internal thoracic
Ans: A b. Supraclavicular
66. 40 years old woman exposed to massive c. Inferior Phrenic
radiation. It has massive effect on d. Anterior Pectoral
a. Ovary e. Sub Mammary
b. Endometrium Ans: D
c. Cervix 74. Regarding Insulin
d. Breast a. Mother’s insulin crosses placenta
e. Fallopian tubes b. Fetus doesnot synthesize its own insulin from
Ans: A oancreas
66. No valves are seen in c. Insulin helps in growrh of Fetus
a. Ivc Ans: C
b. SVC 75. Aldosteone secretion is increased by
Ans: B a. Hypokalemia
66. 48 hours old newborn suffering from meningitis. b. Decrease Blood volume
Causative agent Ans: B
a. E.coli 76. Bromocriptine
b. Streptococcus a. Enhance dopamine effect on anterior pituitary
c. N. Menigitidis b. Inhibit Prolactin release from anterior pituitary
d. Crytococcus Ans: B
e. Staphylococcus 77. for prescribing a Drug rationally
Ans: B (Group B Streptococcus) Ans: Non Pharmacological measures should preferably
67. Patient has mitral stenosis develops Atrial be tried for minor self limiting conditions
fibrillation. Most suitable finding 78. A lady drone in water. She was taken out and
a. Saw tooth appearance on ECG was resuscitated, develop ARDS. Mechanis,
b. P Mitrale a. interstitial Oedema
c. Irregular R-R interval b. Increase Lung Compliance
Ans: C c. Oxygen causes reversal of symptoms
68. Prevalance d. Hypoxia improves with Oxygen therapy
a. All cases Ans: D
b. New cases 79. Regarding Oogenesis
c. Old Cases a. All primary oocytes complete their 1st meiotic
Ans: A division before Ovulation
69. Cause of Night blindness b. Significant reduction from birth to puberty
a. Retinol c. 1st meiotic division at prophase
b. Retinoic acid d. Same genetic material in primary and
c. Retinal aldehyde secondary oocytes
d. Rhodopsin Ans: A
e. Retinal (explanation: primary oocyte begins meiosis during fetal
Ans: A life...and complete meiosis ...Prior to ovulation....
70. Pulse Pressure Reference ...first aid reprduction chapter.
a. difference between Systolic and Diastolic PP 80. Tachyphylaxis
b. decreases in old age a. diminished response develop rapidly
Ans: A b. diminished response develop gradually
71. Respiratory Alkalosis causes: Ans: A
Radiant Notes—FCPS PEARLS Golden File 4 By: Dr. Rafi Ullah
81. for diagnosis of Gonococci c. Self reward
a. Exudate in Gram stain Ans: A
b. Widal test 90. Hormone that acts via cGMP pathway
c. Blood culture a. ADH (V 2 RECEPTOR)
d. Sputum culture b. Cholecystokinin.
e. PCR c. Calcitriol.
Ans: A d. Gastrin.
82. Transplant rejection e. ANP and NO
a. B cells Ans: E
b. Macrophages 91. Live attenuated Vaccine
c. NK cells a. Tuberculosis (BCG)
Ans.C b. Typhoid and Paratyphoid
83. Atrial Fibrillation leads to c. Tetanus
a. Thrombus d. Whooping Cough
b. Embolus e. Polio
c. Infarction Ans: A
d. bacteremia 92. Incidence of Pre eclampsiais decreased because
e. Oedema of
Ans: B a. Decrease in Maternal and Perinatal Mortality rate
84. Ketogenesis occurs due to b. Decrease in induction of labor
a. Insulin deficiency c. Decrease Oedema
b. Thyroid d. Increase rate of C.section
c. Parathyroid Ans: A
d. Cortisol 93. Laceration on thigh, few weeks after creitations,
e. Growth Hormone patient is hypotensive.
Ans: A a. Toxemia
85. Breast lie above b. Protolysis
a. Pectoralis Major muscle Ans: A
b. Pectoralis Facia 94. Primitive streak mesoderm migrates to anterior
c. Serratus Anterior abdominal wall and around cloaca. A rupture of this
d. latismus Dorsi mesoderm CRANIALLY will produce
e. Deltoid a. Ectopic Anal opening
Ans: A b. Imperforate Anus
86. Regarding Rh Agglutinins c. Rectovesical fistula
a. Predominantly IgM d. Recto vaginal fistula
b. Predominantly IgG e. Exostrophy of Bladder
c. also called Hemolysins Ans: A (Same stem with CAUDAL opening then
d. Doesnot cross placenta during 1st Trimester answer is E)
e. Always present in RH negative blood 95. Arithmatic Average used for population
Ans: B a. Mean
87. Lowest Oxygen saturation b. Median
a. SVC c. Mode
b. IVC d. Variance
c. Aorta e. Prevalance
d. Umblical Artery Ans: A
e. Renal artery 96. Tumor in Superficial Perineal pouch will damage
Ans: A a. Great vestibular gland
88. 68 year old women with purple patches on skin. b. Bulbourethral gland
She had no liver disease, no history of epistaxis or c. Sphincter Urethrae
bleeding, all Labs within normal range d. Deep transverse Perineal Muscles
a. Senile purpura e. Ureter
b. Vitamin K deficiency Ans: A
c. Thrombocytopenia 97. PICA supply
Ans: A a. Dorsolateral of Medulla
89. Lady with 40 LBS overweight for 02 years. Now b. Pons
on weight reduction plan.. Which of the following is c. Midbrain
most useful. d. cerebellum
a. self monitoring e. Cerebrum
b. Self punishment Ans: A
Radiant Notes—FCPS PEARLS Golden File 4 By: Dr. Rafi Ullah
98. Type of cells with no basement membrane e. release neutrophils
a. Ependyma Ans: B (repeated twice)
b. Mesothelium 107: Pus contains
c. Transitional epithelium a. Neutrophils
Ans: A b. Eosinophils
99. Calcitonin c. Dead bacteria
a. given in Vitamin D intoxication d. Basophils
b. Calicum absorption from gut e. mast cells
Ans: A Ans: A
100. A patient receive Treatment for Cancer and 108: Changes that occur after birth
developed, nausea, headache and PR bleeding. a. Ductus arteriosus converted into ligamentum
What treatment most likely received arteriosum
a. Radiotherapy b. Ductus venosus form Ligamentum teres
b. Chemotherapy c. Umblical artery form Median Umblical ligament
c. Neoadjuvant Ans: A
d. Surgery 109. Vitamin used in synthesis of COA
e. Hormonal therapy a. Riboflavin
Ans: A b. pantothenic acid
101: Recurrent Abortion test that should be done c. Cobalamine
a. Coagulation profile d. Thiamine
b. Lupus anticoagulant e. Niacin
c. Anti mitochondrial antibodies Ans: B (vitamin B5)
d. Anti microsomal antibodies 110: Patient present after laparotomy, abdominal
e. Anti histone antibodies distension, tachycardia, hypotension
Ans: B a. hypovolemic shock
102: Buffer which is present in kidney but not in b. Neurogenic shock
body fluids c. Cardiogenic shock
a. Bicarbonate d. Toxemic shock
b. phosphate Ans: A
c. Ammonia 111: Circadian rhythm
d. Urea a. Suprachiasmatic nucleus
e. Blood b. Posterior hypothalamus
Ans: B c. Anterior hypothalamus
103: Glycogen Ans: A
a. complex branched structure 112: When temperature of skin reaches 52 degree
b. Have 1,6 linkage in 1,4 branches centigrade which receptors are stimulated?
c. Storage form a. Pacinian corpuscles
Ans: A b. Meisner’s corpuscle
104: Effect of TCA (tri cyclic antidepressants) starts c. Merkel corpuscles
as analgesic at d. Nociceptive receptors
a. 3-4 weeks e. Ruffini receptors
b. 1-2 weeks Ans: D
c. 1 day 113: Vitamin A
d. after 6 weeks a. Retinol
Ans: A b. Retinaldehyde
105: Major cells Involved in hookworm infestation c. Retinal
are Ans: A
a. basophils 114: About hering beur reflex
b. neutrophils a. Stretch receptors in bronchial wall
c. eosinophils b. Air epithelium
d. mast cells c. J receptors
e. platelets Ans: A
Ans: C 115: Central part of Spinal Fibre was stimulated by
106: Major cells Involved in hookworm infestation a. Group 1b
are b. Group 1a
a. Receptors of IgE c. Gamma fibers
b. release Eosinophilic chemotactic factor d. Alpha fibers
c. shows abundant histamine and heparin Ans: B
d. release basophils 116: About Doctor Patient relationship
Radiant Notes—FCPS PEARLS Golden File 4 By: Dr. Rafi Ullah
a. to maintain dignity 126. 4 year girl suffering from Sorethroat and
b. Visual impediment copious amount of Serosangious discharge from
c. Assurance about condition vagina
Ans: A a. Foreign body
117. Suprarenal gland is supplied by b. Streptococcus
a. Greater Thoracic Splanchnic nerve c. Candidiasis
b. lesser Thoracic Splanchnic nerve d. N. Gonorrhea
c. least Thoracic Splanchnic nerve e. Chlamydia
d. posterior Vagal trunk Ans: B
e. Anterior Vagal trunk 127. Causative agent for Yellowish Vaginal
Ans: A discharge
118. Antibodies in pernicious anemia a. Chlamydia Trachomatis
a. Anti microsomal antibodies b. Trichomonas Vaginalis
b. Anti parietal canalicuar c. N. Gonorrhea
c. Lupus anticoagulant d. Bacterial Vaginosis
d. Anti mitochondrial antibodies e. Toxoplasmosis
e. Anti histone antibodies Ans: A
Ans: B 128: Most common Carcinogen
119. A cell active in protein synthesis will have a. Nitrosamine
a. Dense Prominent nucleolus b. Alcohol
b. Heterochromatic c. Nicotine
c. Eosinophilic Ans: A
d. Golgi 129. A patient with tumor that has metastatized to
Ans A (RER) Para-aortic lymphnodes. She received Hormonal
120: Corticostriate Fibers releae Therapy. After sometime Tumor regressed in size..
a. Glutamate process
b. Dopamine a. Apoptosis
c. Serotonin b. Necrosis
d. Acetylcholine c. Phagocytosis
e. Glucagon d. Pinocytosis
Ans: A e. Radical Injury
121: Staging shows Ans: A
a. lymphovascular invasion 130. about Cholera
b. extent of Invasion a. Transmitted via food and water
c. Pleomorphism b. cause Bloody diarrhea
d. Mitosis c. Vaccination provides specific protection
Ans: B d. Not dangerous
122: Differencebetween Right lung root from left e. usually treated with oral and Parenteral
Lung root antibiotics
a. Number of Primary Bronchi Ans: E
b. No. of Terminal bronchioles 131. Patient complained of Abdominal Pain and
c. No. of arteries persistent vomiting. Cause?
d. No. of veins a. Exotoxin
Ans: A b. Endotoxin
123: Well differentiated squamous cell carcinoma c. Enterotoxin
a. Keratin d. Erthyrogenic toxin
b. Vementin e. Phagotoxin
c. Dextran Ans: C
Ans: A 132. Diagnostic of Malignant tumor
124: After giving Normal saline a. Metastasis
a. Urine osmolality Increases b. Invasion
b. Plasma Osmolality Increases c. Pleomorphism
c. sodium concentration increases in urine d. Mitosis
Ans: C e. increase N/C ratio
Ans: A
125. Turbulence increases with
a. Increase viscosity of blood 133. Change in size, shape and polarity is
b. Decrease Viscosity of blood a. Anaplasia
Ans: B b. Dysplasia
c. Metaplasia
Radiant Notes—FCPS PEARLS Golden File 4 By: Dr. Rafi Ullah
d. Hyperplasia d. Anterior Pectoral
e. Hypertrophy e. Sub Mammary
Ans: B Ans: A
134. Freely filtered through ascending limb of loop 146. 4th Heart Sound
of Henle a. Atrial Systole
Ans: Water b. Ventricular Systole
135. Long stem In the end Mast cells release c. AV Valve open
a. Histamine d. Semilunar Valve closure
b. Bradykinin e. During exercise
c. leukotrienes Ans: A
d. Prostaglandins 147. A type 1 diabetic with persistent morning
e. Hormones hyperglycemia. What should be given?
Ans: A a. Ultrashort acting Insulin before meals
136. Most Important function of HCG b. Regular Insulin with Intermediate Insulin twice
a. Prevent involution of Corpus Luteum daily
b. decrease in Choriocarcinoma c. long acting insulin twice daily
c. mild raise in Molar Pregnancy Ans: C
d. Breast Development 148. End Diastolic Volume depends upon
e. Uterus increase in size Ans: Venous return
Ans: A 149: Use of Terbutaline in prevention of preterm
137. Functional Residual capacity labor causes
a. ERV + RV a. Hypoglycemia
b. VT + IRV b. Hyperkalemia
c. VT + ERV c. Hypernatremia
d. ERV + IRV+ RV d. Acidosis
e. FVC Ans: A (Terbutaline is Beta agonist)
Ans: A 150. Most common Congenital Anomaly of Heart
138. A child with rash and bile stains on diaper a. TOF
Ans: Biliary Atresia b. VSD
139. Superior Thyroid artery is related to c. TGA
a. Recurrent laryngeal nerve d. ASD
b. External Laryngeal nerve Ans: B (PDA wa not in option)
c. Internal Carotid 151. Maternal Mortality calculation 50 mothers died
d. Inferior Thyroid artery in 100000 live births
Ans: B Ans: 5/1000 live births
140. IgM+, HBsAg+, HbeAg + 152. 1st enzyme to appear after 02 hours of MI
Ans= Acute Hepatitis B a. Trop T
141. about Nocireceptors b. CK MB
Ans= Temperature above 52 degree celsus c. LDH
142. Uterus is prolapsed but anus is still intact and d. ALP
it’s in position Ans: B
a. Perineal body 153. Pregnant lady At 34weeks of gestation with
b. Urogenital diaphragm Vaginal bleeding and coagulation problem. First step
c. Pelvic diaphragm of Management
d. Levator ani a. Blood Transfusion
e. External Anal Sphincter b. Heparin therapy
Ans: C c. Platelet Transfusion
143. Biopsy of Introcular tissue shows necrosis and d. Evacuation of uterus
Langhan’s giant cells. Cause is e. Streptokinase
a. Tuberculoid Leprosy Ans: D
b. Histoplasmosis 154. FEV1/FVC ratio
c. Toxoplasmosis a. 80%
d. Tuberculosis b. 60%
Ans: A c. 50%
144. 145. Medial l part of Breast drained via. d. 60%
a. Internal thoracic e. 90%
b. Supraclavicular Ans: A
c. Inferior Phrenic 155. Tumor suppressor gene
Radiant Notes—FCPS PEARLS Golden File 4 By: Dr. Rafi Ullah
a. N-Myc b. Pie chart
b. C-Myc c. Chi square
c. P53 d. Frequency polygon curve
d. C-erb e. Standard Deviation
e. RAS Ans: B
Ans: C 164. Temporomandibular Joint dislocation will lead
156. Patient with skin pigmentation and can not to which muscle paralysis
maintain Balance. MCV 110, Iron 60fl, serum ferritin a. Masseter
600 b. Temporalis
a. Sideroblastic anemia c. Medial Pterygoid
b. Anemia of chronic disease d. Lateral Pterygoid
c. Megaloblastic Anemia e. Buccinator
d. IDA Ans: D
e. G6PD deficiency 165. Content of deep Perineal Pouch.
Ans: C Ans: Bulbourethral gland
157. Pain mediator 166. Inferior rectal is a branch of
a. Bradykinin a. External pudendal
b. Histamine b. internal pudendal
c. Leukotrienes c. External Iliac
Ans: A d. Femoral
158. 13 years old looking like 09 years girl. Mental e. Obturator
retardation. Hormone involved. Ans: B
a. ADH (V 2 RECEPTOR) 167. A patient presented with lesion of Right
b. Cholecystokinin. Hypoglossal nerve lesion.
c. Calcitriol. a. Tongue deviated towards right with atrophy
d. Gastrin. b. Tongue deviated to Right without atrophy
e. THyroid c. Tongue deviated towards left with atrophy
Ans: E (congenital hypothyroidism) d. Tongue deviated to left without atrophy
159. 60 years old male patient case of Ans: A
atherosclerosis was under your treatment with 168. In Mediolateral Episiotomy there is risk of
blockage of coronary artery suddenly brain ischemia rupture of
occurs cause is a. Bulbospongiosis
a. Thrombus b. Perineal body
b. Ischemia c. Sphincter Urethra
c. Embolus d. Ureter
d. Infarction e. External anal sphincter
e. Infection Ans: A
Ans: C 169. EBV is associated with:
160. Ischiorectal fossa abscess drainage will result Ans: Burkitt Lymphoma
in damage to. 170. Pulmonary artery Supplies
a. Inferior rectal nerve Ans: Alveoli
b. Sacrotuberous ligament
171. A female suffering from immune
c. Internal pudendal artery
disease for last 01 year. Having lymphoma and
d. Sacrospinous ligament
Cervical lymph nodes palpable cause is.
e. Obturator externus
Ans: AIDS
Ans: A
172. Essential fatty acis
161. Chronic Hypertension with Pedal Oedema
Ans: Linoleic Acid
a. Increase Oncotic Pressure
b. Decrease Hydrostatic pressure 173. Most common presentation of patient with
c. Increase Hydrostatic pressure Turner syndrome
d. Exudative a. short strature
Ans: C b. XXY
162. Lady with Hyperemesis Gravidarum. What c. Autosomal Recessive
change is most likely seen? d. Increase AFP
Ans: Hypokalemia Ans: A
163. In a study 40% women received HRT, 60% have 174: A Researcher testing efficacy of Penicillin and
not received HRT. The best way to represent such cephalosporins for streptococcal infection.These
study is drugs are prescribed randomly to 02 groups of
a. Bar chart patients A & B. What type of study it is
Radiant Notes—FCPS PEARLS Golden File 4 By: Dr. Rafi Ullah
a. Case control d. Cerebellum
b. Cohort e. Cerebrum
c. Randomized Control Trial Ans: A
d. Cross sectional 189. Posterior Triangle of Neck
Ans: C a. Contain C2,C3,C4
175. A female with warty lesion on vulva b. Posteriorly bounded by Splenius
a. COndyloma Accuminata c. Floor Formed by Trapezius
b. Condyloma Lata d. Roof formed by Investing Layer of Pretracheal facia
c. Papilloma Ans: D
d. Paget Disease 190. Artery deep to Posterior belly of Digastric and
e. HSV Stylohyoid Pass through Submandibular gland is
Ans: A a. Lingual
176. Significant change in viscosity occur during b. Facial
a. Pregnancy c. External Carotid
b. High Altitude d. Internal Carotid
Ans: B Ans: A
178. Inferior layer of urogenital diaphragm is known 191. In SLE cause of Thrombocytopenia is
as Ans: Antibodies against Platelets
Ans: Perineal membrane 192. Patient uses Smokeless tobacco and develops
179. Lymph drainage of Labia Majora submucosal fibrosis. It is
Ans: Medial group of Superficial Inguinal lymphnodes a. Pre malignant
180. Structure with no valve b. Neoplastic
a. SVC c. Hyperplasia
b. IVC d. Metaplasia
c. Great cardiac vein e. Hypertrophy
Ans: A Ans: A
181. Adrenal Medulla derived from 193. Most important function of platelet is
f. Endoderm Ans: Clot retraction
g. Ectoderm 194. Regarding Foramen Ovale
h. Paraxial mesoderm a. Provide Right to Left shunt
i. Neural crest Cells b. Form Fossa Ovalis in mature heart
j. Endoderm Ans: A
Ans: D 195.Falciparum malaria
182. Lady with Vaginal discharge, on Pap smear a. Sporozite
Koilocytes were seen. Causative agent is b. Shizont
a. HSV Ans: A
b. HPV 196. Patient Underwent Lower Segment Cesarean
c. N. Gonorrhea section, patient become hypotensive, pale which
d. Chlamydia artery is injured
e. Trichomonas Ans: Branch of uterine artery
Ans: B 197. Which is a DNA respiratory virus
183. Sudden right Retinal artery occlusion a. Respiratory Syncytial virus
a. Thrombus b. Adeno Virus
b. Embolus c. HSV
c. Giant cell arteritis Ans: B
Ans: B 198. Adrenergic Beta Action
185. Inferior rectal vein causes External a. Renin release
Hemorrhoids covered with skin and drain into b. Sphincter contraction
Ans. Superficial Inguinal Lymphnodes. Ans: B
186. Heparin monitored by 199. Sweat gland
a. PT a. Tubular structure
b. APTT b. Absent in palms and soles
c. INR c. Stratified cuboidal epithelium
Ans: B d. Sympathetic nicotinic or muscarinic receptors
187. Dorsal Nucleus of Vagus is located in Ans: C
a. Medulla 200. Structure damaged close to Phrenic nerve
b. Pons Ans: Pericardiophrenic arter
c. Midbrain
Ans: A
Gynae May 4 2017 Morning 11. Most common cervical carcinogen:
a. HPV.
1. External jugular vein formed by Retromandibular
b. IUCDS.
and
c. Herpes 2.
a. Facial vein.
d. HCV.
b. Post auricular vein.
Ans: A
c. Superficial temporal vein
Ans: B 12. Female after radical mastectomy swelling and
edema in anterior upper side of arm cause of
2. Double layer structure:
edema?
a. Golgi
a. Na retention
b. Nucleus
b. Lymphatic obstruction.
c. RER
c. Decrease hydrostatic pressure.
Ans: B (all three are double layered)
d. Angiosarcoma
3. 2nd arch derivative:
Ans: B
a. Stylohyoid ligament.
13. Ascitic tap.
b. Stylomandibular ligament.
a. E-coli
c. Sphenomandibular ligament.
b. Pseudomonas Aeruginosa.
Ans: A
c. Staphylococcus Aureus.
4. Body of Hyoid bone level
d. Streptococcus Pyogenes
a. C3
Ans: A
b. C4
14. One was cervical cancer stage 4. Lymph node
c. C5
most likely involved will be:
Ans: A
a. Internal iliac nodes.
5. Trachea is supplied by which nerve?
b. External iliac nodes.
a. recurrent laryngeal nerve
c. Internal iliac node+ External iliac node
b. glossopharyngeal nerve
Ans: C
c. intercostal nerve
15. Ovarian carcinoma lymphatic drainage involve:
Ans: A
a. Paraortic Lymph node.
6. Male pelvis type
b. Internal iliac nodes.
a. Android
c. Inguinal lymph nodes.
b. Anthropoid
d. Popliteal lymph nodes
c. Gynecoid
Ans: A
Ans: A
16. Micturition reflex
7. Thorn prick:
a. Self-generative reflex.
a. Staphylococcus aureus.
b. Control by sacral segments of spinal
b. Pseudomonas.
cord.
c. Staphylococcus Epidermidis
c. Involve parasympathetic sacral
Ans: A
segments.
8. Right border heart on x-ray
Ans: B>A
a. SVC.
17. Ca rectum Pain transmitted to posterior thigh
b. Right Atrium + SVC.
nerve damage:
c. Right ATRIUM + Right ventricle.
a. Sciatic nerve.
d. Right Auricle.
b. Femoral nerve.
Ans: B
c. Obturator nerve
9. Atrial fibrillation:
d. Sacral plexus.
a. Pulse deficit.
Ans: D
b. P wave prominent.
18. Ampulla of Rectum nerve supply:
c. Regular rhythm
a. Inferior rectal nerve.
Ans: A
b. Inferior hypogastric plexuses
10. Infection pretracheal fascia
Ans: B
a. Anterior mediastinum.
19. Middle rectal artery is branch of
b. Posterior mediastinum.
a. Internal iliac artery.
c. Middle mediastinum.
b. Pudendal Artery.
d. Superior mediastinum.
c. Common iliac artery. d. Thirst
d. Femoral Artery. Ans: A
Ans: A 30. Laparotomy done for ovarian mass removal.
20. Pelvic splanchnic nerve? Artery injured includes:
a. Supply small intestine. a. Internal iliac artery.
b. Parasympathetic nerve b. External iliac artery.
Ans: B c. Common iliac artery.
21. Respiratory alkalosis: cause includes: d. Ovarian artery.
a. Narcotics overdose Ans: A
b. Pulmonary edema. 31. Nerve in cavernous sinus is:
c. GBS. a. Abducent nerve.
Ans: A b. Occulomotor nerve.
22. There is a patient with Jugulodiagastric lymph c. Trochlear.
node enlargement it is due the d. Mandibular nerve
a) pharyngeal tonsil Ans: A
b) palatine tonsils 32. Uterus connect with labia majora by:
c) parotid a. Round ligament.
d) mandibular b. Broad ligament.
e) Sublingual c. Cardinal ligament.
Ans: B d. Suspensory ligament of ovary
23. Proto-oncogenes converted to oncogene Ans: A
a. Point mutation 33. Which of the following prevents Ovary to fall:
b. Amplification a. Uterus
Ans. A b. Broad
24. Cohort study: c. Pelvis
a. It can be retrospective. Ans: B
b. Cohort tells about outcome of disease 34. Female with FSH level increase but low
Ans: A Estrogen:
25. Patient prescribe medicine for clear watery a. primary hypogonadism.
discharge then develop thin curdy discharge, b. Secondary hypogonadism.
mouth taste changes drug: c. FSH secreting tumour.
a. Metronidazole d. Teritary Hypogonadism
b. Clotrimazole. Ans: A
c. Tetracycline. 35. Pain on defecation??
Ans: A Ans: Inf. Rectal nerve
26. Richest source of Vit k? 36. Hepatitis B route:
a. tubers a. Orofecal
b. cereals b. Sexual
c. green veg c. Direct contact
d. pulses Ans: B
Ans: C 37. Pregnant Female present at 20 weeks gestation
27. Fever: with bleeding and passage of vesicles
a. IL1 Ans: Beta HCG raised
b. Prostaglandin 38. Agonist:
Ans: A a. Have affinity and efficacy
28. Water intoxication: b. Only intrinsic activity
a. Slow pulse. c. Bound by ionic bonding have prolonged
b. Distended neck. effect
c. Pulmonary edema. Ans: A
d. Increase alertness 39. Malignant histopathology:
Ans: A a. Pleomorphism
29. Posterior hypothalamus: b. Invasion
a. Shivering. c. N/C ratio.
b. Temperature. Ans: A
c. Hunger. 40. Which is diagnostic of dead cell:
a. Cell swelling. c. sympathetic innervation
b. Pyknosis. d. parasympathetic
c. Karyolysis. Ans: D
d. Karyorehexis. 51. After hysterectomy, Asthmatic patient, post-
e. Cell shrinkage operative pain killer
Ans: C a. Pethidine
41. Mitochondria b. Diclofenac
a. Self-replicating c. Ketorolac
b. Division of mitochondria Ans: C
c. Fragmentation of mitochondria 52. Relation of Lateral Fornix of Vagina
d. Replication of RNA a. Ureter
Ans: A b. Urethra
42. Sub-acute endocarditis: c. Rectum
a. Streptococcus viridian. d. Cervix
b. Staphylococcus aureus. e. Bladder
c. Streptococcus Fecalis. Ans: A
d. Streptococcus pneumoniae 53. Hypertrophy?
Ans: A a. increase in size of cell
43. Single gene Autosomal dominant: b. increase in number of cell
a. Vit D resistant rickets Ans: A
b. Achondroplasia 54. Which of following is rapidly adapting?
c. Thalassemia a. Ruffini
d. Duchene Muscular dystrophy b. Golgi tendon
Ans: B c. Pacinian
44. Multiple Lung abscesses d. Krause receptor
a. H. influenzae e. Merkel disc
b. Staphylococcus aureus. Ans: C
c. Streptococcus
55. If parasympathetic cut which of following will be
d. Pseudomonas. affected mostly
Ans: B
a. Sweat glands
45. Plasmodium falciparum: b. Salivary glands
a. Flagellate. c. Muscle of GIT
b. Sporozoite. d. Skeletal muscles
c. Schizont. Ans: C
Ans: B
56. Smooth muscles
46. Natural anticoagulant by mast cell:
Ans: Slow and sustained contraction (ANS)
a. Heparin.
57. Oxytocin:
b. Plasminogen.
a. Increase uterine contraction.
c. Warfarin
b. Increase BMR.
Ans: A
c. Control blood glucose levels
47. Secondary ossification:
Ans: A
a. Epiphysis.
b. Metaphysis. 58. Primary Gravida is in second stage of labor,
c. Diaphysis. ischial spine more prominent type of pelvis:
d. Epiphyseal plate. a. Android pelvis.
Ans: A b. Anthropoid pelvis.
c. Gynecoid pelvis.
48. Carbohydrates structure
d. Platypelloid pelvis.
Ans: maltose monosaccharide
Ans: B
49. Lumbar triangle bounded anteriorly by
a. External abdominal oblique 59. Anal canal Blood supply:
b. Latissimus dorsi a. Superior rectal artery.
Ans: A b. Median sacral artery
c. Middle rectal artery
50. Distention of rectum sensation carried by?
Ans: A
a. superior rectal nerve
b. inf. Rectal nerve
60. A 60 year old male living at high altitude 12000 b. Increase plasma cld osmtc pres
feets height. Previous history of MI have c. Lymphatic obstruction
developed edema in lower limbs and bluish d. Hydrostatic pres Inc.
colour cause? Hb was 12 mg only: Ans: C
a. Venous thrombosis. 70. Increased intermediate filament in carcinoma
b. Cardio- respiratory failure. a. Keratin
c. Anemia. b. Vesmin
d. Thrombo-embolism. c. Desmin
e. Arterial blockage. Ans: A
Ans: B 71. Cause of edema in CCF?
61. Drugs that decrease sugar level: a. Inc permeability
a. Adrenaline. b. Inc hydrostatic pressure
b. Thiadize. c. Dec oncotic pressure
c. Guanethidine Ans: B
d. Chlorpromazine 72. Factor II associated with
Ans: C a. Vit. K
62. Nerve damage in pelvic wall laterally: b. Vit. D
a. Pudendal nerve. Ans: A
b. Obturator nerve. 73. Bradykinin is formed?
c. Femoral nerve a. Platelets
Ans: B b. by action killikein
63. Obturator internus muscle nerve supply: Ans: B
a. Sacral plexus. 74. Alpha adrenergic causes
b. Femoral nerve. a. Mydriasis
c. Pudendal nerve. b. Bronchodilation
d. Pudendal nerve along with perineal Ans: A
branch of s4 75. During pregnancy which decrease:
Ans: A a. Minute ventilation.
64. Fused spinous process of sacrum is called: b. CO
a. Median Sacral Crest. c. GFR.
b. Sacral hiatus. d. Blood Volume.
c. Sacral promontory. e. PCO2
Ans: A Ans: E
65. 20 yr. Primigravida died during delivery. Most 76. Decrease in old age:
common cause in our country: a. Pulse deficit.
a. Malnutrition. b. Heart rate.
b. Sepsis. c. Pulse pressure.
c. Hemorrhage. d. MAP.
Ans: C Ans: B
66. Bladder Ca is caused by 77. 16 yr old girl with epistaxis and menorrhagia. PT,
a. Schistosoma mansonin Aptt and BT all prolonged. Cause:
b. Schistosoma Haematobium a. Disseminated intravascular coagulation.
Ans: B b. Von willibrand disease.
c. Liver disease.
67. Middle rectal artery is branch of
Ans: B
a. Inferior mesenteric artery
78. Pubic hair development:
b. Internal iliac artery
a. A: Testosterone.
Ans: B
b. B: Diepiendrostenidione.
68. Ejaculation
c. C: Estrogen.
a. Sympathetic
d. D: Progesterone.
b. Parasympathetic
e. E: Adrenal Androgens.
Ans: A
Ans: E
69. Rt side radical mastectomy post patient develop
79. Which hormone responsible for female growth
painless RT arm swelling due to
and lack in males:
a. Vascular insufficiency
a. 17- 20 lyase.
b. Aromatase 99. Most common fertilization site
c. Cholesterol desmolase Ans: ampulla
d. 5 HT reductase. 100. Uterus functional layer which shed down
e. Adrenal androgens. during menstruation
Ans: B a. compactum and spongiosum
80. Epicardium blood supply b. capsularis and basalis
Ans: coronary Ans: A
81. Endogenous muscle relaxant 101. Hyperparathyroidism
Ans: endorphin Ans: decrease phosphate
82. pelvic splanchnic nerves 102. Kidney buffer
Ans: are all parasympathetic Ans: Phosphate
83. Alpha adrenergic 103. Which of the following in maternal serum for
Ans: contraction of radial muscle of eye rubella should be an indication for termination of
84. Proto-Oncogenes mutation pregnancy
Ans: ionizing radiation a. IgG
85. Ionizing radiation b. IgM
Ans: leukemia c. IgE
86. Scenario of carcinoma pt on chemotherapy. Size Ans: B
of abdominal mass reduced 104. Fibrinogen role in
Ans: Apoptosis Ans: clotting
87. Lymphatic obstruction 105. Pelvic diaphragm muscle
Ans: cystic hygroma Ans: pubo-coccxygeos
88. Broad ligament part attracts with uterine tube is 106. Increased ketogenesis with deficiency of
a. Mesovarium. Ans: Insulin
b. Mesosalpinx. 107. Sacral hiatus
Ans: B Ans: unfused S4 S5
89. Widest epidural space 108. Fused spinous processes of sacrum also
Ans: L2 called
90. Young Patient has dyspnea and anemic Ans: median sacral crest
symptoms... MCV 100, MCHC 30, Hb 9. What is 109. Tramadol
the type of anemia? a. Strong Mu receptors
a. Normocytic Normochromic. b. Weak kappa receptors
b. Hypochromic Normocytic c. Weak Mu receptors
c. Macrocytic Normochromic Ans: C
Ans: A 110. normal PT normal aPTT only
91. Which of the fallowing is carcinogen thrombocytopenia
a. Asbestos Ans: ITP
b. Cisplatin 111. Supra- renal gland supplied by:
Ans: A a. Thoracic sympathetic trunk.
92. both extra and intrapelvic extension b. Right related to central tendon of
Ans: Round Ligament of uterus diaphragm.
93. Upper boundary of pelvic inlet c. Hypogastric plexus on both sides.
Ans: pubic crest. d. Relation to IVC on either side.
e. Both related to celiac plexus.
94. Ureter cross uterine artery at which level
Ans: A
Ans: bifurcation of common iliac (ANS)
112. Pubic hair development:
95. Pregnant lady, cat pet
a. Testosterone.
Ans: Toxoplasmosis
b. Diepiendrostenidione.
96. Chlamydia c. Estrogen.
Ans: intracellular obligate d. Progesterone.
97. Diphtheria immediate diagnosis e. Adrenal Androgens.
Ans: special stain Ans: E
98. Pregnant 6 weeks HCG positive RIF pain 113. A 12 year patient with thick sputum. She
Ans: ectopic have this problem for last10 years.
Hyperinflation on chest x-ray 60seg, d. Good bedside manners.
20bands....11esinophill, Sputum consists of Ans: A
eosinophils: 124. Glycolipid is necessary for which enzymatic
a. Bronchial asthma reaction:
b. Cystic fibrosis a. Absorption of iron
c. Mycoplasma pneumoni. b. Absorption of cobalamin
d. Interstitial pneumonitis c. Conversion of retinol to retinoic acid
Ans: A Ans: C
114. Scenario of pregnant lady which develop 125. Anterior fibers of levator ani:
some skin change just like her uncle who is HCV a. Ureter.
+ and cirrhosis … which are those changes? b. Bulb of vestibule.
a. Spider telangiectasia and palmer c. Perineal body
erythema Ans: C
b. Spider telangiectasia and 126. Tabes dorsalis bladder Atony reason:
hyperpigmentation a. Dorsal column ganglion effected.
Ans: A b. Sacral nerves effected.
115. Essential fatty acid c. Peripheral nerves effected
Ans: Linoleic acid Ans: A
116. Cause of Night blindness 127. Pre-terminal bronchioles contain epithelium:
a. Retinol a. Simple Squamous Epithelium.
b. Retinoic acid b. Simple Colummar Epithelium.
c. Retinal aldehyde c. Pseudostratified ciliated Epithelium
d. Rhodopsin Ans: C
e. Retinal 128. RBC’S in CSF:
Ans: A a. Subarachnoid Hemorrhage.
117. In dividing cell Chromosome defect cancer b. Subdural hemorrhage.
cause INCREASE production of cell which c. Epidural hemorrhage.
enzyme d. Intracerebral hemorrhage.
Ans: Telomerase Ans: A
118. Normal decent of ovary occur in early life 129. Semi-comatose pregnant patient with high
which support ovary from further descent and B.p, came in emergency, (Eclampsia patient),
remains within pelvis (Abdomen): what to give to control b.p ?
a. Uterus. a. Methyldopa.
b. Labia majora b. Hydralazine.
c. Obliquity if inguinal ligament. Ans: B
d. Narrow internal ring. 130. Fluid which has 0.5 gram proteins:
e. Broad pelvis pelvic brim a. Saliva.
Ans: A b. Lymph.
119. Neurohypophysis vesicles are stored in c. CSF.
Ans: Nerve ending Ans: B
120. Pt renal end stage and anemia you will give 131. hydroxy proline formed by
Ans: erythropoietin Ans: ascorbic acid
121. Female with waddling gate on walk and 132. Dysplasia is seen in:
pelvic dislocation which cause injury to the a. Epithelium.
muscles includes nerve damage: b. Bone.
a. Superior gluteal nerve. c. Cartilage.
b. Inferior gluteal nerve d. Muscle.
Ans: A Ans: A
122. Release of prolactin inhibited by 133. Hypothyroidism, on FNAC lymphocyte
Ans: progesterone Ans: Hashimoto
123. Doctor patient relationship: 134. Pelvic diaphragm synonym with
a. Active listening. Ans: pelvic floor
b. Having sound medical knowledge. 135. Metaplasia is seen in
c. Updating skills.
Ans: Epithelium Ans: acidic drugs
136. Metaplasia is most likely 149. dizygotic twins
Ans: Functional and morphological change Ans: its own placenta, Chorion, amniotic cavity
137. Lesion of sympathetic tract...leads to 150. benign growth
circulatory collapse: a. hamartoma
a. Vasomotor shock. b. Warthin’s tumor
b. Vasomotor tone. Ans: B
c. Increase vascular permeability. 151. Ethambutol Anti TB drug causing
d. Cardiac output increase or decrease. Ans: optic neuritis
Ans: A 152. The plane from sacral promontory to the
138. Testosterone inner posterior of pubic symphysis
Ans: Descent of testes in late fetal life Ans: obstetric conjugate
139. Ovarian fossa and lateral pelvic wall 153. Preterminal bronchiole epithelium
structure damage likely Ans: psuedostratified columnar
Ans: internal iliac artery 154. A person doing regular exercise what factor
140. Middle aged man had RTA with pelvic bone causes in this condition to increase his
fracture = respiratory rate
Ans: Severe hemorrhage Ans: stretch receptors in lung
141. increased B.p with decreased heart rate 155. Systolic phase corresponds to
Ans: ICP Ans: QT interval
142. Which is most related to thrombin formation 156. Left leg pain temp lost which tract affected
1st step is Ans: Right lateral spinothalamic
Ans: platelet adhesion 157. vasopressin release increased by
143. Endogenous substance used to measure Ans: decrease atrial pressure
GFR 158. Postganglionic fibers present in
Ans: creatinine Ans: all spinal nerves
144. Antigenic property of bacteria due to 159. Pressure maximally dissipated at
Ans: carbohydrate structures in cell wall a. Arterial capillary
145. In normal human blood b. venular capillary
Ans: IgE with mast cells Ans: A
146. Nerve supply of obturator internus 160. Hypoxia causes
Ans: sacral plexus Ans: Pulmonary vasoconstriction
147. tall man + gynecomastia 161. Acute inflammation
Ans: Klinefelter syndrome Ans: Local response of tissues
148. Albumin bind with
14. data collected on the basis of presence n absence
Radiology 1st March morning of vomitting
paper... Ans. Nominal
1. Cervical branch of facial nerve supplies 17. A specific age group was selected than
Ans: Platysma subgroups were form for collection of data
2. cervical CA Drains into Ans. Stratified random
a. External iliac 18. Sickness defined as
b. Internal iliac a. What psychian thinks
Ans: B b. What patient feels
3. prostate supplied by c. What's clinically diagnosed
a. Superior vesical Ans. B
b. INF vesical 19. HLA DR4 Association
Ans. B Ans. RA
4. A boy presented with hemorrhoids doc 20. Cemintidine n sucralfate are not given
examined INF rectal vein...That drains into togather because
a. Internal iliac vein Ans. Cemintidine decreases surlfate Metabolism
b. Internal pudendal 21. Receptor effect
c. External pudendal a. Heparin anatagonism by protamine sulphate
Ans.B b. Mannitol osmotic diuretic
5. thyroid is invested in c. Lebtalol causes bradycardia
a. Prevertebral fascia Ans: A
b. Pretracheal fascia 22. Heparin required which of the following
Ans. B anticoagulant for its function
6. bulbar urethra rupture urine extravasates into Ans. AntiThrombin 3
a. Superficial pouch 23. Oat cell carcinoma secrets
b. Deep pouch Ans. ACTH
Ans. A 24. Renal faliure organ hypertrophic
7. syphilis diagnosis sample would be taken from Ans. Parathyroid
Ans. Penile ulcer 25. Isthmus lies at level of
8. Thymus blood supply Ans. C2, 3, 4 tracheal rings
Ans. Internal thoracic n inf. thyroid artery 26. Pain of small intestine referred to
9. lymph of duodenum drains into Ans. T10 umbilicus
Ans. Celiac n sup mysenteric 27. a boy fall from top story building and complaints
10. arterial supply of 1st part of duodenum of urinary incontinence segments involved...
a. Sup pancreatocoduodenal Ans. S2, 3, 4
b. INF pancreatocoduodenal 28. At L4 Level not contained
c. Right gastric a. White matter
d. Gastroduodenal b. Grey matter
Ans. A c. Ventral horn
11. pain sensations from peptic ulcers by d. Ant horn
a. Lesser splanic e. Lateral horn
b. Least splanic Ans. E
c. Greater splanic 29. Structures at hepatodudenal ligament
Ans. C (It was asked thrice) Ans. CBD, portal vein, hepatic artery
12. a boy had head injury after that his eye pulsate 30. Deep cervical is a branch of
with heart beat it is due to relations a. Costocervical
a. Cavernous sinus with internal carotid artery b. Thryocervical
b. Cavernous with external carotid c. Axillary
c. Cavernous with extraocular muscles Ans. A
Ans. A 31. During playing football a boy had fracture of
13. Suprapubic catherization done from the neck of fibula, artery damaged is
Surface not cover by peritoneum a. Ant tibial artery
a. Post b. Commonn peroneal artery
b. Sup Ans. A (if asked nerve then ommon peroneal nerve)
c. Ant 32. Most common cause of subacute
Ans. C endocarditis
Ans. Strep viridian
33. After appendicotomy patient feel yellowish 47. A young boy presented with swollen big toe
discharge from wound with urea level 40 the diagnostic test would be
Ans. Actinomycin isralae Ans. Synovial fluid examination for sodium urate
34. Appendicular artery is a branch of crystal.
a. Middle colic 48. Diabtes with complication
b. Sma a. Dry gangrene
c. Ilioceacal b. Wet gangrene.
Ans. C Ans. B
35. Antipyrine n heavy water use to measure 49. Person will die after bilateral cut down of
a. Total body water a. Trigeminal
b. ECF volume b. Vagus
c. ICF volume c. Occulomotor
Ans. A Ans: B
36. to increase ECF vol the solution to be infused 50. Cardiac plexus contains
a. Hypotonic saline Ans: Both sympathetic and parasympathetic
b. Hypertonic saline 51. Gastric motility increases by
Ans. B a. Gastrin
37. Ductus deference opens into b. Secretin
Ans. Ejaculatory ducts Ans. A
38. Lymph from glans n corpora drains into 52. Portl hypertension abdomenal vessels dilated
a. Deep ingunial they are connected with portal vein through
b. Sup inguinal Ans. Para umbilical vein
c. Paraortic 53. Uncinate process of pancrease
Ans. A contain/related to anteriorly
39. Acrosomal Cap of sperms formed by a. SMA
Ans. Golgi apparatus b. IMA
40. Mediator of inflammation c. AORTA
a. 5-HT3(SEROTONIN) Ans. A
b. Bradykinin 54. Head of pancrease lies ant to
Ans. A a. Aorta
41. Pregnant female with deranged values of b. IVC
LFTS went to remote area 4 weeks back cause c. SMA
Ans. Hep E Ans: None of the option seems correct, here is the
42. Cheliosis n dermatitis explanation
Ans. Riboflavin Head of Pancreas
43. 25 year gravida 4 presents with weakness n 
occult blood in stools artery
a. IDH 
b. Anemia of chronic disease Neck:
Ans. A 
44. Pregnant female taking ferrous sulphate only vein and beginning of portal vein.
would have deficiency of.. Uncinate process
a. Vit E  nteric vessels
b. Vit A 
c. Folic Acid 55. Warthy lesion on vulva... it would be.
d. Vit D a. Condyloma
Ans. C b. Sq cell CA
45. Female, Hb 5.6 tlc 3500 platelate 1,20,000. Ans. A
Weakness during walk tingling in feet.what is cause? 56. Awoman has cervix biopsy with large nuclei
a. Def.folic acid. she came again after 6 months for follow up lesion?
b. Vit.b12 def a. Dysplasia
c. Aplastic anemia b. Metaplasia
Ans. B c. Hsv
46. Heat loss due to endotracheal intubation Ans. A
a. Evaporation 57. Liver Ca tumor markers would be
b. Conduction Ans. Alpha fetoprotein
c. Convection
Ans. A
58. A young boy have resected colon due to some Ans. B
growth his father died of same prob the lesion would 73. Apex zone one of lungs
be a. Ventilation high
Ans. Adenomatus b. Perfusion high
59. Cruciate ligament of knee joint are c. V/q increases
Ans. Intracapsular d. More dead space
60. Erbduchne paralysis is due to trauma to Ans. D
Ans. Upper trunks of brachial plexus 74 A child is continuosly breathing in a plastic
61. TEBRCULIN test negative due to bag his resp rate has increased best option
Ans. Patient taking immunosuppressive a. increase in alveolar PCO2
62. To increase vitamins in diet a dietraian advice b. increase in arterial PCO2
to increase in diet c. increase in PO2
a. Milk Ans. B
b. Eggs 75. Commonest presentation of vitamin a
c. Green vegetables deficiency
Ans. C Ans. Night blindness
63. Mother blood group A children with blood 76. 2nd ordr neurons of nociceptrs r locatd in
group O and AB father blood group would be a. post root ganglion
Ans. B b. post horn
64. Calcium in smooth muscles present in Ans. A
Ans. Calmodulin 77. Pyramidal corticospinal tract
65. Iron binds with a. Contain cortical neurons
Ans. Heptoglobulin b. Axons arise from parietal and pre motor
66. After surgery bone regeneration occurs c. Projecting fibers
through d. Decussate at corpus callosum.
Ans. Periosteum Ans. A
67. Node of Ranviar deficient at 78. Most common feature of renal tubular
a. Myelin sheath acidosis
b. Axoplasm a. hyperkalemia
Ans: A b. Proteinuria
c. Increase urea n normal creatinine
68. Pt.is living in desert and use plenty of water.
Ans. A
From which segment most of water absorb?
a. Pct 79. Most common vavlve disease in
b. Dct interventricular septal defect.
c. Collecting duct a. Tricuspid
Ans. A b. Aortic
c. Mitral
69. Thyroid gland originate from
Ans. A
a. Foregut ventral side
b. Foregut medial side 80. Infective endocarditis sequence
c. Endoderm a. Damage valve, thrombosis, BacterMia,
d. Mesoderm perforation
e. 4th pharyngeal pouch b. damage valve, Bactremia, thrombosis,
Ans. C perforation
Ans. A
70. Pt has heaviness n pain in perinium on
examination have extarnaloss in introitus .aesa Kuch 81. Severe diarrhoea leads to
tha scenario Ans. Metabolic acidosis with normal anion gap...
a. Uterosacral ligament 82. Na/k ATPase pump is stimulated by?
b. Levator anii a. Adrenaline
c. Perineal body b. Noradrenaline
Ans. A c. Dopamine
71. Fibrocartilage? d. Insulin
a. Amorphous material Ans. D
b. Present in disc 83. A young girl has only 3 days history of
Ans. B jaundice reticulicyte 10% HB 5mg/dl
72. Seasmoid cartilage present in a. Etceric
a. Layrnx b. hemolytic anemia
b. Ala of nose Ans: B
c. Biceps femoris 84. Nucleus disappear in which stage
a. pronormoblast b. ADH
b. reticulocytes c. Both correct
c. normoblast Ans. C
Ans. C 94. 1st sign of pencilling allergy
85. 3 months baby presented with critically ill a. Rash
condition...PH: 7.2 PCO2:32, HCO3:14, Na: 142, K: 2.5. b. Fever
Diagnosis??? c. Joint pain
a. Addison disease d. Bronchospasms
b. Compensated resp acidosis Ans. A
c. diabetic acidosis 95. Alpha adrengic stimuliaon
d. lactic acidosis a. Pupils dilatation
e. Renal tubular Acidosis b. brochodilation
Ans: E c. vasodilation
86. Smooth endoplasmic reticulm abundant in Ans. A
a. nerve cells 96. In lactation no pregnancy occour why?
b. cardiac cells a. Releas of prolactin
c. skeletal cells b. Inhibition of gonodrotropins
d. Gonadal cells c. Progesterone
e. Endocrine d. Estrogen
Ans. D Ans. B
87. Septomarginal (modrator band) z attached to 97. Right sided paralysis n left sided facial
a. Rt atrium deviation
b. Rt ventrical a. Pons
c. Lt ventrical b. Internal capsule
Ans. B Ans. B
88. Patient with hypotention, CO is 2.lit/min. jvp is 98. After C-section patient feel pain in lumbar
2.mm. Cause region with urinary symptoms structure injured
a. hypovolemic shock a. Ureters
b. cardiac temponade b. Uterine artery
c. Congestive cardiac failure. Ans. A
d. septic shock 99. A police man took opiate n cannabean what
Ans: A finding u wil see in him?
89. A student observing 1st time a surgery got a. Euphoria (ans)
collapsed with decrease BP Cause b. Denial
a. Increased cardiac output c. Antisocial
b. Decrease cardiac output d. Depression
c. Decrease TPR Ans. A (do not confused it, its asked sometimes that
Ans. C what would be he first response then click denial, if
90. Patient with parotid gland Swelling. Stones is asked what you will find him then click euphoria)
found in duct. Which proceess is involve 100. Medial epicanthus ligament attach at?
a. apoptosis a. Medial canthal
b. necrosis b. Nasal
c. Degeneration c. Fronto nasal
d. Hyalinization d. Ant and posterior canthal
Ans. A Ans. D
91. Secretions from zona gromelulosa is 101. What is most true aboul parallel breast
regulated by nodes?
a. Angiotensin II a. Internal nodes to parasternal
b. ADH b. Axillary node to axilla...
c. Renin Ans. A
d. Potassium 102. Which of following are present at mossssstttt
Ans. C posterior of diaphragm?
92. When drug is bound to albumin a. Esophagus,aorta,thorasic duct
a. Active b. IVC, Aorta, thorasic duct
b. Inactive until get activated in liver c. Left vagus, azygos,aorta
Ans: B d. Right vagus, thorasic duct,aorta
93. Which of followong acts by cAMP e. Aorta, thoracicduct, azygous vein
a. TSH Ans: E
103. Most of organs in embronic life laid down c. Meningiocel
during which d. Spina bifida occulta
a. 4 month e. Omphelocel
b. 6 month f. Sacrococcygeal tumor...
c. 3 month Ans. D
d. 6th week ( 115. Portal vein formed by SMV and Splenic vein at
Ans. D lower border of
104. At 7_8 week mostly gonadal ridges of both a. Duodenum
sexrssame...but what point wil difrent in boys b. Transverse mesocolon
a. Testes descend with gebrnaculum regression c. Spleen
b. Regression ogguberculum id called swelling d. Neck of pancrease
method.. Ans. D
c. 3rd option b guberculumtha. 116. True about fallopn tubes?
Ans. A a. 6cm
105. After birth what happens? b. Laterally move to ovaries medially
a. Truncus arteriosis become ligamentum arteriosis c. Infudibulum has tortous structure..
b. Foramen close and Dec LV pressure d. Infundibulum have fibria
Ans: B Ans. D
106. Basillar artery divided into 117. 55 yr old male suffering from DM since 25
Ans: Posterior cerebral arteries yrs...Kidney issue?
107. Pt has itching, Patient cant scrach his back Ans: Kimmel syndrome
due to lesion of? 118.. Stagnant hypoxia?
a. Serratus anterior a. Polycythemia
b. Latismus dorsi b. Cynide poisning
c. Teres minor c. Co monxidepoisning
Ans. A d. Severe anemia
108. Aorta n pulmonary can't separate defects lies Ans. A
in 119. A lady in kitchen cut her radial artery
Ans. Truncus arterious accidently which mechanism wil reduce pressure in
110. In carpal tunnel syndrom due to ulnar nerve vessel and stop bleeding?
involvement which muscle involved? a. β fibers
a. Oppones pollicis b. Post ganglionic C
b. Abductor policis c. Post ganglionic B
c. Flexor carpi ulnaris d. Delta
d. Extensor carpi ulnaris Ans: B
Ans. C 120. During accident a lordotic thin man got a
111. Supination n extension is finction of? wound of 3 cm to right and below umbilicus initially
a. Radial + musculocutaneus vitals were normal but aftet sometime bplow..Which
b. Ulnar + musculocutanrious structure involved?
c. Axillary + radial a. IVC
d. Medial + radial b. Aorta
Ans. A Ans. A
112. Musle of lower trunk of brachial plexus? 121. Midbrain lesion involve which nerve
a. Supraspinitus a. Occulomotor
b. Deltoid b. Vagus
c. Teres minor c. Trigemminal
Ans. A d. Vagus
113. Foreign body get obstructed in Ans. A
a. Right segmental 122. Patient had MI Inferior wall infarct on ECG.artery
b. Right main bronchus involved?
c. Left inferior a. Rt.marginal
d. Left apical b. LAD
e. Left main Ans. B
Ans. A 123. Patient can't see laterally can't close his eyes
114. In which of gollowing vertebrae involved but and has hearing problem damage would be at
NO spinal cord involved? a. Cerebrpontine angle
a. Cordoma b. Mid brain
b. Anencephaly Ans. A
124. Atrioventricular node (A.V node) supplied by? b. Pressure more than dura
a. RCA at origin of aorta c. Absorb by choroid
b. Left coronary artery d. Sometime enter in lymph...
c. RCA because it is present in posterior e. Slightly acidic than plasma
interventricular septum Ans: E
Ans. C 135.. 9% N/S solution
125. A female presents with Arterial disease on Ans. 9mg in 1000ml of distilled water
examination u find pulsatile mass on abdomen it 136. Regarding intercollateral vein in chest wall?
originates from a. 2 in mumber
a. T9-T11 b. Supply chest muscles
b. T12-L2 c. Give rise lateral cutsneous branches
c. S2-4 d. Run in Subcoastal grove
d. S5-4 Ans. B
e. L2-S4 137. Wound strength provided by
Ans. B a. Type 1 collagen
126. Pnuemothorax? b. Type 2
Ans. Ipsilateral lung collapsed ipsilateral chest sprung out c. Type 3
127. DVT lodge in Ans. A
a. Pulomnary artery 138. Base of bladder oblique ridges called?
b. Pulomnary vein a. Rice bar.
Ans. A b. Marciers ridge
128. Female got stab wound in c. Interurteric ridge
chest...Hyperresonant and, absent sound on right... Ans. B
Mediastinum towards left 139. Cholangiography done of comn hepatc
a. Right pneumothorax duct..a sructure z vusible wich winds towrd its target
b. Hemothotax organ..
Ans. A a. Cystic DUCT
129. Femoral artery pulsation feel at b. Right hepatic
a. Mid inguinal point. c. CBD
b. Mid point at ingunial ligament Ans. A
c. Yarmtlbdonoaik he baathai 140. Lateral fornix of vagina is closely related to
Ans: A a. Uterine artery
130. Boy high grade fever.took anti malarials now b. Uterine tube
presented with cola color urine lethargy n weakness c. Ureters
a. G6PD d. Round ligament
b. Black water fever Ans. C
c. Hemolytic anemia 141. During football game a boy injured and now
Ans. A unable to dorsiflex foot..dorsal sendation is
131. A pt has Inc glucose but after taking fructose normal...nerve involved?
glucose reading normal which is involved? a. common peroneal nerve
a. Glucose6 phosphate b. Deep peroneal
b. Tyokinasa c. Sup peroneal
c. Pyruvate d. Ant tibial
Ans. A Ans. A
132. Portal hypertension vein involve 142. During game boy injured which area of
a. INF epigastric eyeball that now there is vertical diplopia and
b. Esophageal enophthalmos?
c. Imv a. Apex
d. Smv b. Base
Ans. C c. Roof
133. A pt has white thick patch in mouth which cnt d. Floor
be removed by pressure the lesion would be Ans. D
a. Leukoplakia 143. Vertcal movement of thorasic cage....
b. Erythoplaklia a. Contraction of diaphragm
c. Candida b. Bucket handle.
Ans. A c. Movement of costovertebral joint
134. True about CSF? d. Movement of costochondrial joint
a. Protein more than plasma Ans. A
144. Female having hb5 GGT 156. Circulation doesn't compromise during
raised...Hepatomegaly,pruitis xanrhomas test would moderate exercise
be a. Coronary
Ans. Antimidrocondrial antibody b. Cerebral
145. Aik or bhetha same prutits with deranged lfts c. Renal
specially GGT n raised ALP d. SK muscle
a. Extra hepatic cholelithiasis Ans: B
b. Intrahepatic 157. Regarding basal ganglia true is
c. Gall stones a. Internal capsule is medial to GP
Ans. A b. IC is medial to red nucleus
146. Metasis occurs due to c. Caudate forms neostriatum
Ans. Loss of E-cadherin Ans. A
147. What specific action to be performed to check 158. A male had injury after that presents with loss
cerebellum lesion... of 2 point discrimation at T8 on same side...N pain
a. Ataxia loss on contralateral side lesion would be at...
b. Dysdiadocokynesia a. A spinothalamic tract at T8
Ans. B b. Hemi section of cord at T12
148. Suprapubuic inscion Bleeding is due to c. Springomyila.
a. Sup epigastric Ans. A
b. INF epigastric 159. A baby came with intestinal obstruction
c. Deep circumflex hushrprung disease...Segment involve
Ans. C a. S2,3,4
149. Dull, steady, epigastric slightly left sided pain b. S4,5
radiating to back is c. L2,3
a. Duodenum d. L4,5
b. Pancrease Ans. A
c. Gall bladder 160. Cells that release transform into mast cells n
d. Stomach release histamine
Ans: B a. Monocytes
150. In all patients of anthrax the lesions of organ b. Neutrophils
must invlove something like that c. Macrophages
a. Skin d. Basophillis
b. Heart e. Eiosnophills
c. Lungs Ans: D
Ans. A 161. Erythroid stem cell generates in bone marrow
151. True hermaphrodite xx . ambiguous genitals under the influence of
a. cause a. Thyroxine
b. Congenital adrenal hyperplasia (ans) b. Growth hormone
Ans. B c. Growth factor
152. Boys with long arms n short body typical d. Erythropoietin
scenario of achondroplasia e. Cortisol
Ans. Autosomal dominant Ans: D
153. Pulse pressure increase due to 162. 1st response of body against bacterial infection
a. increase SV a. Complement system.
b. Cardiac out put b. Skin
c. TPR c. Opsonization
Ans. A d. Chemotactic
e. No relase
154. Basophillia in cell is due
Ans: B
a. Golgi
b. Smooth Endoplasimic Retulum 163. In injured tissue the chemotactic factor is
c. Ribosomes a. Histamine
Ans. C b. C3a
c. C5-9
155. Insulin dependent transport of GLUCOSE in
d. Interlukeins b
a. Skeletal muscle
e. TNF
b. Heart
Ans: D
c. Kidney
Ans. A 164. Surgically breast tissue
a. hyperrpohy
b. separated by interlobar septum b. Interlobar
c. emgere at common opening like that c. Medullary rays
Ans: B d. Calyces
165. CA of colon metasis 1st to Ans. C
a. Liver 175. Stratified epithelium is
b. Kidney a. Present in lungs
c. Lymph nodes b. Have same height
d. Heart c. Lies at same base ans
Ans. C Ans. B
166. True about femoral artery 176. HELLP syndrome includes
a. Medial to femoral nerve Ans. Thrombo cytopenia, elevated leucocyte, hemolysis
b. Lies in medial compartment 177. Inf.rectal drains in
c. Lies Infront of inguinal ligament a. Int.pudendal
d. Enters thigh by pass through adductor canal b. Ext.pudedal
Ans. A Ans. A
167. True about esophagus 178. After surgery of bone regenration occour
a. Pass left to arch of aorta throuh?
b. Lies ant to left bronchus Ans. Periosteum
c. Commence at cricoid 179. Outer layer of nerve fiber regenrate at?
d. Pass through medial arcuate ligament of Ans. Epineurium
diaphragm
180. Erbs palsy lesion involve?
Ans. C
Ans. C5 and c6
168. In an adult the common site of urinary stone
obstruction 181. Mediator of allergy?
a. Uretropelvic junction a. Bradykinn
b. Where it enters bladder b. 5 hydroxy tryptophan
c. Where it crosses internal iliac Ans. B
d. At pelvic brim 182. Pyramidal corticospinal tract?
Ans. A Ans. Dynamic response.
169. Most common cause of cancer 183. True about fallopn tubes?
a. over expression of protooncogenes a. 6cm
b. reparable damage to DNA b. Laterally move to ovaries medially
Ans: A c. Infudibulum has tortous structure
170. For aretrial blood gass sample taken d. Ampulla have fimbria
a. Whole venous blood Ans. D
b. Whole arterial blood 184.Pt is working in smokeless cigarette factory he
c. Centrifuged plasma has a nodule on biopsy sub mucous fibrosis what
d. Heparinized could be lesion?
Ans. D a. -dysplasia
171. Dipthrria toxoid hypersensitivity reaction b. -metaplasia
a. Type-1 c. -precancerous lesion
b. Type-2 Ans. C
c. Type-3 185.Pregnant lady deranged lfts go to remote area ?
d. Type-4 Ans. Hep E
Ans. C 186 .25 year para 4 has occlut blood in stool?
172. Plasma and interstitial osmolarity diff.. Ans. Iron deficiency anemia
a. I mosm 187 Young boy with inflamed big toe febrile, raised
b. 3 mosm TLC ,UTI,urea level raised.For Diagnosis?
c. 4 mosm a. Monosodium urate crystal in synovial fluid
Ans. A b. Synovial fluid for culture and sensitivity
173. Which one is diffrent from others? Ans. B
a. Osteoclast 188. Cruciate ligament of knee joints
b. Osteoblast a. Intracapsular
c. osteocytes b. Surrounded by synovial membrane
d. chondroblast Ans. A
Ans: A
189. Oat cell CA tumor markers
174. Collecting duct? Ans. ACTH.
a. Projection fibers
190 Circulation doesn't compromise during
moderate exercise 3. Fast vibration
a) Coronary Ans. pacinian corpuscles
b) Cerebral 4. Necrosis in brain
c) Renal Ans: liquefective
d) SK muscle 5. 60 year old on off fever lethargy
Ans. B dyspnea on excertion. HB 7.8 urea 140
191.Na/k atpase pump is stimulated by? creatinin 3
a) Adrenaline a) Erythropoitein def
b) Noradrenaline b) Acute renal failiure
c) Insuline c) Malnutrition
Ans. C Ans: A
192.Sever diarrhoea ? 6. Autoimmune pancreatitis antibody
Ans. normal anion anion gap metabolic acidisis a) igA
193 Infective endocarditis scenario. Damage? b) igG
Ans: Damage= thrombus= bacterimia= perforation Ans. B
194. Pyramid corticospinal tract? 7.) Infection in cavernous sinus.
a) Contain cortical neurons. a) connects to which vein in brain
b) Axons arise from parietal and pre motor b) superior ophthalmic
c) Projecting fibers c) inf ophthalmic
d) Decursate at corpus callosum. Ans: C
e) Olivary 8) Unlocking of knee joint
Ans. E Ans. popliteus
195. Superior thyroid artery is related to 9) Arm could not be raised to comb lesion in
a) ELN Ans: serratus ant (trapezius was not in option)
b) RLN 10) Person works in tyre factory presented with
Ans. A painless hematuria cause is
196. s4 heart sound produced by? a) aromatic amine
a) Rapid ventricular filling b) nitrites nitrates
b) atrial systol Ans: A (smoking wasnt in option)
Ans. B 11) Does not cross blood brain barrier
197..primary syphilis dx can be made by scraping ? a) Dopamine
a) mouth ulcer b) L-d0pa
b) genital lesion Ans. A
c) skin 12) Outer quadrant of breast drains
Ans. B a) anterior axilary ln
198. During episotomy muscle cut b) posterior axilary ln
a) Bulbospongious c) apical axilary ln
b) Levator anni d) central ln
c) Perneal membrane Ans. A
Ans. A 13) Majority lymph parralel to vessels in breast
199.A 14 years old boy with creatinine 8 bp 180/110 a) axilary along axillary vessels
stucture involved is b) internal along internal thoracic to parasternal
a) Jg cells Ans: B
b) Glomerulus 14) ST segment elevation in lead II which artery
Ans. A effected?
200.UV prolapsed most common complication? a) left main coronary
a) Discharge b) left marginal
b) Chronic cervicitis and metaplsia c) right main coronary
Ans. B d) left obtuse
e) left circumflex
Radilogy May 3rd Morning Ans: C
1. Infection in superficial perineal pouch will effect 15) Continous pain with fever and rustle sound on
a) Deep transverse perinoei apex area of chest on auscultation
b) Greater vestibular gland a) pericarditis
Ans: A b) pleural
2. Inferior orbital artery is terminal branch of c) myocarditis
Ans: A
Ans. Maxillary artery
16) Left umblical artery remanant
Ans: Medial umbilical ligament Ans. A
17) Ant to 2nd part of duodenum 32) Cystic artery arises from
a) Rt ureter Ans: Rt hepatic
b) transverse colon 33) Whuch when goes directly into svx
Ans: B a) Azygous
18) Malignancy on scortal skin lymph will go to b) Hemiazygous
Ans: superficial inguinal Ans. A
19) Glans and corpora infection lymph drainage 34) Associated with heart disease (coronary risk)
Ans: deep inguinal a) ldl -C
20) Fibrillin is defective in b) plasma cholestrol
Ans. Marfan syndrome c) triglycerides
21) Prostate lymph drainage d) hdl
a. internal iliac Ans. A
b. internal plus external iliac 35) Long thoracic suplies.
Ans: A Ans: serratus ant
22) Lateral of ant 2/3 tongue lymph will first go to 36) Last to ossify in humerus
a) submental a) lateral epicondyl
b) jugulodiagastric b) medial epicondyl
c) submandibular c) capitulum
Ans. C d) diaphysis
23) Muscle with one motor supply e) epiphysis
a. diagastric Ans. A
b. addoctor longus 37) Intestines appear as segements these are
c. diaphgram called
Ans. C a) tenia coli
24) Immunosuppressants common side effect b) haustra
a) Inc in pyogenic Inf. Ans: B
b) Dec in lymphocytes 38) Can not raise hand above shoulder joint..
Ans. B Lesion in
25) Superior boundry of superficial perineal pouch a) Serratus ant.
Ans: perineal membrane b) supraspinatus
26) Child with pedal and periorbital edema cause of c) lattismus dorsi
edema. (Trepizius NHI tha in options)
Ans. Na retention and Albuminuria Ans: A
27) Superficial cardiac plexsus are made by 39) 4th cranial nerve lesion causes prblm in
a) right vagus a) downward gaze and addction
b) left vagus b) upward gaze and abduction
Ans. B Ans. A
28) Tall male gynaecomastia .no/ scanty pubic hair. 40) Dvt occurs beacuse of
a) Fagile-X syndrome a) Surgery
b) Kleinfelter b) Prolonged immobilisation
c) Downs Ans. B
Ans. B 41) Wat makes intestines luk as if they are divided
29) GVE nucleus of vagus nerve in.. into segments
a) Dorsal motor nucleus a) Huastra
b) Nucleus ambiguous b) Taenia colli
Ans. A c) Flexures
Ans: A (similar question resembling same
30) Patient sustained injury to s234 wat will happen
statement asked twice)
a) Rectal incontinence.....
b) No pain in menstruation 42) Wat is most appropriate about spleen
c) Painless labor a) Has a thick fibrous capsule
Ans. A b) Grows towards left iliac fossa
c) accesary spleen if present commonly found
31) What structure is associated with of disease
in tail of pancrease
if pancreas
Ans: C
a) Bile duct
b) Duodenum 43) Successful stellate ganglion block wat is least
c) Liver . likely to occur
a) Nasal congestion
b) Pupillary constriction 53) About fetal head ....asked abt post frontelle
c) Anhudrosis a) valut is bigger than face
d) unilateral arm asthesia b) Three bone occupies it
e) Horasnsess of voice Ans. A
Ans. A 54) Major inspiratory muscle in breathing
44) Child cycling uphill presented to e.r hasnt Ans: diaphgram
passed urine.lesion is most likely in 55). stem was like...which produces high energy
a) Kidney phosphate bonds/compound.
b) Ureters a) 1mole galactose
c) Bladder b) 1mole glucose
d) Pelviureteric junction c) 1 mole long chain fatty acids
e) Urethra Ans. C
Ans: E 56) respiration duration
45) Part of brain receive excitory input from Ans. 2 sec inspiration 3 sec expiration
cerebral cortex but does not send it back 57) Systemic cause of poor wound healing
a) Cuadate a) diabetes militus
b) Substansia nigra b) Vit c deficiency
c) Purkinji of cerebellum Ans: A
Ans: A 58) Under tension what prevents
46) Which is involved in planing and organisation tearing/overstretching.
of movements. a) GTO
a) Neurocrebellum and basal ganglia b) Muscle spindle
b) spinocerebellum Ans. A
c) Cortico spinal and cirtiobulbar tracts 59) superior to piriformis what is injured
d) Motor cortex ... a) superior gluteal artery
e) Basal ganglia b) sciatic nerve
Ans: A Ans: A
47) Trauma case all muscles of Abd nd chest 60) If one leg is raised in air which muscle
paralyzed stil breathing. Which level injury stablizes pelvis
occurred? Ans: gluteus medius
a) c4
61) In adrenal medulla what receptors are
b) c7
present...
c) t4
a) nicotinic,
d) t11
b) muscarinic ,
Ans: C
c) beta adrenergic,
48) Anterior gap between inferior rami pubic bone d) alpha adrenergic
is filled by Ans. A
a) levator ani
62) About pitutary gland
b) urogenital diaphgram
a) above diaphramatic sella
Ans: B
b) abnrmality causes junctional scotoma
49) Defecation mainly carried by c) lateral to cavernous sinus
a) mass movement Ans. C
b) parasympathetic reflex
63) Greatest Numb of sodium channels in which
c) Anorectal reflex
part of axon
Ans: B
a) internodal
50) Cremastric afferent reflex fibers b) where axon leaves
a) ilioinguinal nerve Ans: A (node of ranvier nhi tha in option)
b) iliohypogastric
64) All factors are used up in clotting except
c) genitofemoral
a) factor 2
Ans. A
b) factor 5
51) Surgicaly breast lobes are c) faactor 7
a) indepandent territories Ans. B
b) separated by stroma
65) Teratogtogensis in which week
Ans: B
Ans. 3-8weeks
52) Common cause of osteoprosis
66) Patient on TPN will develop
a) old age
Ans: hyperglycemia
b) post menopause
Ans: B 67) A lady in anesthesia aspired fluid.. It would b
best prevented by
a) approximation of false vocal cord 5. Innermost intercostal muscle
b) approximation of true vocal cords 6. Endothoracic Simpsons fascia
c) ant bent of larynx 7. Parietal pleura
d) backward falling of tongue 8. Visceral pleural.
Ans. A 79) Pt recovered from meningitis now presented
68) Smoker with nodules on vocal cords will least with papilledema cause
likely hav Ans: hydrocephalus
Ans: atrophy 80) Works through cGMP
69) Common cause of non bacterial endocarditis Ans: ANP
a) Sle 81) Major bulk of semen
b) terminal cancer pt Ans: seminal vesicles
Ans. B 82) Injury at elbow ulnar nerve damage
70) Most common cause of subacute Ans: loss of action of 3rd 4th lumbricals
endocarditis 83) Study btw diseased and non- diseased and
Ans: strptococus viridian looking into their history for risk factor exposure
71) Carotid sinus detects changes in arterial Ans: case control
blood pressure 84) Fracture of shaft humerus artery injured
Ans: Barorecptors a) profunda brachii
72) Neutophilis production stimulated by b) brachial artery
a) Granulocyte colony stimulating factor Ans: A
b) lL 8 85) Primary prevention against dental caries
c) lL 2 Ans: Proper tooth brushing technique
d) IL 1 86) A doctor is good if he is
Ans. A a) honest
73) Cervical enlargement is due to b) realistic
Ans: C5 to T1 c) up-to-date with knowledge
74) Ventral rami of c3 c4 form d) listens to pt carefully
a) Greater auricular nerve e) respects pt decision regarding treatment
b) Nerve occipitis capitis Ans: E
c) Supraclavicular Nerves 87) Cannabis found in urine of a police officer.
d) Transverse cervical.. Behavior he will show......
Ans: C Ans: denial
75) Pt. can close eyes but rest of right 88) Patient unable to fold arm and scratch the
Body hemiplegic which part ofinternal back of the body muscle involved
Capsule has lesion a) Latissimus dorsi
a) ant part of posterior limb b) Subscapularis
b) ant part of post limb plus genu c) Tere minor
Ans: A d) Supraspinatus
76) 7 days after birth juandice diaper wet typical Ans: A
scenario of 89) Excessive eversion injury caused by which
Ans: billary atresia ligament
77) Microscopic feature of malignancy a) Deltoid
a) N/C ratio b) Talonavicular
b) pleomorphism c) Tendo calcaneus
c) desmoplasia Ans. A
Ans. B (invasion metastasis was not in option) 90) Deltoid weakness, reduced biceps jerk,
78) A stab wound start at and including external numbness of thumb - which nerve root
intercostal muscle and (parietal plueral and Ans: C5, C6
pleural cavity as 2 layers) piereces how many 91) Ceroplasmin and C - reactive protein are
layers Ans: Acute Phase Proteins
a) 4 92) Superficial cardiac plexus nerve endings
b) 5 related question
c) 6 93) Complication of measles
d) 7 Ans: subacute sclerosing encephalitis
e) 8
94) Causing cooling of body
Ans: E
a) Preoptic area of hypothalamus
1. Skin
b) post thalamus
2. Superficial fascia c) thalamus
3. External intercostal muscle d) mammillary body
4. Internal intercostal muscle Ans: A
95) Arches over root of right lung 110) hematoma in broad ligament
a) azygous a) ovarian artery
b) phrenic b) uterine artery
Ans. A Ans: B
96) Defense against melanoma 111) vague initial referred pain from appendicitis
a) cytotoxic t cells a) umbilicus
b) natural killer cells b) Rt flank
Ans: A Ans: A
97) Psuedostratified epithelium 112) main artery of subclavian which takes part in
Ans. all cell base are at membrane shoulder joint anastomosis
98) Selective reabsorption of substances occurs a) subscapular
a) renal papIa b) Thyrocervical
b) tubules and cortical ducts Ans. A/ B
c) glomerulus corpuscles 113) meningitis turbid CSF lymphocytes 75%
Ans: B while neutrophils 25%
99) If a substance is found in renal artery but not Ans. Tuberculous meningitis
in renal vein 114) portal anastomoses at
a) clearance is equal to GFR a) lower anal canal
b) secreted by tubules b) lower esophagus
c) freely filtered Ans. B
Ans: A 115) most superficial in parotid gland
100) Foramen caecum is embryological site of Ans: facial nerve
origin of 116) premalignant lesion
a) Anterior pituitary a) intradermal nevus
b) Posterior pituitary b) solar keratosis
c) Superior parathyroid c) Submucosal fibrosis
d) Thyroid Ans. B
e) Thymus
117) raised ulcer on cheek
Ans: D Ans. actinic keratosis
101) lobe of lung with 2 segments 118) superior rectal artery is continuation of
Ans. right middle lobe Ans. inferior mesenteric artery
102) transversus abdominus muscle 119) seen in acromegaly.
aponeurosis forms a) chondrocalcinosis
a) conjoint tendon b) vertebral scalloping
b) external spermatic fascia Ans. B
Ans: A 120) typical scenario of Achondroplasia
103) Diff between simple and facilitated diffusion Ans: Autosomal Dominant
Ans: V-max 121) cerebral hemorrhage with renal scenario
104) benign growth Ans: autosomal dominant
Ans: warthin tumors 122) superficial external pudendal emerges from
105) patient drinks 2 L water after two liter Ans. Saphenous Opening
sweating 123) a thrombus in artery to retina will do
Ans: Inc ECF volume a) Liquefactive Necrosis
106) volatile acids in respiratory acidosis are b) infarction
neutralized by c) hemorrhage
a) hyperventilation Ans: B
b) bicarbonate sys of kidney
124) menopause
c) ammonia acid system of kidney
Ans: A a) ovarian failure
b) inc FSH Dec estrogen and Dec LH
107) severe diarrhea c) inc FSH and inc LH and Dec estrogen
a) inc osmolarity of ECF Ans: C
b) metabolic alkalosis
125) most common cause of septic meningitis
c) Dec sodium in ECF
Ans. C a) streptococcus
b) legionella
108) temporal lobe part in medial side c) Klebsiella
a) Para hippocampus Ans. A
b) Cingulum
126) a polyp in endo cervix shows stratified
Ans: B squamous epithelium.it is
109) adrenal cortex arises from a) hyperplasia
Ans: mesoderm b) Metaplasia.
Ans. B 148) in jogging individual most likely
127) injury to a structure near mandibular Mechanism of venous return
canal will cause a) Contraction of leg muscles
a) inferior gum anesthesia b) Vasoconstriction
b) all sensation lost from ant 2/3 of tongue Ans. A
Ans: A 149) Scenario of sarcoidosis with rash on
128) descending thoracic aorta begins at lower forearms hilar nodes noncaseating granuloma
border of Ans: Sarcoidosis
a) t4 150) narrowest point of airway in child
b) t3 Ans: cricoid
Ans: A 151) Homonymous hemianopia lesion in
129) middle part of CBD a) Geniculocalcarine tract
a) on left of hepatic vein b) Optic chiasm
b) in hepatoduodenal ligament c) Optic tract
c) Behind duodenum d) Visual cortex
Ans. C e) Optic nerve
130) Type of autonomic preganglionic fibers Ans: C
Ans: b fibers 152) Old women hypersegmented neutrophils
131) after 6 hrs of lithotomy position for labor a raised mcv anemic
lady can't dorsiflex ankles. Which a) Pernicious ans
nerve? b) Anaemia of chronic disease
Ans. common peroneal c) Hemolytic anemia
132) moon face truncal obesity what else will be Ans. A
found 153) what immunoglobulin involved in bee sting
Ans. hyperglycemia allergy scenario Ig involved?
134) Heinz bodies a) IgE
Ans. G6PD Deficiency b) IgG
135) spleen rupture Ans. A
Ans. Howell jowly 154) True about thyroid
136) bulbourethral gland present in a) Enclosed in pretracheal fascia
Ans. deep perineal pouch b) Middle thyroid is the main blood supply
Ans. A
137) Brunner’s gland
Ans. submucosa duodenum 155) what will cause severe reaction
a) A to o
138) Vit def with pancreatic disease
b) to a
Ans. fat soluble vitamins
c) A + to A -
139) feco-oral transmission Ans. A
Ans. Hep E 156) what is the functional unit of liver called
140) immune-compromised meningitis is a) Hepatic acinus
Ans. Cryptococcus b) Hepatic sinusoid
141) Pus in adductor canal compress which c) Hepatocyte
structures??? Ans. A
Ans. femoral vein 157) about blood in veins
142) 2 very difficult scenarios from venous a) chloride is less outside
drainage of heart and nerve Endings of superficial b) Size of Rbc in veins is greater than in arteries
cardiac plexus c) co2 is carried as Carboxyhemoglobin
143) connecting lesser curvature to liver is Ans: B
a) gastro hepatic ligament 158) right sided facial palsy with right sided
b) ligament teres hemiplegia internal capsule lesion scenario it was
Ans. A asked that which artery is responsible for this
144) nasal lesions with hematuria area blood supply
Ans: Wegener’s a) branches of middle cerebral
145) body defense against tumors b) branches of ant cerebral
Ans. apoptosis c) rest of the options i forgot
146) chromosome shortening Ans: A
Ans: telomerase 159) least nutritious
147) ECG recording is during a. cereal
a) complete depolarization b. tubers
b) complete repolarization Ans: B
c) Partly depolarization partly repolarization. 160) Drugs with short plasma half life
Ans. C a) Usually have low therapeutic index
b) Achieve steady plasma concentration quickly Ans. increased venous return causes increased cardiac
c) usually strongly bound to plasma proteins output
Ans. B 177) Tall T wave
161) Primary hyperparathyroidism most Ans: hyperkalemia
common in 178) Plasmodium Falciparum
a) Young females Ans. Black water fever
b) Young males 179) 90% anthrax
c) post menopause Ans: skin lesion
d) Adolescents 180) Circumduction
Ans. C Ans: Flexion + Extension + Abduction + Adduction
162) metastasis from uterus to labia majora is
through 181) Sickle Cell disease due to defect in
Ans: Round Ligament of Uterus Ans: Beta globulin
163) anatomy dissection student faint 182) Penicillin allergy profound effect...
a) hypotension A. rash over body
b) loss of vasomotor tone b. Bronchospasm
Ans: B (1st effect is rash while profund effect/sever
Ans: B
effect is bronchospasm)
164) elbow bleeding 1st response
183) slowest growing tumor of Thyroid
Ans. vasoconstriction
a) Papillary
165) uterus prolapse in introitus ligament damage b) Follicular
Ans: Uterosacral c) Medullary
166) regarding great cardiac vein starts at apex Ans: A
Ans. runs in AV groove with LCX to open in coronary 184) Nasopharyngeal Ca caused by
sinus Ans: EBV
167) oxygen unbinding in tissue does not occur 185) most common cause of Fat Necrosis
in a) Trauma
a) cyanide poisoning b) Enzymes
b) carbon monoxide Ans: A
Ans. B
187. Pulse oximiter giving Wrong value
168) H+ ion secretion Inc in a) Inc HbF
a) hypokalemia b) dark color
b) carbonic anhydrase inhibitors c) Methohemboglobinmia
c) Inc Ph. Ans:
Ans: A
188. One scenario of b12 anemia
169) antibody which works best with compliment Ans: Pt have loss of sense of vibration
sys for Opsonization 189. Pt chew beetle nut for years cant open mouth
a) IgG due to ulcer one Cause
b) IgM Ans. submucosal fibrosis
Ans: A
170) dysarthria dysphagia artery wala typical
scenario
Ans. PICA
171) neural tube forms
a) ant and post neural pores
b) primitive streak
Ans: A
172) brain tissue exposed to irritation by
amniotic fluid
a) syntosis
b) meningomyocele
c) anencephaly
Ans: B
173) renin is released
Ans. sypmapethic nerves stimulation
174) At L1
Ans: SMA originates
175) Left circumflex supplies.
Ans. left atrium and ventricle
176) Starling Law

You might also like